You are on page 1of 196

Foreign Licensing Examination

DRILL BOOK

TABLE OF CONTENTS
Phase I:
1. Orientation on Foreign Exams
2. Test Taking Strategies
3. Foundations of Nursing
Phase II: Medical and Surgical Nursing
1. Neurovascular and Sensory System
2. Respiratory and Hematology
3. Cardiovascular System
4. Gastrointestinal System
5. Renal with Fluids and Electrolytes
6. Musculoskeletal System
7. Oncology with Pain Management
8. Endocrine System
Phase III. Women and Child's Health
1. Maternal Nursing
2. Pediatric Nursing
Phase IV: Special Nursing Topics
1. Emergency and Disaster Nursing
2. Transcultural Nursing
3. Nursing Issues
4. Communicable Diseases
5. Perioperative Nursing
6. Geriatrics
7. Nutrition
8. Delegation and Legal Nursing
*Pharmacology is included in every concepts.

PART I

FUNDAMENTALS
OF NURSING

1. When examining a patient with abdominal pain the nurse in charge should assess:
A. Any quadrant first
B. The symptomatic quadrant first
C. The symptomatic quadrant last
D. The symptomatic quadrant either second or third
CORRECT ANSWER: C.
The nurse should systematically assess all areas of the abdomen, if time and the patients
condition permit, concluding with the symptomatic area. Otherwise, the nurse may elicit pain in
the symptomatic area, causing the muscles in other areas to tighten. This would interfere with
further assessment.
2. A female patient with a terminal illness is in denial. Indicators of denial include:
A. Shock dismay
B. Numbness
C. Stoicism
D. Preparatory grief
CORRECT ANSWER: A.
Shock and dismay are early signs of denial-the first stage of grief. The other options are
associated with depressiona later stage of grief.
3. To evaluate a patient for hypoxia, the physician is most likely to order which laboratory
test?
A. Red blood cell count
B. Sputum culture
C. Total hemoglobin
D. Arterial Blood Gas (ABG) analysis
CORRECT ANSWER: D.
All of these test help evaluate a patient with respiratory problems. However, ABG analysis is the
only test evaluates gas exchange in the lungs, providing information about patients oxygenation
status.
4. An employer establishes a physical exercise area in the workplace and encourages all
employees to use it. This is an example of which level of health promotion?
A. Primary Prevention
B. Secondary Prevention
C. Tertiary Prevention
D. Passive Prevention
CORRECT ANSWER: A.
Primary prevention precedes disease and applies to health patients. Secondary prevention
focuses on patients who have health problems and are at risk for developing complications.
Tertiary prevention enables patients to gain health from others activities without doing anything
themselves.
5. Which nursing action is essential when providing continuous enteral feeding?
A. Elevating the head of the bed
B. Positioning the patient on the left side
C. Warming the formula before administering it
D. Hanging a full days worth of formula at one time

CORRECT ANSWER: A.
Elevating the head of the bed during enteral feeding minimizes the risk of aspiration and allows
the formula to flow in the patients intestines. When such elevation is contraindicated, the patient
should be positioned on the right side. The nurse should give enteral feeding at room
temperature to minimize GI distress. To limit microbial growth, the nurse should hang only the
amount of formula that can be infused in 3 hours.
6. Which action by the nurse in charge is essential when cleaning the area around a JacksonPratt wound drain?
A. Cleaning from the center outward in a circular motion
B. Removing the drain before cleaning the skin
C. Cleaning briskly around the site with alcohol
D. Wearing sterile gloves and mask
CORRECT ANSWER: A.
The nurse always should clean around a wound drain, moving from center outward in everlarger circles, because the skin near the drain site is more contaminated than the site itself. The
nurse should never remove the drain before cleaning the skin. Alcohol should never be used to
clean around a drain; it may irritate the skin and has no lasting effect on bacteria because it
evaporates. The nurse should wear sterile gloves to prevent contamination, but a mask is not
necessary.
7. The doctor orders dextrose 5% in water, 1,000 ml to be infused over 8 hours. The I.V.
tubing delivers 15 drops per milliliter. The nurse in charge should run the I.V. infusion at
a rate of:
A. 15 drops per minute
B. 21 drops per minute
C. 32 drops per minute
D. 125 drops per minute
CORRECT ANSWER: C.
Giving 1,000 ml over 8 hours is the same as giving 125 ml over 1 hour (60 minutes) to find the
number of milliliters per minute: 125/60 min = X/1 minute 60X = 125X = 2.1 ml/minute To find
the number of drops/minute: 2.1 ml/X gtts = 1 ml/15 gtts X = 32 gtts/minute, or 32 drops/minute.
8. A female patient undergoes a total abdominal hysterectomy. When assessing the patient
10 hours later, the nurse identifies which finding as an early sign of shock?
A. Restlessness
B. Pale, warm, dry skin
C. Heart rate of 110 beats per minute
D. Urine output of 30ml per hour
CORRECT ANSWER: A.
Early in shock, hyperactivity of the sympathetic nervous system causes increased epinephrine
secretion, which typically makes the patient restless, anxious, nervous, and irritable. It also
decreases tissue perfusion to the skin, causing pale, cool clammy skin. An above-normal heart
rate is a late sign of shock. A urine output of 30 ml/hour is within normal limits.
9. Which pulse should the nurse palpate during rapid assessment of an unconscious male
adult?
A. Radial

B. Brachial
C. Femoral
D. Carotid

CORRRECT ANSWER: D.
During a rapid assessment, the nurses first priority is to check the patients vital functions by
assessing his airway, breathing, and circulation. To check a patients circulation, the nurse must
assess his heart and vascular network function. This is done by checking his skin color,
temperature, mental status and, most importantly, his pulse. The nurse should use the carotid
artery to check a patients circulation. In a patient with a circulatory problems or a history of
compromised circulation, the radial pulse may not be palpable. The brachial pulse is palpated
during rapid assessment of an infant.
10. Which statement regarding heart sounds is correct?
A. S1 and S2 sound equally loud over the entire cardiac area.
B. S1 and S2 sound fainter at the apex
C. S1 and S2 sound fainter at the base
D. S1 is loudest at the apex, and S2 is loudest at the base
CORRECT ANSWER: D.
The S1 soundthe lub soundis loudest at the apex of the heart. It sounds longer, lower, and
louder there than the S2 sounds. The S2the dub soundis loudest at the base. It sounds
shorter, sharper, higher, and louder there than S1.
11. A female patient is receiving furosemide (Lasix), 40 mg P.O. b.i.d. in the plan of care, the
nurse should emphasize teaching the patient about the importance of consuming:
A. Fresh, green vegetables
B. Bananas and oranges
C. Lean red meat
D. Creamed corn
CORRECT ANSWER: B.
Because furosemide is a potassium-wasting diuretic, the nurse should plan to teach the patient to
increase intake of potassium-rich foods, such as bananas and oranges. Fresh, green vegetables;
lean red meat; and creamed corn are not good sources of potassium.
12. A female patient is diagnosed with deep-vein thrombosis. Which nursing diagnosis
should receive highest priority at this time?
A. Impaired gas exchanges related to increased blood flow
B. Fluid volume excess related to peripheral vascular disease
C. Risk for injury related to edema
D. Altered peripheral tissue perfusion related to venous congestion
CORRECT ANSWER: D.
Altered peripheral tissue perfusion related to venous congestion takes highest priority because
venous inflammation and clot formation impede blood flow in a patient with deep-vein
thrombosis. Option A is incorrect because impaired gas exchange is related to decreased, not
increased, blood flow. Option B is inappropriate because no evidence suggest that this patient
has a fluid volume excess. Option C may be warranted but is secondary to altered tissue
perfusion.
13. When positioned properly, the tip of a central venous catheter should lie in the:

A.
B.
C.
D.

Superior vena cava


Basilic vein
Jugular vein
Subclavian vein

CORRECT ANSWER: A.
When the central venous catheter is positioned correctly, its tip lies in the superior vena cava,
inferior vena cava, or the right atriumthat is, in central venous circulation. Blood flows
unimpeded around the tip, allowing the rapid infusion of large amounts of fluid directly into
circulation. The basilica, jugular, and subclavian veins are common insertion sites for central
venous catheters.
14. A female client is readmitted to the facility with a warm, tender, reddened area on her
right calf. Which contributing factor would the nurse recognize as most important?
A. A history of increased aspirin use
B. Recent pelvic surgery
C. An active daily walking program
D. A history of diabetes
CORRECT ANSWER: B.
The client shows signs of deep vein thrombosis (DVT). The pelvic area is rich in blood supply,
and thrombophlebitis of the deep vein is associated with pelvic surgery. Aspirin, an antiplatelet
agent, and an active walking program help decrease the clients risk of DVT. In general, diabetes
is a contributing factor associated with peripheral vascular disease.
15. Which intervention should the nurse in charge try first for a client that exhibits signs of
sleep disturbance?
A. Administer sleeping medication before bedtime
B. Ask the client each morning to describe the quantity of sleep during the previous
night
C. Teach the client relaxation techniques, such as guided imagery, medication, and
progressive muscle relaxation
D. Provide the client with normal sleep aids, such as pillows, back rubs, and snacks
CORRECT ANSWER: D.
The nurse should begin with the simplest interventions, such as pillows or snacks, before
interventions that require greater skill such as relaxation techniques. Sleep medication should be
avoided whenever possible. At some point, the nurse should do a thorough sleep assessment,
especially if common sense interventions fail.
16. While examining a clients leg, the nurse notes an open ulceration with visible
granulation tissue in the wound. Until a wound specialist can be contacted, which type of
dressings is most appropriate for the nurse in charge to apply?
A. Dry sterile dressing
B. Sterile petroleum gauze
C. Moist, sterile saline gauze
D. Povidone-iodine-soaked gauze
CORRECT ANSWER: C.
Moist, sterile saline dressings support would heal and are cost-effective. Dry sterile dressings

adhere to the wound and debride the tissue when removed. Petroleum supports healing but is
expensive. Povidone-iodine can irritate epithelial cells, so it shouldn't be left on an open wound.
17. A male client is on prolonged bed rest has developed a pressure ulcer. The wound shows
no signs of healing even though the client has received skin care and has been turned
every 2 hours. Which factor is most likely responsible for the failure to heal?
A. Inadequate vitamin D intake
B. Inadequate protein intake
C. Inadequate massaging of the affected area
D. Low calcium level
CORRECT ANSWER: B.
A client on bed rest suffers from a lack of movement and a negative nitrogen balance. Therefore,
inadequate protein intake impairs wound healing. Inadequate vitamin D intake and low calcium
levels arent factors in poor healing for this client. A pressure ulcer should never be massaged.
18. A 73-year-old patient who sustained a right hip fracture in a fall requests pain
medication from the nurse. Based on his injury, which type of pain is this patient most
likely experiencing?
A. Phantom
B. Visceral
C. Deep somatic
D. Referred
CORRECT ANSWER: C.
Deep somatic pain originates in ligaments, tendons, nerves, blood vessels, and bones.
Therefore, a hip fracture causes deep somatic pain. Phantom pain is pain that is perceived to
originate from a part that was removed during surgery. Visceral pain is caused by deep
internal pain receptors and commonly occurs in the abdominal cavity, cranium, and thorax.
Referred pain occurs in an area that is distant to the original site.
19.The nurse assesses clients' breath sounds. Which one requires immediate
medical attention? A client who has:
A. Crackles
B. Rhonchi
C. Stridor
D. Wheezes

CORRECT ANSWER: C
Stridor is a sign of respiratory distress, possibly airway obstruction. Crackles and
rhonchi indicate fluid in the lung; wheezes are caused by narrowing of the airway.

Crackles, rhonchi, and wheezes indicate respiratory illness and are potentially
serious but do not necessarily indicate respiratory distress that requires
immediate medical attention.

20.A client with a stage 2 pressure ulcer has methicillin-resistant


Staphylococcus aureus (MRSA) cultured from the wound. Contact
precautions are initiated. Which rule must be observed to follow contact
precautions?
A. A clean gown and gloves must be worn when in contact with the
client.
B. Everyone who enters the room must wear a N-95 respirator mask.
C. All linen and trash must be marked as contaminated and send to
biohazard waste.
D. Place the client in a room with a client with an upper respiratory
infection.

CORRRECT ANSWER: A.
A clean gown and gloves must be worn when any contact is anticipated with the
client or with contaminated items in the room. A respirator mask is required only
with airborne precautions, not contact precautions. All linen must be doublebagged and clearly marked as contaminated. The client should be placed in a
private room or in a room with a client with an active infection caused by the
same organism and no other infections.

21.A client requires protective isolation. Which client can be safely paired
with this client in a client-care assignment?
A. Admitted with unstable diabetes mellitus.
B. Who underwent surgical repair of a perforated bowel.
C. With a stage 3 sacral pressure ulcer.
D. Admitted with a urinary tract infection.

CORRECT ANSWER: A.
The client with unstable diabetes mellitus can safely be paired in a client-care
assignment because the client is free from infection. Perforation of the bowel
exposes the client to infection requiring antibiotic therapy during the
postoperative period. Therefore, this client should not be paired with a client in
protective isolation. A client in protective isolation should not be paired with a
client who has an open wound, such as a stage 3 pressure ulcer, or with a client
who has a urinary tract infection.

22.A mother who breastfeeds her child passes on which antibody through
breast milk?
A. IgA
B. IgE
C. IgG
D. IgM

CORRECT ANSWER: C.
The antibody IgG is passed to the child through the mother's breast milk during
breastfeeding. IgA, IgE, and IgM are produced by the child's body after exposure
to an antigen.

23.The nurse is assessing a patient admitted to the hospital with rectal


bleeding. The patient had a hip replacement 2 weeks ago. Which position
should the nurse avoid when examining this patient's rectal area?
A. Sims'
B. Supine
C. Dorsal recumbent
D. Semi-Fowler's

CORRECT ANSWER: A
Sims' position is typically used to examine the rectal area. However, the position
should be avoided if the patient has undergone hip replacement surgery The
patient with a hip replacement can assume the supine, dorsal recumbent, or semiFowler's positions without causing harm to the joint. Supine position is lying on
the back facing upward. The patient in dorsal recumbent is on his back with knees
flexed and soles of feet flat on the bed. In semi-Fowler's position, the patient is
supine with the head of the bed elevated and legs slightly elevated.

24.The nurse performs an initial abdominal assessment on a patient newly


admitted for abdominal pain. The nurse hears what she describes as "clicks
and gurgles in all four quadrants" as well as "swishing or buzzing sound
heard in one or two quadrants." Which of the following statements is
correct?
A. The frequency and intensity of bowel sounds varies depending on the
phase of digestion.
B. In the presence of intestinal obstruction, bowel sounds will be louder
and higher pitched.
C. A swishing or buzzing sound may represent the turbulent blood flow
of a bruit and is not normal.
D. All of the above.
CORRECT ANSWER IS D.
All of the statements are true. The gurgles and clicks described in the question
represent normal bowel sounds, which vary with the phase of digestion. Intestinal
obstruction causes the sounds to intensify as the normal flow is blocked by the
obstruction. The swishing and buzzing sound of turbulent blood flow may be heard
in the abdomen in the presence of abdominal aortic aneurism, for example, and
should always be considered abnormal.
25.A nurse is performing routine assessment of an IV site in a patient receiving
both IV fluids and medications through the line. Which of the following would
indicate the need for discontinuation of the IV line as the next nursing action?
A. The patient complains of pain on movement.

B. The area proximal to the insertion site is reddened, warm, and painful.
C. The IV solution is infusing too slowly, particularly when the limb is
elevated.
D. A hematoma is visible in the area of the IV insertion site.
CORRECT ANSWER IS B.
An IV site that is red, warm, painful and swollen indicates that phlebitis has
developed and the line should be discontinued and restarted at another site. Pain
on movement should be managed by maneuvers such as splinting the limb with an
IV board or gently shifting the position of the catheter before making a decision to
remove the line. An IV line that is running slowly may simply need flushing or
repositioning. A hematoma at the site is likely a result of minor bleeding at the time
of insertion and does not require discontinuation of the line.

PART II

MEDICAL & SURGICAL


NURSING

NEUROVASCULAR
&
SENSORY SYSTEM

1. For a client having an episode of acute narrow-angle glaucoma, a nurse expects to give
which of the following medications?
A. Acetazolamide (Diamox)
B. Atropine
C. Furosemide (Lasix)
D. Urokinase (Abbokinase)
CORRECT ANSWER: A.
Acetazolamide, a carbonic anhydrase inhibitor, decreases intraocular pressure (IOP) by
decreasing the secretion of aqueous humor. Atropine dilates the pupil and decreases outflow of
aqueous humor, causing further increase in IOP. Lasix is a loop diuretic, and Urokinase is a
thrombolytic agent; they arent used for the treatment of glaucoma.
2. A client with Menieres disease is experiencing severe vertigo. Which instruction would
the nurse give to the client to assist in controlling the vertigo?
A. Increase fluid intake to 3000 ml a day
B. Avoid sudden head movements
C. Lie still and watch the television
D. Increase sodium in the diet
CORRECT ANSWER: B.
The nurse instructs the client to make slow head movements to prevent worsening of the vertigo.
Dietary changes such as salt and fluid restrictions that reduce the amount of endolymphatic fluid
sometimes are prescribed. Lying still and watching television will not control vertigo.

3. The nurse is reviewing the physicians orders for a client with Menieres disease. Which
diet will most likely be prescribed?
A. Low-cholesterol diet
B. Low-sodium diet
C. Low-carbohydrate diet

D. Low-fat diet
CORRECT ANSWER: B
Dietary changes such as salt and fluid restrictions that reduce the amount of endolymphatic fluid
sometimes are prescribed.
4. Cataracts result in opacity of the crystalline lens. Which of the following best explains the
functions of the lens?
A. The lens control stimulation of retina
B. The lens orchestrates eye movement
C. The lens focuses light rays on the retina
D. The lens magnifies small objects
CORRECT ANSWER: C.
The lens allow light to pass through the pupil and focus light on the retina. The lens does not
stimulate the retina, assist with eye movement or magnify small objects, so answers A, B, and D
are incorrect.
5. During the early postoperative period, the client who had a cataract extraction complains
of nausea and severe eye pain over the operative site. The initial nursing action is to:
A. Call the physician
B. Administer the ordered main medication and antiemetic
C. Reassure the client that this is normal.
D. Turn the client on his or her operative side
CORRECT ANSWER: A.
Severe pain or pain accompanied by nausea is an indicator of increased intraocular pressure
and should be reported to the physician immediately. The other options are inappropriate.

6. The client is being discharged from the ambulatory care unit following cataract removal.
The nurse provides instructions regarding home care. Which of the following, if stated by
the client, indicates an understanding of the instructions?
A. I will take Aspirin if I have any discomfort.
B. I will sleep on the side that I was operated on.
C. I will wear my eye shield at night and my glasses during the day.

D. I will not lift anything if it weighs more than 10 pounds.


CORRECT ANSWER: C.
The client is instructed to wear a metal or plastic shield to protect the eye from accidental and is
instructed not to rub the eye. Glasses may be worn during the day. Aspirin or medications
containing aspirin are not to be administered or taken by the client and the client is instructed to
take acetaminophen as needed for pain. The client is instructed not to sleep on the side of the
body on which the operation.

7. Which of the following signs and symptoms of increased ICP after head trauma would
appear first?
A. Bradycardia
B. Large amounts of very dilute urine
C. Restlessness and confusion
D. Widened pulse pressure
CORRECT ANSWER: C.
The earliest symptom of elevated ICP is a change in mental status. Bradycardia, widened pulse
pressure, and bradypnea occur later. The client may void large amounts of very dilute urine if
theres damage to the posterior pituitary.
8. The client with glaucoma asks the nurse is complete vision will return. The most
appropriate response is:
A. Although some vision has been lost and cannot be restored, further loss may be
prevented by adhering to the treatment plan.
B. Your vision will return as soon as the medications begin to work.
C. Your vision will never return to normal.
D. Your vision loss is temporary and will return in about 3-4 weeks.
CORRECT ANSWER: A.
Vision loss to glaucoma is irreparable. The client should be reassured that although some vision
has been lost and cannot be restored, further loss may be prevented by adhering to the treatment
plan. Option C does not provide reassurance to the client.

9. A client experiences loss of consciousness, tongue biting, and incontinence, along with
tonic and clonic phases of seizure activity. The nurse should document this episode as
which type of seizure?
A. Jacksonian
B. Absence
C. Generalized
D. Sensory
CORRECT ANSWER: C.
A generalized seizure causes generalized electrical abnormality in the brain. The client typically
falls to the ground, losing consciousness. The body stiffens (tonic phase) and then alternates
between episodes of muscle spasm and relaxation (clonic phase). Tongue biting, incontinence,
labored breathing, apnea, and cyanosis may also occur. A Jacksonian seizure begins as a
localized motor seizure. The client experiences a stiffening or jerking in one extremity,
accompanied by a tingling sensation in the same area. Absence seizures occur most often in
children. They usually begin with a brief change in the level of consciousness, signaled by
blinking or rolling of the eyes, a blank stare, and slight mouth movements. Symptoms of a
sensory seizure include hallucinations, flashing lights, tingling sensations, vertigo, dj vu, and
smelling a foul odor.
10. Myasthenic crisis and cholinergic crisis are the major complications of myasthenia
gravis. Which of the following is essential nursing knowledge when caring for a client in
crisis?
A. Weakness and paralysis of the muscles for swallowing and breathing occur in
either crisis
B. Cholinergic drugs should be administered to prevent further complications
associated with the crisis
C. The clinical condition of the client usually improves after several days of
treatment
D. Loss of body function creates high levels of anxiety and fear
CORRECT ANSWER: A.
The client cannot handle his own secretions, and respiratory arrest may be
imminent. Atropine may be administered to prevent crisis. Anticholinergic drugs are
administered to increase the levels of acetylcholine at the myoneural junction.
Cholinergic drugs mimic the actions of the parasympathetic nervous system and
would not be used.

11. The nurse is developing a teaching plan for the client with glaucoma. Which of the
following instructions would the nurse include in the plan of care?
A. Decrease fluid intake to control the intraocular pressure
B. Avoid overuse of the eyes

C. Decrease the amount of salt in the diet


D. Eye medications will need to be administered lifelong
CORRECT ANSWER: D.
The administration of eye drops is a critical component of the treatment plan for the client with
glaucoma. The client needs to be instructed that medications will need to be taken for the rest of
his or her life.
12. The nurse is caring for a client with a diagnosis of detached retina. Which assessment
sign would indicate that bleeding has occurred as a result of the retinal detachment?
A. Complaints of a burst of black spots or floaters
B. A sudden sharp pain in the eye
C. Total loss of vision
D. A reddened conjunctiva
CORRECT ANSWER: A.
Complaints of a sudden burst of black spots or floaters indicate that bleeding has occurred as a
result of the detachment.
13. The client with Alzheimers disease is being assisted with activities of daily living when
the nurse notes that the client uses her toothbrush to brush her hair. The nurse is aware
that the client is exhibiting:
A. Agnosia
B. Apraxia
C. Anomia
D. Aphasia
CORRECT ANSWER: B.
Apraxia is the inability to use objects appropriately. Agnosia is loss of sensory comprehension,
anomia is the inability to find words, and aphasia is the inability to speak or understand, so
answers A, C, and D are incorrect
14. A client admitted with a cerebral contusion is confused, disoriented, and restless. Which
nursing diagnosis takes highest priority?
A. Disturbed sensory perception (visual) related to neurologic trauma
B. Feeding self-care deficit related to neurologic trauma
C. Impaired verbal communication related to confusion
D. Risk for injury related to neurologic deficit
CORRECT ANSWER: D.
Because a cerebral contusion causes altered cognition, the nurse should identify Risk for injury
as the primary nursing diagnosis and focus on interventions that promote client safety and

prevent further injury. The other options are pertinent but don't take precedence over client
safety.
15. Family members would like to bring in birthday cake for a client with nerve damage.
What cranial nerve needs to be functioning so the client can chew?
A. Cranial nerve II
B. Cranial nerve V
C. Cranial nerve IX
D. Cranial nerve X
CORRECT ANSWER:
B. Chewing is a function of cranial nerve V. Swallowing is a motor function of cranial nerves IX
and X. Cranial nerve II doesn't possess a motor function.
16. The client with suspected Meningitis is admitted to the unit. The doctor is performing an
assessment to determine meningeal irritation and spinal nerve root inflammation. A
positive Kernigs sign is charted if the nurse notes:
A. Pain on flexion of the hip and knee
B. Nuchal rigidity on flexion of the neck
C. Pain when the head is turned to the left sign
D. Dizziness when changing positions
CORRECT ANSWER: A.
Kernigs sign is positive if pain occurs on flexion of the hip and knee. The Brudzinski reflex is
positive if pain occurs on flexion of the head and neck onto the chest so answer B is incorrect.
Answers C and D might be present but are not related to Kernigs sign.
17. Physiologically, the middle ear, containing the three ossicles, serves primarily to:
A. Maintain balance
B. Translate sound waves into nerve impulses
C. Amplify the energy of sound waves entering the ear
D. Communicate with the throat via the Eustachian tube
CORRECT ANSWER: C
The middle ear contains the three ossiclesmalleus, incus, and stapeswhich, along with the
tympanic membrane and oval window, form an amplifying system.
18. A client admitted to the hospital with a subarachnoid hemorrhage has complaints of
severe headache, nuchal rigidity, and projectile vomiting. The nurse knows lumbar
puncture (LP) would be contraindicated in this client in which of the following
circumstances?

A. Vomiting continues
B. Intracranial pressure (ICP) is increased
C. The client needs mechanical ventilation
D. Blood is anticipated in the cerebrospinal fluid (CSF)
CORRECT ANSWER: B.
Sudden removal of CSF results in pressures lower in the lumbar area than the brain and favors
herniation of the brain; therefore, LP is contraindicated with increased ICP. Vomiting may be
caused by reasons other than increased ICP; therefore, LP isnt strictly contraindicated. An LP
may be performed on clients needing mechanical ventilation. Blood in the CSF is diagnostic for
subarachnoid hemorrhage and was obtained before signs and symptoms of ICP.

19. A client with subdural hematoma was given mannitol to decrease intracranial pressure
(ICP). Which of the following results would best show the mannitol was effective?
A. Urine output increases
B. Pupils are 8 mm and nonreactive
C. Systolic blood pressure remains at 150 mm Hg
D. BUN and creatinine levels return to normal
CORRECT ANSWER: A.
Mannitol promotes osmotic diuresis by increasing the pressure gradient in the renal tubes. Fixed
and dilated pupils are symptoms of increased ICP or cranial nerve damage. No information is
given about abnormal BUN and creatinine levels or that mannitol is being given for renal
dysfunction or blood pressure maintenance.

20. A nurse would question an order to irrigate the ear canal in which of the following
circumstances?
A. Ear pain
B. Hearing loss
C. Otitis externa

D. Perforated tympanic membrane


CORRECT ANSWER: D
Irrigation of the ear canal is contraindicated with perforation of the tympanic membrane
because the solution entering the inner ear may cause dizziness, nausea, vomiting, and infection.
21. The nurse is caring for a client following enucleation. The nurse notes the presence of
bright red blood drainage on the dressing. Which nursing action is appropriate?
A. Notify the physician
B. Continue to monitor the drainage
C. Document the finding
D. Mark the drainage on the dressing and monitor for any increase in bleeding
CORRECT ANSWER: A.
If the nurse notes the presence of bright red drainage on the dressing, it must be reported to the
physician because this indicated hemorrhage.

22. The client is having a lumbar puncture performed. The nurse would plan to place the
client in which position for the procedure?
A. Side-lying, with legs pulled up and head bent down onto the chest
B. Side-lying, with a pillow under the hip
C. Prone, in a slight Trendelenburgs position
D. Prone, with a pillow under the abdomen.
CORRECT ANSWER: A.
The client undergoing lumbar puncture is positioned lying on the side, with the legs pulled up to
the abdomen, and with the head bent down onto the chest. This position helps to open the spaces
between the vertebrae.

23. The nurse is performing an assessment in a client with a suspected diagnosis of cataract.
The chief clinical manifestation that the nurse would expect to note in the early stages of
cataract formation is:

A. Eye pain
B. Floating spots
C. Blurred vision
D. Diplopia
CORRECT ANSWER: C.
A gradual, painless blurring of central vision is the chief clinical manifestation of a cataract.
Early symptoms include slightly blurred vision and a decrease in color perception.

24. The nurse is caring for the client with increased intracranial pressure. The nurse would
note which of the following trends in vital signs if the ICP is rising?
A. Increasing temperature, increasing pulse, increasing respirations, decreasing blood
pressure.
B. Increasing temperature, decreasing pulse, decreasing respirations, increasing
blood pressure.
C. Decreasing temperature, decreasing pulse, increasing respirations, decreasing
blood pressure.
D. Decreasing temperature, increasing pulse, decreasing respirations, increasing
blood pressure.
CORRECT ANSWER: B.
A change in vital signs may be a late sign of increased intracranial pressure. Trends include
increasing temperature and blood pressure and decreasing pulse and respirations. Respiratory
irregularities also may arise.

25. The client with a head injury has been urinating copious amounts of dilute urine through
the Foley catheter. The clients urine output for the previous shift was 3000 ml. The nurse
expects the physician to order:
A. Desmopressin (DDAVP, stimate)
B. Dexamethasone (Decadron)

C. Ethacrynic acid (Edecrin)


D. Mannitol (Osmitrol)
CORRECT ANSWER: A.
A complication of a head injury is diabetes insipidus, which can occur with insult to the
hypothalamus, the antidiuretic storage vesicles, or the posterior pituitary gland. Urine output
that exceeds 9 L per day generally requires treatment with desmopressin. Dexamethasone, a
glucocorticoid, is administered to treat cerebral edema. This medication may be ordered for the
head injured patient. Ethacrynic acid and mannitol are diuretics, which would be
contraindicated.

RESPIRATORY
&
HEMATOLOGY

1. A 43-year-old African American male is admitted with sickle cell anemia. The nurse
plans to assess circulation in the lower extremities every two hours. Which of the
following outcome criteria would the nurse use?
A.
B.
C.
D.

Body temperature of 99F or less


Toes moved in active range of motion
Sensation reported when soles of feet are touched
Capillary refill of < 3 seconds

CORRECT ANSWER: D.
It is important to assess the extremities for blood vessel occlusion in the client with sickle cell
anemia because a change in capillary refill would indicate a change in circulation. Body
temperature, motion, and sensation would not give information regarding peripheral circulation.
2. The primary physiological alteration in the development of asthma is:
A. Bronchiolar inflammation and dyspnea
B. Hypersecretion of abnormally viscous mucus
C. Infectious processes causing mucosal edema
D. Spasm of bronchiolar smooth muscle
CORRECT ANSWER:D.
Asthma is the presence of bronchiolar spasms. This spasm can be brought on by allergies or
anxiety. Answer A is incorrect because the primary physiological alteration is not inflammation.
Answer B is incorrect because there is the production of abnormally viscous mucus, not a
primary alteration. Answer C is incorrect because infection is not primary to asthma.
3. A 30-year-old male from Haiti is brought to the emergency department in sickle cell
crisis. What is the best position for this client?
A. Side-lying with knees flexed
B. Knee-chest
C. High Fowlers with knees flexed
D. Semi-Fowlers with legs extended on the bed
CORRECT ANSWER: D.
Placing the client in semi-Fowlers position provides the best oxygenation for this client. Flexion
of the hips and knees, which includes the knee-chest position, impedes circulation and is not
correct positioning for this client. Therefore, answers A, B, and C are incorrect.
4. A 25-year-old male is admitted in sickle cell crisis. Which of the following interventions
would be of highest priority for this client?
A. Taking hourly blood pressures with mechanical cuff
B. Encouraging fluid intake of at least 200mL per hour
C. Position in high Fowlers with knee gatch raised
D. Administering Tylenol as ordered
CORRECT ANSWER: B.

It is important to keep the client in sickle cell crisis hydrated to prevent further sickling of the
blood. Answer A is incorrect because a mechanical cuff places too much pressure on the arm.
Answer C is incorrect because raising the knee gatch impedes circulation. Answer D is incorrect
because Tylenol is too mild an analgesic for the client in crisis.
5. The nurse is conducting a physical assessment on a client with anemia. Which of the
following clinical manifestations would be most indicative of the anemia?
A. BP 146/88
B. Respirations 28 shallow
C. Weight gain of 10 pounds in six months
D. Pink complexion
CORRECT ANSWER: B.
When there are fewer red blood cells, there is less hemoglobin and less oxygen. Therefore, the
client is often short of breath, as indicated in answer B. The client with anemia is often pale in
color, has weight loss, and may be hypotensive. Answers A, C, and D are within normal and,
therefore, are incorrect.
6. The nurse is teaching the client with polycythemia vera about prevention of
complications of the disease. Which of the following statements by the client indicates a
need for further teaching?
A. I will drink 500mL of fluid or less each day.
B. I will wear support hose.
C. I will check my blood pressure regularly.
D. I will report ankle edema.
CORRECT ANSWER: A.
The client with polycythemia vera is at risk for thrombus formation. Hydrating the client with at
least 3L of fluid per day is important in preventing clot formation, so the statement to drink less
than 500mL is incorrect. Answers B, C, and D are incorrect because they all contribute to the
prevention of complications. Support hose promotes venous return, the electric razor prevents
bleeding due to injury, and a diet low in iron is essential to preventing further red cell formation.
7. An elderly client with pneumonia may appear with which of the following symptoms
first?
A. Altered mental status and dehydration
B. fever and chills
C. Hemoptysis and dyspnea
D. Pleuretic chest pain and cough
CORRECT ANSWER: A.
Fever, chills, hemoptysis, dyspnea, cough, and pleuretic chest pain are the common symptoms of
pneumonia, but elderly clients may first appear with only an altered mental status and
dehydration due to a blunted immune response.
8. Which of the following pathophysiological mechanisms that occurs in the lung

parenchyma allows pneumonia to develop?


A.
B.
C.
D.

Atelectasis
Bronchiectasis
Effusion
Inflammation

CORRECT ANSWER: D.
The most common feature of all types of pneumonia is an inflammatory pulmonary response to
the offending organism or agent. Atelectasis and brochiectasis indicate a collapse of a portion of
the airway that doesnt occur with pneumonia. An effusion is an accumulation of excess pleural
fluid in the pleural space, which may be a secondary response to pneumonia.
9. A client with hemophilia has a nosebleed. Which nursing action is most appropriate to
control the bleeding?
A. Place the client in a sitting position
B. Administer acetaminophen (Tylenol)
C. Pinch the soft lower part of the nose
D. Apply ice packs to the forehead
CORRECT ANSWER: C.
C is correct because direct pressure to the nose stops the bleeding. Answers A, B, and D are
incorrect because they do not stop bleeding.

10.The nurse understands that the client with pernicious anemia will have which
distinguishing laboratory findings?
A. Schillings test, elevated
B. Intrinsic factor, absent.
C. Sedimentation rate, 16 mm/hour
D. RBCs 5.0 million

CORRECT ANSWER: B.
The defining characteristic of pernicious anemia, a megaloblastic anemia, is lack of the intrinsic
factor, which results from atrophy of the stomach wall. Without the intrinsic factor, vitamin B12
cannot be absorbed in the small intestines, and folic acid needs vitamin B12 for DNA synthesis
of RBCs. The gastric analysis was done to determine the primary cause of the anemia. An

elevated excretion of the injected radioactive vitamin B12, which is protocol for the first and
second stage of the Schilling test, indicates that the client has the intrinsic factor and can absorb
vitamin B12 into the intestinal tract. A sedimentation rate of 16 mm/hour is normal for both men
and women and is a nonspecific test to detect the presence of inflammation. It is not specific to
anemias. An RBC value of 5.0 million is a normal value for both men and women and does not
indicate anemia.
11. Emergency treatment of a client in status asthmaticus includes which of the following
medications?
A.
B.
C.
D.

Inhaled beta-adrenergic agents


Inhaled corticosteroids
I.V. beta-adrenergic agents
Oral corticosteroids

CORRECT ANSWER: A.
Inhaled beta-adrenergic agents help promote bronchodilation, which improves oxygenation. I.V.
beta-adrenergic agents can be used but have to be monitored because of their greater systemic
effects. Theyre typically used when the inhaled beta-adrenergic agents dont work.
Corticosteriods are slow-acting, so their use wont reduce hypoxia in the acute phase.
12.The nurse devises a teaching plan for the patient with aplastic anemia. Which
of the following is the most important concept to teach for health
maintenance?

A.
B.
C.
D.

Eat animal protein and dark leafy vegetables each day


Avoid exposure to others with acute infection
Practice yoga and meditation to decrease stress and anxiety
Get 8 hours of sleep at night and take naps during the day

CORRECT ANSWER IS B.
Clients with aplastic anemia are severely immunocompromised and at risk for infection and
possible death related to bone marrow suppression and pancytopenia. Strict aseptic technique
and reverse isolation are important measures to prevent infection. Although diet, reduced stress,
and rest are valued in supporting health, the potentially fatal consequence of an acute infection
places it as a priority for teaching the client about health maintenance. Animal meat and dark
green leafy vegetables, good sources of vitamin B12 and folic acid, should be included in the
daily diet. Yoga and meditation are good complimentary therapies to reduce stress. Eight hours
of rest and naps are good for spacing and pacing activity and rest.
13.A client was admitted with iron deficiency anemia and blood-streaked emesis.
Which question is most appropriate for the nurse to ask in determining the

extent of the clients activity intolerance?

A.
B.
C.
D.

What activities were you able to do 6 months ago compared with the present?
How long have you had this problem?
Have you been able to keep up with all your usual activities?
Are you more tired now than you used to be?

CORRECT ANSWER: A.
It is difficult to determine activity intolerance without objectively comparing activities from one
time frame to another. Because iron deficiency anemia can occur gradually and individual
endurance varies, the nurse can best assess the clients activity tolerance by asking the client to
compare activities 6 months ago and at the present. Asking a client how long a problem has
existed is a very open-ended question that allows for too much subjectivity for any definition of
the clients activity tolerance. Also, the client may not even identify that a problem exists.
Asking the client whether he is staying abreast of usual activities addresses whether the tasks
were completed, not the tolerance of the client while the tasks were being completed or the
resulting condition of the client after the tasks were completed. Asking the client if he is more
tired now than usual does not address his activity tolerance. Tiredness is a subjective evaluation
and again can be distorted by factors such as the gradual onset of the anemia or the endurance
of the individual.
14. Which of the following assessment findings would help confirm a diagnosis of asthma in
a client suspected of having the disorder?
A.
B.
C.
D.

Circumoral cyanosis
Increased forced expiratory volume
Inspiratory and expiratory wheezing
Normal breath sounds

CORRECT ANSWER: C.
Inspiratory and expiratory wheezes are typical findings in asthma. Circumoral cyanosis may be
present in extreme cases of respiratory distress. The nurse would expect the client to have a
decreased forced expiratory volume because asthma is an obstructive pulmonary disease. Breath
sounds will be tight sounding or markedly decreased; they wont be normal.
15.A client with pernicious anemia asks why she must take vitamin B 12 injections
for the rest of her life. What is the nurses best response?

A. The reason for your vitamin deficiency is an inability to absorb the vitamin
because the stomach is not producing sufficient acid.
B. The reason for your vitamin deficiency is an inability to absorb the vitamin
because the stomach is not producing sufficient intrinsic factor.
C. The reason for your vitamin deficiency is an excessive excretion of the vitamin
because of kidney dysfunction.
D. The reason for your vitamin deficiency is an increased requirement for the

vitamin because of rapid red blood cell production.


CORRECT ANSWER: B.
Most clients with pernicious anemia have deficient production of intrinsic factor in the stomach.
Intrinsic factor attaches to the vitamin in the stomach and forms a complex that allows the
vitamin to be absorbed in the small intestine. The stomach is producing enough acid, there is not
an excessive excretion of the vitamin, and there is not a rapid production of RBCs in this
condition.
16. A client is to receive epoetin (Epogen) injections. What laboratory value
should the nurse assess before giving the injection?

A.
B.
C.
D.

Hematocrit
Partial thromboplastin time
Hemoglobin concentration
Prothrombin time

CORRECT ANSWER: A.
Epogen is a recombinant DNA form of erythropoietin, which stimulates the production of RBCs
and therefore causes the hematocrit to rise. The elevation in hematocrit causes an elevation in
blood pressure; therefore, the blood pressure is a vital sign that should be checked. The PTT,
hemoglobin level, and PT are not monitored for this drug.
17. A 58-year-old client with a 40-year history of smoking one to two packs of cigarettes a
day has a chronic cough producing thick sputum, peripheral edema, and cyanotic nail
beds. Based on this information, he most likely has which of the following conditions?
A.
B.
C.
D.

Adult respiratory distress syndrome (ARDS)


Asthma
Chronic obstructive bronchitis
Emphysema

CORRECT ANSWER: C.
Because of his extensive smoking history and symptoms, the client most likely has chronic
obstructive bronchitis. Clients with ARDS have acute symptoms of and typically need large
amounts of oxygen. Clients with asthma and emphysema tend not to have a chronic cough or
peripheral edema.
18. A 66-year-old client has marked dyspnea at rest, is thin, and uses accessory muscles to
breathe. Hes tachypneic, with a prolonged expiratory phase. He has no cough. He leans
forward with his arms braced on his knees to support his chest and shoulders for
breathing. This client has symptoms of which of the following respiratory disorders?
A. ARDS
B. Asthma

C. Chronic obstructive bronchitis


D. Emphysema
CORRECT ANSWER: D.
These are classic signs and symptoms of a client with emphysema. Clients with ARDS are acutely
short of breath and require emergency care; those with asthma are also acutely short of breath
during an attack and appear very frightened. Clients with chronic obstructive bronchitis are
bloated and cyanotic in appearance.
19.Clients with chronic obstructive bronchitis are given diuretic therapy. Which of
the following reasons best explains why?

A.
B.
C.
D.

Reducing fluid volume reduces oxygen demand.


Reducing fluid volume improves clients mobility.
Restricting fluid volume reduces sputum production.
Reducing fluid volume improves respiratory function.

CORRECT ANSWER: A.
Reducing fluid volume reduces the workload of the heart, which reduces oxygen demand and, in
turn, reduces the respiratory rate. It may also reduce edema and improve mobility a little, but
exercise tolerance will still be harder to clear airways. Reducing fluid volume wont improve
respiratory function, but may improve oxygenation.
20.A client with macrocytic anemia has a burn on her foot and states that she
had been watching television while lying on a heating pad. What is the
nurses first response?

A.
B.
C.
D.

Assess for potential abuse


Check for diminished sensations
Document the findings
Clean and dress the area

CORRECT ANSWER: B.
Macrocytic anemias can result from deficiencies in vitamin B12 or ascorbic acid. Only vitamin
B12 deficiency causes diminished sensations of peripheral nerve endings. The nurse should assess
for peripheral neuropathy and instruct the client in self-care activities for her diminished
sensation to heat and pain. The burn could be related to abuse, but this conclusion would require
more supporting data. The findings should be documented, but the nurse would want to address
the clients sensations first. The decision of how to treat the burn should be determined by the
physician.
21. Teaching for a client with chronic obstructive pulmonary disease (COPD) should include
which of the following topics?

A. How to have his wife learn to listen to his lungs with a stethoscope from WalMart.
B. How to increase his oxygen therapy.
C. How to treat respiratory infections without going to the physician.
D. How to recognize the signs of an impending respiratory infection.
CORRECT ANSWER: D.
Respiratory infection in clients with a respiratory disorder can be fatal. Its important that the
client understands how to recognize the signs and symptoms of an impending respiratory
infection. It isnt appropriate for the wife to listen to his lung sounds, besides, you cant purchase
stethoscopes from Wal-Mart. If the client has signs and symptoms of an infection, he should
contact his physician at once.
22. Which of the following measures can reduce or prevent the incidence of atelectasis in a
post-operative client?
A.
B.
C.
D.

Chest physiotherapy
Mechanical ventilation
Reducing oxygen requirements
Use of an incentive spirometer

CORRECT ANSWER: D.
Using an incentive spirometer requires the client to take deep breaths and promotes lung
expansion. Chest physiotherapy helps mobilize secretions but wont prevent atelectasis.
Reducing oxygen requirements or placing someone on mechanical ventilation doesnt affect the
development of atelectasis.
23. A nurse reviews the arterial blood gas results of a patient and notes the following: pH
7.45; PCO2 30 mm Hg; and bicarbonate concentration of 22 mEq/L. The nurse analyses
these results as indicating:
A. Metabolic acidosis, compensated.
B. Metabolic alkalosis, uncompensated.
C. Respiratory alkalosis, compensated.
D. Respiratory acidosis, compensated.
CORRECT ANSWER: C.
The normal pH is 7.35 to 7.45. In a respiratory condition, an opposite (see-saw) will be seen
between the pH and the PCO2. In this situation, the pH is at the high end of the normal value and
the PCO2 is low. In an alkalotic condition, the pH is up. Therefore, the values identified in the
question indicate a respiratory alkalosis. Compensation occurs when the pH returns to a normal
value. Because the pH is in the normal range at the high end, compensation has occurred.
24. A nurse is caring for a client with a nasogastric tube that is attached to low suction. The
nurse monitors the client, knowing that the client is at risk for which acid-base disorder?
A. Respiratory acidosis
B. Respiratory alkalosis

C. Metabolic acidosis
D. Metabolic alkalosis
CORRECT ANSWER: D.
Loss of gastric fluid via nasogastric suction or vomiting causes metabolic alkalosis as a result of
the loss of hydrochloric acid.
25. Theophylline (Theo-Dur) tablets are prescribed for a client with chronic airflow
limitation, and the nurse instructs the client about the medication. Which statement by the
client indicates a need for further teaching?
A. I will take the medication on an empty stomach.
B. I will take the medication with food.
C. I will continue to take the medication even if I am feeling better.
D. Periodic blood levels will need to be obtained.
CORRECT ANSWER: A.
Theo-Dur is a bronchodilator. The medication should be administered with food such as milk
and crackers to prevent GI irritation.

CARDIOVASCULAR
SYSTEM

1. When a client has suffered severe burns all over his body, the most effective method of
monitoring the cardiovascular system is:
A. Cuff blood pressure
B. Arterial Pressure
C. Pulmonary Artery Pressure
D. Central Venous Pressure
CORRECT ANSWER: C.
Pulmonary Artery Pressure is the most effective method. Clients with a large percentage of
burned body surface often does not have an area where the cuff can be applied. Pulse
monitoring is not accurate enough to detect subtle changes in the system. Central Venous
Pressures are less than optimal because changes in the left heart pressure (sign of pulmonary
edema) are often not reflected in the right heart pressures.
2. A client is admitted complaining of chest pain. Which of the following drug orders should
the nurse question?
A. Nitroglycerin
B. Ampicillin
C. Propranolol
D. Verapamil
CORRECT ANSWER: B.
Clients with chest pain can be treated with nitroglycerin, a beta blocker such as propranolol, or
Verapamil. There is no indication for an antibiotic such as Ampicillin, so answers A, C, and D
are incorrect.
3. Following a heart transplant, a client is started on medication to prevent organ rejection.
Which category of medication prevents the formation of antibodies against the new
organ?
A. Antivirals
B. Antibiotics
C. Immunosuppressants
D. Analgesics
CORRECT ANSWER:C.
Immunosuppressants are used to prevent antibody formation. Antivirals, antibiotics, and
analgesics are not used to prevent antibody production, so answers A, B, and D are incorrect.
4. The nurse on the telemetry unit is faced with various situations. Which situation takes
priority?
A. A client's cardiac monitor suddenly reveals sinus tachycardia with isolated
premature ventricular contractions.
B. A client's cardiac rhythm suddenly changes from normal sinus rhythm to
uncontrolled atrial fibrillation.
C. A client is requesting help to go to the bathroom.
D. The cardiologist is asking the nurse to make rounds with him to his clients.
CORRECT ANSWER: B.

The client whose cardiac rhythm suddenly changes from normal sinus rhythm to uncontrolled
atrial fibrillation takes priority. This cardiac rhythm change may cause clots to shower from the
atria placing the client at risk for a stroke. The client whose cardiac monitor reveals sinus
tachycardia with isolated premature ventricular contractions isn't experiencing a lifethreatening situation; therefore, he doesn't take priority. The nurse can ask her ancillary staff
member to assist the client to the bathroom. Making rounds with the physician can wait until the
nurse addresses the needs of the client in atrial fibrillation.
5. Which of the following symptoms is the most likely origin of pain the client described as
knifelike chest pain that increases in intensity with inspiration?
A. Cardiac
B. Gastrointestinal
C. Musculoskeletal
D. Pulmonary
CORRECT ANSWER: D.
Pulmonary pain is generally described by these symptoms. Musculoskeletal pain only increases
with movement. Cardiac and GI pains dont change with respiration.

6. Which of the following blood tests is most indicative of cardiac damage?


A. Lactate dehydrogenase
B. Complete blood count (CBC)
C. Troponin I
D. Creatine kinase (CK)
CORRECT ANSWER: C.
Troponin I levels rise rapidly and are detectable within 1 hour of myocardial injury. Troponin I
levels arent detectable in people without cardiac injury. Lactate dehydrogenase (LDH) is
present in almost all body tissues and not specific to heart muscle. LDH isoenzymes are useful in
diagnosing cardiac injury. CBC is obtained to review blood counts, and a complete chemistry is
obtained to review electrolytes. Because CK levels may rise with skeletal muscle injury, CK
isoenzymes are required to detect cardiac injury.

7. After an anterior wall myocardial infarction, which of the following problems is indicated
by auscultation of crackles in the lungs?
A. Left-sided heart failure
B. Pulmonic valve malfunction
C. Right-sided heart failure
D. Tricupsid valve malfunction
CORRECT ANSWER: A.
The left ventricle is responsible for most of the cardiac output. An anterior wall MI may result in
a decrease in left ventricular function. When the left ventricle doesnt function properly, resulting
in left-sided heart failure, fluid accumulates in the interstitial and alveolar spaces in the lungs
and causes crackles. Pulmonic and tricuspid valve malfunction causes right sided heart failure.

8. What is the first intervention for a client experiencing MI?


A. Administer morphine
B. Administer oxygen
C. Administer sublingual nitroglycerin
D. Obtain an ECG
CORRECT ASNWER: B
Administering supplemental oxygen to the client is the first priority of care. The myocardium is
deprived of oxygen during an infarction, so additional oxygen is administered to assist in
oxygenation and prevent further damage. Morphine and nitro are also used to treat MI, but
theyre more commonly administered after the oxygen. An ECG is the most common diagnostic
tool used to evaluate MI.

9. In which of the following disorders would the nurse expect to assess sacral edema in a
bedridden client?
A. Diabetes
B. Pulmonary emboli

C. Renal failure
D. Right-sided heart failure
CORRECT ANSWER:D. The most accurate area on the body to assess dependent edema in a
bed-ridden client is the sacral area. Sacral, or dependent, edema is secondary to right-sided
heart failure.
10. Which of the following classes of medications maximizes cardiac performance in clients
with heart failure by increasing ventricular contractibility?
A. Beta-adrenergic blockers
B. Calcium channel blockers
C. Diuretics
D. Inotropic agents
CORRECT ANSWER: D.
Inotropic agents are administered to increase the force of the hearts contractions, thereby
increasing ventricular contractility and ultimately increasing cardiac output.
11. Which of the following conditions is most closely associated with weight gain, nausea,
and a decrease in urine output?
A. Angina pectoris
B. Cardiomyopathy
C. Left-sided heart failure
D. Right-sided heart failure
CORRECT ANSWER: D.
Weight gain, nausea, and a decrease in urine output are secondary effects of right-sided heart
failure. Cardiomyopathy is usually identified as a symptom of left-sided heart failure. Left-sided
heart failure causes primarily pulmonary symptoms rather than systemic ones. Angina pectoris
doesnt cause weight gain, nausea, or a decrease in urine output.

12. If medical treatments fail, which of the following invasive procedures is necessary for
treating cardiomyopathy?

A. Cardiac catherization
B. Coronary artery bypass graft (CABG)
C. Heart transplantation
D. Intra-aortic balloon pump (IABP)
CORRECT ANSWER: C.
The only definitive treatment for cardiomyopathy that cant be controlled medically is a heart
transplant because the damage to the heart muscle is irreversible.
13. Which of the following is a compensatory response to decreased cardiac output?
A. Decreased BP
B. Alteration in LOC
C. Decreased BP and diuresis
D. Increased BP and fluid retention
CORRECT ANSWER: D.
The body compensates for a decrease in cardiac output with a rise in BP, due to the stimulation
of the sympathetic NS and an increase in blood volume as the kidneys retain sodium and water.
Blood pressure doesnt initially drop in response to the compensatory mechanism of the body.
Alteration in LOC will occur only if the decreased cardiac output persists.

14. A client who had cardiac surgery 24 hours ago has a urine output averaging 19 ml/hr for 2
hours. The client received a single bolus of 500 ml of IV fluid. Urine output for the
subsequent hour was 25 ml. Daily laboratory results indicate the blood urea nitrogen is 45
mg/dL and the serum creatinine is 2.2 mg/dL. A nurse interprets the client is at risk for:
A. Hypovolemia
B. UTI
C. Glomerulonephritis
D. Acute renal failure
CORRECT ANSWER: D.

The client who undergoes cardiac surgery is at risk for renal injury from poor perfusion,
hemolysis, low cardiac output, or vasopressor medication therapy. Renal insult is signaled by
decreased urine output, and increased BUN and creatinine levels. The client may need
medications such as dopamine (Intropin) to increase renal perfusion and possibly could need
peritoneal dialysis or hemodialysis.

15. A client has frequent bursts of ventricular tachycardia on the cardiac monitor. A nurse is
most concerned with this dysrhythmia because:
A. It is uncomfortable for the client, giving a sense of impending doom.
B. It produces a high cardiac output that quickly leads to cerebral and myocardial
ischemia.
C. It is almost impossible to convert to a normal sinus rhythm.
D. It can develop into ventricular fibrillation at any time.
CORRECT ANSWER: D.
Ventricular tachycardia is a life-threatening dysrhythmia that results from an irritable ectopic
focus that takes over as the pacemaker for the heart. The low cardiac output that results can lead
quickly to cerebral and myocardial ischemia. Clients frequently experience a feeling of
impending death. Ventricular tachycardia is treated with antidysrhythmic medications or
magnesium sulfate, cardioversion (client awake), or defibrillation (loss of consciousness),
Ventricular tachycardia can deteriorate into ventricular defibrillation at any time.

16. Which of the following reflects the principle on which a clients diet will most likely be
based during the acute phase of MI?
A. Liquids as ordered
B. Small, easily digested meals
C. Three regular meals per day
D. NPO
CORRECT ANSWER: B.
Recommended dietary principles in the acute phase of MI include avoiding large meals because
small, easily digested foods are better digested foods are better tolerated. Fluids are given
according to the clients needs, and sodium restrictions may be prescribed, especially for clients

with manifestations of heart failure. Cholesterol restrictions may be ordered as well. Clients are
not prescribed a diet of liquids only or NPO unless their condition is very unstable.

17. The most important assessment for the nurse to make after a client has had a
femoropopiteal bypass for peripheral vascular disease would be:
A. Incisional pain
B. Pedal pulse rate
C. Capillary refill time
D. Degree of hair growth
CORRECT ANSWER:C.
Checking capillary refill provides data about current perfusion of the extremity. While the
presence and quality of the pedal pulse provide data about peripheral circulation, it is not
necessary to count the rate.
18. The physician refers the client with unstable angina for a cardiac catheterization. The
nurse explains to the client that this procedure is being used in this specific case to:
A. Open and dilate the blocked coronary arteries
B. Assess the extent of arterial blockage
C. Bypass obstructed vessels
D. Assess the functional adequacy of the valves and heart muscle.
CORRECT ANSWER: B. Cardiac catheterization is done in clients with angina primarily to
assess the extent and severity of the coronary artery blockage, A decision about medical
management, angioplasty, or coronary artery bypass surgery will be based on the
catheterization results.
19. A 57-year-old client with a history of asthma is prescribed propanolol (Inderal) to control
hypertension. Before administered propranolol, which of the following actions should the
nurse take first?
A. Monitor the apical pulse rate
B. Instruct the client to take medication with food
C. Question the physician about the order
D. Caution the client to rise slowly when standing.

CORRECT ANSWER: C.
Propranolol and other beta-adrenergic blockers are contraindicated in a client with asthma, so
the nurse should question the physician before giving the dose. The other responses are
appropriate actions for a client receiving propranolol, but questioning the physician takes
priority. The clients apical pulse should always be checked before giving propranolol; if the
pulse rate is extremely low, the nurse should withhold the drug and notify the physician.
20. Following a treadmill test and cardiac catheterization, the client is found to have coronary
artery disease, which is inoperative. He is referred to the cardiac rehabilitation unit.
During his first visit to the unit he says that he doesnt understand why he needs to be
there because there is nothing that can be done to make him better. The best nursing
response is:
A. Cardiac rehabilitation is not a cure but can help restore you to many of your
former activities.
B. Here we teach you to gradually change your lifestyle to accommodate your heart
disease.
C. You are probably right but we can gradually increase your activities so that you
can live a more active life.
D. Do you feel that you will have to make some changes in your life now?
CORRECT ANSWER: A.
Such a response does not have false hope to the client but is positive and realistic. The answer
tells the client what cardiac rehabilitation is and does not dwell upon his negativity about it.

21. Which of the following signs and symptoms would most likely be found in a client with
mitral regurgitation?
A. Exertional dyspnea
B. Confusion
C. Elevated creatine phosphokinase concentration
D. Chest pain
CORRECT ANSWER: A.
Weight gain, due to fluid retention and worsening heart failure, causes exertional dyspnea in
clients with mitral regurgitation. The rise in left atrial pressure that accompanies mitral valve
disease is transmitted backward into pulmonary veins, capillaries, and arterioles and eventually
to the right ventricle. Signs and symptoms of pulmonary and systemic venous congestion follow.
22. The nurse expects that a client with mitral stenosis would demonstrate symptoms
associated with congestion in the:

A.
B.
C.
D.

Aorta
Right atrium
Superior vena cava
Pulmonary circulation

CORRECT ANSWER: D.
When mitral stenosis is present, the left atrium has difficulty emptying its contents into the left
ventricle. Hence, because there is no valve to prevent backward flow into the pulmonary vein,
the pulmonary circulation is under pressure.
23. The adaptations of a client with complete heart block would most likely include:
A. Nausea and vertigo
B. Flushing and slurred speech
C. Cephalalgia and blurred vision
D. Syncope and low ventricular rate
CORRECT ANSWER: D.
In complete atrioventricular block, the ventricles take over the pacemaker function in the heart
but at a much slower rate than that of the SA node. As a result there is decreased cerebral
circulation, causing syncope.

24. When auscultating the apical pulse of a client who has atrial fibrillation, the nurse would
expect to hear a rhythm that is characterized by:
A. The presence of occasional coupled beats
B. Long pauses in an otherwise regular rhythm
C. A continuous and totally unpredictable irregularity
D. Slow but strong and regular beats
CORRECT ANSWER: C.
In atrial fibrillation, multiple ectopic foci stimulate the atria to contract. The AV node is unable
to transmit all of these impulses to the ventricles, resulting in a pattern of highly irregular
ventricular contractions.

25. A client has been admitted to the hospital with a diagnosis of suspected bacterial
endocarditis. The complication the nurse will constantly observe for is:
A. Presence of heart murmur
B. Systemic emboli
C. Fever
D. Congestive heart failure
CORRECT ANSWER: B.
Emboli are the major problem; those arising in the right heart chambers will terminate in the
lungs and left chamber emboli may travel anywhere in the arteries. Heart murmurs, fever, and
night sweats may be present, but do not indicate a problem with emboli. CHF may be a result,
but this is not as dangerous an outcome as emboli.

GASTROINTESTINAL
SYSTEM

1. The physician has prescribed Nexium (esomeprazole) for a client with erosive gastritis.
The nurse should administer the medication:
A. 30 minutes before a meal
B. With each meal
C. In a single dose at bedtime
D. 30 minutes after meals
CORRECT ANSWER: A.
Proton pump inhibitors should be taken prior to the meal. Answers B, C, and D are incorrect
times for giving proton pump inhibitors like Nexium.
2. Which statement made by the family member caring for the client with a percutaneous
gastrostomy tube indicates understanding of the nurses teaching?
A. I must flush the tube with water after feedings and clamp the tube.
B. I must check placement four times per day.
C. I will report to the doctor any signs of indigestion.
D. If my father is unable to swallow, I will discontinue the feeding and call the
clinic.
CORRECT ANSWER: A.
The clients family member should be taught to flush the tube after each feeding and clamp the
tube. The placement should be checked before feedings, and indigestion can occur with the PEG
tube, just as it can occur with any client, so answers B and C are incorrect. Medications can be
ordered for indigestion, but it is not a reason for alarm. A percutaneous endoscopy gastrostomy
tube is used for clients who have experienced difficulty swallowing. The tube is directly inserted
into the stomach and does not require swallowing; therefore answer D is incorrect.
3. A client with acute pancreatitis is experiencing severe abdominal pain. Which of the
following orders should be questioned by the nurse?
A. Meperidine 100mg IM m 4 hours PRN pain
B. Mylanta 30 ccs m 4 hours via NG
C. Cimetadine 300mg PO m.i.d.
D. Morphine 8mg IM m 4 hours PRN pain
CORRECT ANSWER: D.
Morphine is contraindicated in clients with gallbladder disease and pancreatitis because
morphine causes spasms of the Sphincter of Oddi. Meperidine, Mylanta, and Cimetadine are
ordered for pancreatitis, making answers A, B, and C incorrect.
4. A client with an abdominal cholecystectomy returns from surgery with a Jackson-Pratt
drain. The chief purpose of the Jackson-Pratt drain is to:
A. Prevent the need for dressing changes
B. Reduce edema at the incision
C. Provide for wound drainage
D. Keep the common bile duct open

CORRECT ANSWER: C.
A Jackson-Pratt drain is a serum-collection device commonly used in abdominal surgery. A JP
drain will not prevent the need for dressing changes, reduce edema or keep the common bile duct
open, so A, B, and D are incorrect. A t-tube is used to keep the common bile duct open.
5. A client who underwent abdominal surgery who has a nasogastric (NG) tube in place
begins to complain of abdominal pain that he describes as "feeling full and
uncomfortable." Which assessment should the nurse perform first?
A. Measure abdominal girth
B. Auscultate bowel sounds
C. Assess patency of the NG tube
D. Assess vital signs
CORRECT ANSWER: C.
When an NG tube is no longer patent, stomach contents collect in the stomach giving the client a
sensation of fullness. The nurse should begin by assessing patency of the NG tube. The nurse can
measure abdominal girth, auscultate bowels, and assess vital signs, but she should check NG
tube patency first to help relieve the client's discomfort.
6. A client, age 82, is admitted to an acute care facility for treatment of an acute flare-up of
a chronic GI condition. In addition to assessing the client for complications of the current
illness, the nurse monitors for age-related changes in the GI tract. Which age-related
change increases the risk of anemia?
A. Atrophy of the gastric mucosa
B. Decrease in intestinal flora
C. Increase in bile secretion
D. Dulling of nerve impulses
CORRECT ANSWER: A.
Atrophy of the gastric mucosa reduces hydrochloric acid secretion; this, in turn, impairs
absorption of iron and vitamin B12, increasing the risk of anemia as a person ages. A decrease
in hydrochloric acid increases, not decreases, intestinal flora; as a result, the client is at
increased risk for infection, not anemia. A reduction, not increase, in bile secretion may lead to
malabsorption of fats and fat-soluble vitamins. Dulling of nerve impulses associated with aging
increases the risk of constipation, not anemia.
7. Which of the following symptoms would a client in the early stages of peritonitis exhibit?
A. Abdominal distention
B. Abdominal pain and rigidity
C. Hyperactive bowel sounds
D. Right upper quadrant pain
CORRECT ANSWER: B.
Abdominal pain causing rigidity of the abdominal muscles is characteristic of peritonitis.
Abdominal distention may occur as a late sign but not early on. Bowel sounds may be normal or
decreased but not increased. Right upper quadrant pain is characteristic of cholecystitis or

hepatitis.
8. Which of the following aspects is the priority focus of nursing management for a client
with peritonitis?
A. Fluid and electrolyte balance
B. Gastric irrigation
C. Pain management
D. Psychosocial issues
CORRECT ANSWER: A.
Peritonitis can advance to shock and circulatory failure, so fluid and electrolyte balance is the
priority focus of nursing management. Gastric irrigation may be needed periodically to ensure
patency of the nasogastric tube. Although pain management is important for comfort and
psychosocial care will address concerns such as anxiety, focusing on fluid and electrolyte
imbalance will maintain hemodynamic stability.
9. Which of the following substances is most likely to cause gastritis?
A. Milk
B. Bicarbonate of soda, or baking soda
C. Enteric coated aspirin
D. Nonsteriodal anti-inflammatory drugs
CORRECT ANSWER: D.
NSAIDS are a common cause of gastritis because they inhibit prostaglandin synthesis. Milk,
once thought to help gastritis, has little effect on the stomach mucosa. Bicarbonate of soda, or
baking soda, may be used to neutralize stomach acid, but it should be used cautiously because it
may lead to metabolic acidosis. ASA with enteric coating shouldnt contribute significantly to
gastritis because the coating limits the aspirins effect on the gastric mucosa.
10. Youre caring for Beth who underwent a Billroth II procedure (surgical removal of the
pylorus and duodenum) for treatment of a peptic ulcer. Which findings suggest that the
patient is developing dumping syndrome, a complication associated with this procedure?
A. Flushed, dry skin
B. Headache and bradycardia
C. Dizziness and sweating
D. Dyspnea and chest pain
CORRECT ANSWER: C.
After a Billroth II procedure, a large amount of hypertonic fluid enters the intestine. This causes
extracellular fluid to move rapidly into the bowel, reducing circulating blood volume and
producing vasomotor symptoms. Vasomotor symptoms produced by dumping syndrome include
dizziness and sweating, tachycardia, syncope, pallor, and palpitations.
11. Youre developing the plan of care for a patient experiencing dumping syndrome after a
Billroth II procedure. Which dietary instructions do you include?
A. Omit fluids with meals.

B. Increase carbohydrate intake.


C. Decrease protein intake.
D. Decrease fat intake.
CORRECT ANSWER: A.
Gastric emptying time can be delayed by omitting fluids from your patients meal. A diet low in
carbs and high in fat & protein is recommended to treat dumping syndrome.
12. Which of the following tests should be administered to a client suspected of having
diverticulosis?
A. Abdominal ultrasound
B. Barium enema
C. Barium swallow
D. Gastroscopy
CORRECT ANSWER: B.
A barium enema will cause diverticula to fill with barium and be easily seen on x-ray. An
abdominal US can tell more about structures, such as the gallbladder, liver, and spleen, than the
intestine. A barium swallow and gastroscopy view upper GI structures.
13. If a client had irritable bowel syndrome, which of the following diagnostic tests would
determine if the diagnosis is Crohns disease or ulcerative colitis?
A. Abdominal computed tomography (CT) scan
B. Abdominal x-ray
C. Barium swallow
D. Colonoscopy with biopsy
CORRECT ANSWER: D.
A colonoscopy with biopsy can be performed to determine the state of the colons mucosal
layers, presence of ulcerations, and level of cytologic development. An abdominal x-ray or CT
scan wouldnt provide the cytologic information necessary to diagnose which disease it is. A
barium swallow doesnt involve the intestine.
14. Surgical management of ulcerative colitis may be performed to treat which of the
following complications?
A. Gastritis
B. Bowel herniation
C. Bowel outpouching
D. Bowel perforation
CORRECT ANSWER: D.
Perforation, obstruction, hemorrhage, and toxic megacolon are common complications of
ulcerative colitis that may require surgery. Herniation and gastritis arent associated with
irritable bowel diseases, and outpouching of the bowel is diverticulosis.
15. After abdominal surgery, your patient has a severe coughing episode that causes wound

evisceration. In addition to calling the doctor, which intervention is most appropriate?


A. Irrigate the wound & organs with Betadine.
B. Cover the wound with a saline soaked sterile dressing.
C. Apply a dry sterile dressing & binder.
D. Push the organs back & cover with moist sterile dressings.
CORRECT ANSWER: B.
Cover the organs with a sterile, nonadherent dressing moistened with normal saline. Do this to
prevent infection and to keep the organs from drying out.
16. Youre caring for Sarry with liver cirrhosis. Which of the following assessment findings
leads you to suspect hepatic encephalopathy in her?
A. Asterixis
B. Chvosteks sign
C. Trousseaus sign
D. Hepatojugular reflex
CORRECT ANSWER: A.
Asterixis is an early neurologic sign of hepatic encephalopathy elicited by asking the patient to
hold her arms stretched out. Asterixis is present if the hands rapidly extend and flex.
17. An enema is prescribed for a client with suspected appendicitis. Which of the following
actions should the nurse take?
A. Prepare 750 ml of irrigating solution warmed to 100oF
B. Question the physician about the order
C. Provide privacy and explain the procedure to the client
D. Assist the client to left lateral Sims position
CORRECT ANSWER: B.
Enemas are contraindicated in an acute abdominal condition of unknown origin as well as after
recent colon or rectal surgery or myocardial infarction. The other answers are correct only when
enema administration is appropriate.
18. The nurse is monitoring a client for the early signs of dumping syndrome. Which
symptom indicates this occurrence?
A. Abdominal cramping and pain
B. Bradycardia and indigestion

C. Sweating and pallor


D. Double vision and chest pain
CORRECT ANSWER: C.
Early manifestations of dumping syndrome occur 5 to 30 minutes after eating. Symptoms include
vertigo, tachycardia, syncope, sweating, pallor, palpitations, and the desire to lie down.
19. Britney, a 20 y.o. student is admitted with acute pancreatitis. Which laboratory findings
do you expect to be abnormal for this patient?
A. Serum creatinine and BUN
B. Alanine aminotransferase (ALT) and aspartate aminotransferase (AST)
C. Serum amylase and lipase
D. Cardiac enzymes
CORRECT ANSWER: C.
Pancreatitis involves activation of pancreatic enzymes, such as amylase and lipase. These levels
are elevated in a patient with acute pancreatitis.
20. Develop a teaching care plan for Angie who is about to undergo a liver biopsy. Which of
the following points do you include?
A. Youll need to lie on your stomach during the test.
B. Youll need to lie on your right side after the test.
C. During the biopsy youll be asked to exhale deeply and hold it.
D. The biopsy is performed under general anesthesia.
CORRECT ASNWER: B.
After a liver biopsy, the patient is placed on the right side to compress the liver and to reduce the
risk of bleeding or bile leakage.
21. Jerod is experiencing an acute episode of ulcerative colitis. Which is priority for this
patient?
A. Replace lost fluid and sodium

B. Monitor for increased serum glucose level from steroid therapy


C. Restrict the dietary intake of foods high in potassium
D. Note any change in the color and consistency of stools
CORRECT ANSWER: A.
Diarrhea due to an acute episode of ulcerative colitis leads to fluid & electrolyte losses so fluid
replacement takes priority.
22. Anthony, a 60 y.o. patient, has just undergone a bowel resection with a colostomy. During
the first 24 hours, which of the following observations about the stoma should you report
to the doctor?
A. Pink color
B. Light edema
C. Small amount of oozing
D. Trickles of bright red blood
CORRECT ANSWER: D.
After creation of a colostomy, expect to see a stoma that is pink, slightly edematous, with some
oozing. Bright red blood, regardless of amount, indicates bleeding and should be reported to the
doctor.
23. A nurse is monitoring a client admitted to the hospital with a diagnosis of appendicitis.
The client is scheduled for surgery in 2 hours. The client begins to complain of increased
abdominal pain and begins to vomit. On assessment the nurse notes that the abdomen is
distended and the bowel sounds are diminished. Which of the following is the most
appropriate nursing intervention?
A. Administer dilaudid
B. Notify the physician
C. Call and ask the operating room team to perform the surgery as soon as possible
D. Reposition the client and apply a heating pad on a warm setting to the clients
abdomen
CORRECT ANSWER: B.

Based on the signs and symptoms presented in the question, the nurse should suspect peritonitis
and should notify the physician. Administering pain medication is not an appropriate
intervention. Heat should never be applied to the abdomen of a client with suspected
appendicitis. Scheduling surgical time is not within the scope of nursing practice, although the
physician probably would perform the surgery earlier than the prescheduled time.

24. Which goal of the clients care should take priority during the first days of hospitalization
for an exacerbation of ulcerative colitis?
A. Promoting self-care and independence
B. Managing diarrhea
C. Maintaining adequate nutrition
D. Promoting rest and comfort
CORRECT ANSWER: B.
Diarrhea is the primary symptom in an exacerbation of ulcerative colitis, and decreasing the
frequency of stools is the first goal of treatment. The other goals are ongoing and will be best
achieved by halting the exacerbation. The client may receive antidiarrheal medications,
antispasmodic agents, bulk hydrophilic agents, or anti-inflammatory drugs.
25. You promote hemodynamic stability in a patient with upper GI bleeding by:
A. Encouraging oral fluid intake
B. Monitoring central venous pressure
C. Monitoring laboratory test results and vital signs
D. Giving blood, electrolyte and fluid replacement
CORRECT ANSWER: D.
To stabilize a patient with acute bleeding, Normal Saline or Lactated Ringers solution is given
I.V. until BP rises and urine output returns to 30ml/hr.

RENAL SYSTEM
WITH
FLUIDS & ELECTROLYTES

1. A client is admitted with severe diarrhea. Which laboratory value is it most important to
report to the physician?
A. Hgb 10.8 g/dL
B. WBC Count of 12,500/mm
C. BUN 30 mg/dL
D. Potassium 2.0 mEq/L
CORRECT ANSWER: D.
The normal potassium is 3.55.0 mEq/L. Low potassium can be life-threatening and affect every
body system, which makes this answer the priority. Answers A and B are near the normal levels,
so they are incorrect. Answer C is expected in dehydration and can easily be corrected, so it is
incorrect.
2. A client recently started on hemodialysis wants to know how the dialysis will take the
place of his kidneys. The nurses response is based on the knowledge that hemodialysis
works by:
A. Passing water through a dialyzing membrane
B. Eliminating plasma proteins from the blood
C. Lowering the pH by removing nonvolatile acids
D. Filtering waste through a dialyzing membrane
CORRECT ANSWER: D.
Hemodialysis works by using a dialyzing membrane to filter waste that has accumulated in the
blood. It does not pass water through a dialyzing membrane nor does it eliminate plasma
proteins or lower the pH, so answers A, B, and C are incorrect.
3. A client with frequent urinary tract infections asks the nurse how she can prevent the
reoccurrence. The nurse should teach the client to:
A. Douche after intercourse
B. Void every three hours
C. Obtain a urinalysis monthly
D. Wipe from back to front after voiding
CORRECT ANSWER: B.
Voiding every three hours prevents stagnant urine from collecting in the bladder, where bacteria
can grow. Douching is not recommended and obtaining a urinalysis monthly is not necessary,
making answers A and C incorrect. The client should practice wiping from front to back after
voiding and bowel movements, so answer D is incorrect.
4. A client is receiving chemotherapy to treat breast cancer. Which assessment finding
indicates a fluid and electrolyte imbalance induced by chemotherapy?
A. Urine output of 400 ml in 8 hours
B. Serum potassium level of 3.6 mEq/L
C. Blood pressure of 120/64 to 130/72 mm Hg
D. Dry oral mucous membranes and cracked lips
CORRECT ANSWER: D
Chemotherapy commonly causes nausea and vomiting, which may lead to fluid and electrolyte
imbalances. Signs of fluid loss include dry oral mucous membranes, cracked lips, decreased

urine output (less than 40 ml/hour), abnormally low blood pressure, and a serum potassium level
below 3.5 mEq/L.
5. Which of the following symptoms do you expect to see in a patient diagnosed with acute
pyelonephritis?
A. Jaundice and flank pain
B. Costovertebral angle tenderness and chills
C. Burning sensation on urination
D. Polyuria and nocturia
CORRECT ANSWER: B.
Costovertebral angle tenderness, flank pain, and chills are symptoms of acute pyelonephritis.
Jaundice indicates gallbladder or liver obstruction. A burning sensation on urination is a sign of
lower urinary tract infection.
6. You have a patient that is receiving peritoneal dialysis. What should you do when you
notice the return fluid is slowly draining?
A. Check for kinks in the outflow tubing
B. Raise the drainage bag above the level of the abdomen
C. Place the patient in a reverse Trendelenburg position
D. Ask the patient to cough
CORRECT ANSWER: A.
Tubing problems are a common cause of outflow difficulties, check the tubing for kinks and
ensure that all clamps are open. Other measures include having the patient change positions
(moving side to side or sitting up), applying gentle pressure over the abdomen, or having a
bowel movement.
7. Your patient becomes restless and tells you she has a headache and feels nauseous during
hemodialysis. Which complication do you suspect?
A. Infection
B. Disequilibrium syndrome
C. Air embolus
D. Acute hemolysis
CORRECT ANSWER: B.
Disequilibrium syndrome is caused by a rapid reduction in urea, sodium, and other solutes from
the blood. This can lead to cerebral edema and increased intracranial pressure (ICP). Signs and
symptoms include headache, nausea, restlessness, vomiting, confusion, twitching, and seizures.
8. Your patient is complaining of muscle cramps while undergoing hemodialysis. Which
intervention is effective in relieving muscle cramps?
A. Increase the rate of dialysis
B. Infuse normal saline solution

C. Administer a 5% dextrose solution


D. Encourage active ROM exercises
CORRECT ANSWER: B
Treatment includes administering normal saline or hypertonic normal saline solution because
muscle cramps can occur when the sodium and water are removed to quickly during dialysis.
Reducing the rate of dialysis, not increasing it, may alleviate muscle cramps.

9. You expect a patient in the oliguric phase of renal failure to have a 24 hour urine output
less than:
A. 200ml
B. 400ml
C. 800ml
D. 1000ml
CORRECT ANSWER: B
Oliguria is defined as urine output of less than 400ml/24hours.

10. The most common early sign of kidney disease is:


A. Sodium retention
B. Elevated BUN level
C. Development of metabolic acidosis
D. Inability to dilute or concentrate urine
CORRECT ANSWER: B.
Increased BUN is usually an early indicator of decreased renal function.
11. A client is complaining of severe flank and abdominal pain. A flat plate of the abdomen
shows urolithiasis. Which of the following interventions is important?
A. Strain all urine
B. Limit fluid intake
C. Enforce strict bed rest

D. Encourage a high calcium diet


CORRECT ANSWER:A.
Urine should be strained for calculi and sent to the lab for analysis. Fluid intake of 3 to 4 L is
encouraged to flush the urinary tract and prevent further calculi formation. A low-calcium diet is
recommended to help prevent the formation of calcium calculi. Ambulation is encouraged to help
pass the calculi through gravity.
12. A client received a kidney transplant 2 months ago. Hes admitted to the hospital with the
diagnosis of acute rejection. Which of the following assessment findings would be
expected?
A. Hypotension
B. Normal body temperature
C. Decreased WBC count
D. Elevated BUN and creatinine levels
CORRECT ASNWER: D.
In a client with acute renal graft rejection, evidence of deteriorating renal function is expected.
The nurse would see elevated WBC counts and fever because the body is recognizing the graft as
foreign and is attempting to fight it. The client would most likely have acute hypertension.
13. The nurse suspects that a client with polyuria is experiencing water diuresis. Which
laboratory value suggests water diuresis?
A. High urine specific gravity
B. High urine osmolarity
C. Normal to low urine specific gravity
D. Elevated urine pH
CORRECT ANSWER: C.
Water diuresis causes low urine specific gravity, low urine osmolarity, and a normal to elevated
serum sodium level. High specific gravity indicates dehydration. Hypernatremia signals acidosis
and shock. Elevated urine pH can result from potassium deficiency, a high-protein diet, or
uncontrolled diabetes.

14. The client passes a urinary stone, and lab analysis of the stone indicates that it is

composed of calcium oxalate. Based on this analysis, which of the following would the
nurse specifically include in the dietary instructions?
A. Increase intake of meat, fish, plums, and cranberries
B. Avoid citrus fruits and citrus juices
C. Avoid green, leafy vegetables such as spinach
D. Increase intake of dairy products
CORRECT ANSWER: C.
Oxalate is found in dark green foods such as spinach. Other foods that raise urinary oxalate are
rhubarb, strawberries, chocolate, wheat bran, nuts, beets, and tea.
15. Which of the following interventions would be most appropriate for preventing the
development of a paralytic ileus in a client who has undergone renal surgery?
A. Encourage the client to ambulate every 2 to 4 hours
B. Offer 3 to 4 ounces of a carbonated beverage periodically.
C. Encourage use of a stool softener
D. Continue intravenous fluid therapy
CORRECT ASNWER: A.
Ambulation stimulates peristalsis. A client with paralytic ileus is kept NPO until peristalsis
returns. Intravenous fluid infusion is a routine postoperative order that does not have any effect
on preventing paralytic ileus. A stool softener will not stimulate peristalsis.

16. Because a clients renal stone was found to be composed to uric acid, a low-purine,
alkaline-ash diet was ordered. Incorporation of which of the following food items into the
home diet would indicate that the client understands the necessary diet modifications?
A. Milk, apples, tomatoes, and corn
B. Eggs, spinach, dried peas, and gravy
C. Salmon, chicken, caviar, and asparagus

D. Grapes, corn, cereals, and liver


CORRECT ANSWER: A.
Because a high-purine diet contributes to the formation of uric acid, a low-purine diet is
advocated. An alkaline-ash diet is also advocated, because uric acid crystals are more likely to
develop in acid urine. Foods that may be eaten as desired in a low-purine diet include milk, all
fruits, tomatoes, cereals, and corn. Food allowed on an alkaline-ash diet include milk, fruits
(except cranberries, plums, and prunes), and vegetables (especially legumes and green
vegetables). Gravy, chicken, and liver are high in purine.

17. A patient diagnosed with sepsis from a UTI is being discharged. What do you plan to
include in her discharge teaching?
A. Take cool baths
B. Avoid tampon use
C. Avoid sexual activity
D. Drink 8 to 10 eight-oz glasses of water daily
CORREACT ANSWER: D.
Drinking 2-3L of water daily inhibits bacterial growth in the bladder and helps flush the
bacteria from the bladder. The patient should be instructed to void after sexual activity.

18. Youre planning your medication teaching for your patient with a UTI prescribed
phenazopyridine (Pyridium). What do you include?
A. Your urine might turn bright orange.
B. You need to take this antibiotic for 7 days.
C. Take this drug between meals and at bedtime.
D. Dont take this drug if youre allergic to penicillin.
CORRECT ANSWER: A.
The drug turns the urine orange. It may be prescribed for longer than 7 days and is usually
ordered three times a day after meals. Phenazopyridine is an azo (nitrogenous) analgesic; not an
antibiotic.

19. What is the priority nursing diagnosis with your patient diagnosed with end-stage renal
disease?
A. Activity intolerance
B. Fluid volume excess
C. Knowledge deficit
D. Pain
CORRECT ANSWER:B.
Fluid volume excess because the kidneys arent removing fluid and wastes. The other diagnoses
may apply, but they dont take priority.

20. What is the best way to check for patency of the arteriovenous fistula for hemodialysis?
A. Pinch the fistula and note the speed of filling on release
B. Use a needle and syringe to aspirate blood from the fistula
C. Check for capillary refill of the nail beds on that extremity
D. Palpate the fistula throughout its length to assess for a thrill
CORRECT ANSWER: D.
The vibration or thrill felt during palpation ensures that the fistula has the desired turbulent
blood flow. Pinching the fistula could cause damage. Aspirating blood is a needless invasive
procedure.
21. Your patient returns from the operating room after abdominal aortic aneurysm repair.
Which symptom is a sign of acute renal failure?
A. Anuria
B. Diarrhea
C. Oliguria
D. Vomiting

CORRECT ANSWER: C.
Urine output less than 50ml in 24 hours signifies oliguria, an early sign of renal failure. Anuria
is uncommon except in obstructive renal disorders.

22. Which criterion is required before a patient can be considered for continuous peritoneal
dialysis?
A. The patient must be hemodynamically stable
B. The vascular access must have healed
C. The patient must be in a home setting
D. Hemodialysis must have failed
CORRECT ANSWER: A.
Hemodynamic stability must be established before continuous peritoneal dialysis can be started.

23. Polystyrene sulfonate (Kayexalate) is used in renal failure to:


A. Correct acidosis
B. Reduce serum phosphate levels
C. Exchange potassium for sodium
D. Prevent constipation from sorbitol use
CORRECT ANSWER: C. In renal failure, patients become hyperkalemic because they cant
excrete potassium in the urine. Polystyrene sulfonate acts to excrete potassium by pulling
potassium into the bowels and exchanging it for sodium.

24. Your patient has complaints of severe right-sided flank pain, nausea, vomiting and
restlessness. He appears slightly pale and is diaphoretic. Vital signs are BP 140/90
mmHg, Pulse 118 beats/min., respirations 33 breaths/minute, and temperature, 98.0F.
Which subjective data supports a diagnosis of renal calculi?
A. Pain radiating to the right upper quadrant

B. History of mild flu symptoms last week


C. Dark-colored coffee-ground emesis
D. Dark, scant urine output
CORRECT ANWER: D.
Patients with renal calculi commonly have blood in the urine caused by the stones passage
through the urinary tract. The urine appears dark, tests positive for blood, and is typically scant.
25. A patient returns from surgery with an indwelling urinary catheter in place and empty. Six
hours later, the volume is 120ml. The drainage system has no obstructions. Which
intervention has priority?
A. Give a 500ml bolus of isotonic saline
B. Evaluate the patients circulation and vital signs
C. Flush the urinary catheter with sterile water or saline
D. Place the patient in the shock position, and notify the surgeon
CORRECT ANSWER: B.
A total UO of 120ml is too low. Assess the patients circulation and hemodynamic stability for
signs of hypovolemia. A fluid bolus may be required, but only after further nursing assessment
and a doctors order.

MUSCULOSKELETAL
SYSTEM

1. The nurse is caring for several clients on an orthopedic unit. If the following requests
were made simultaneously, which should the nurse do first?
A. A family member calls and requests to speak to the nurse about the patient.
B. A client with a fractured femur complains of pleuritic pain and reports a rash on
the chest and under the arms.
C. A client with a fractured leg requests pain medication
D. A client with a cast reports itching within the cast.
CORRECT ANSWER: B.
The client with pleuritic chest pain and a rash in these areas should be assessed first due to
clinical manifestations of an embolus. The situation in Answer A could be done later by the
nurse calling the family member back, so it is incorrect. The clients in Answers C and D could
have their care delegated to others.
2. Following amputation of the lower extremity, a patient with prosthesis should be
educated to:
A. Wear the prosthesis daily, but remove immediately when discomfort is
experienced.
B. Adjust the fit of the prosthesis by wearing a heavier sock to ensure a tight fit.
C. To put the prosthesis on immediately upon arising in the morning and keep it on
all day.
D. To apply oil or lotion to the stump before applying the prosthesis.
CORRECT ANSWER: C.
The wearing of prosthesis upon arising in the morning will prevent swelling of the stump. The
use of lotions and creams should be avoided as they can result in the excessive softening of the
skin. It can also be a cause of potential bacterial formation and infection. The prosthesis should
also never be adjusted by patient but only by a trained professional.
3. Which of the following interventions is used to prevent compartment syndrome in a
patient with a newly casted right lower extremity?
A. Elevation of right lower extremity
B. Application of heat
C. Encourage ambulation to increase circulation
D. Monitor signs of DVT
CORRECT ANSWER: A.
Compartment syndrome is caused by edema. Elevation of extremity and application of ice are the
best interventions to decrease edema.
4. The client is admitted following cast application for a fractured ulna. Which finding
should be reported to the doctor?
A. Pain at the site
B. Warm fingers
C. Pulses rapid
D. Paresthesia of the fingers
CORRECT ANSWER: D.
At this time, pain beneath the cast is normal. The clients fingers should be warm to touch, and
pulses should be present. Parethesia is not normal and may indicate compartment syndrome.

Therefore answers A, B, and C are incorrect.


5. Colchicine is prescribed for a client with a diagnosis of gout. The nurse reviews the
clients record, knowing that this medication will be used with caution in which disorder?
A. Myxedema
B. Kidney disease
C. Hypothyroidism
D. Diabetes mellitus
CORRECT ANSWER: B.
Colchicine is used with caution in older clients, debilitated clients, or clients with
cardiac, kidney or gastrointestinal disease. The disorders in A, C, and D are not
concerns with the administration of this drug.

6. A two-year-old is admitted for repair of a fractured femur and is placed in Bryants


traction. Which finding by the nurse indicates that the traction is working properly?
A. The infant no longer complains of pain.
B. The buttocks are 15 off the bed.
C. The legs are suspended in the traction.
D. The pins are secured within the pulley.
CORRECT ANSWER: B.
The infants hips should be off the bed approximately 15 in Bryants traction. Answer A is
incorrect because this does not indicate that the traction is working correctly, nor does C.
Answer D is incorrect because Bryants traction is a skin traction, not a skeletal traction.
7. The client is admitted for an open reduction internal fixation of a fractured hip.
Immediately following surgery, the nurse should give priority to assessing the:
A. Serum collection (Davol) drain
B. Clients pain
C. Nutritional status
D. Immobilizer
CORRECT ANSWER: A.
Bleeding is a common complication of orthopedic surgery. The blood-collection device should be
checked frequently to ensure that the client is not hemorrhaging. The clients pain should be
assessed, but this is not life-threatening. When the client is in less danger, the nutritional status
should be assessed and an immobilizer is not used; thus, answers B, C, and D are incorrect.
8. A client with a fractured tibia has a plaster-of-Paris cast applied to immobilize the
fracture. Which action by the nurse indicates understanding of a plaster-of-Paris cast?
The nurse:
A. Handles the cast with the fingertips
B. Petals the cast
C. Dries the cast with a hair dryer

D. Allows 24 hours before bearing weight


CORRECT ANSWER: D.
A plaster-of-Paris cast takes 24 hours to dry, and the client should not bear weight for 24 hours.
The cast should be handled with the palms, not the fingertips, so answer A is incorrect. Petaling
a cast is covering the end of the cast with cast batting or a sock, to prevent skin irritation and
flaking of the skin under the cast. B is incorrect because petaling the cast is done by the health
care provider who applied the cast. The client should be told not to dry the cast with a hair dryer
because this causes hot spots and could burn the client. This also causes unequal drying; thus,
answer C is incorrect.
9. A child with Scoliosis has a spica cast applied. Which action specific to spica cast should
be taken?
A. Check the bowel sounds
B. Assess blood pressure
C. Offer pain medication
D. Check for swelling
CORRECT ANSWER: A.
A body cast or spica cast extends from the upper abdomen to the knees or below. Bowel sounds
should be checked to ensure that the client is not experiencing a paralytic ileus. Checking the
blood pressure is a treatment for any client, offering pain medication is not called for, and
checking for swelling isnt specific to the stem, so answers B, C, and D are incorrect.
10.Primary prevention of osteoporosis includes which measure?
A. Place items within reach of the client.
B. Install bars in the bathroom to prevent falls.
C. Maintain the optimal calcium intake.
D. Use a professional alert system in the home in case a fall occurs when
the client is alone.
CORRECT ANSWER: C.
Primary prevention of osteoporosis includes maintaining optimal calcium intake.
Placing items within reach of the client, using a professional alert system in the
home, and installing bars in bathrooms are all secondary and tertiary prevention
methods to prevent falls.
11.A nurse is caring for a patient who has had hip replacement. The nurse
should be most concerned about which of the following findings?
A. Complaints of pain during repositioning

B. Scant bloody discharge on the surgical dressing


C. Complaints of pain following physical therapy
D. Temperature of 101.8 F (38.7 C)
CORRECT ANSWER: D.
Post-surgical nursing assessment after hip replacement should be principally
concerned with the risk of neurovascular complications and the development of
infection. A temperature of 101.8 F (38.7 C) postoperatively is higher than the low
grade that is to be expected and should raise concern. Some pain during
repositioning and following physical therapy is to be expected and can be managed
with analgesics. A small amount of bloody drainage on the surgical dressing is a
result of normal healing.
12.The nurse is performing pin site care on a client in skeletal traction. Which
normal finding should the nurse expect to note when assessing the pin sites?
A. Loose pin sites
B. Clear drainage form the pin sites
C. Purulent drainage from the pin sited
D. Redness and selling around the pin sites
CORRECT ANSWER: B.
A small amount of clear drainage (weeping) may be expected after cleaning and
removing crusting around the pin sites of skeletal traction. Pins should not be loose;
if this is noted, the heath care provider should be notified. Purulent drainage,
redness and swelling around the pin sites may be indicative of an infection.
13.The nurse is caring for a client who has been placed in Bucks extension
traction. Which action by the nurse provides for counter traction to reduce
shear and friction?
A. Using a foot board
B. Providing an overhead trapeze
C. Slightly elevating the foot of the bed
D. Slightly elevating the head of the bed

CORRECT ANSWER: C.
The part of the bed under an area in traction is usually elevated to aid in countertraction. For the client in Bucks extension traction (which is applied to a leg), the
foot of the bed is elevated. Option C provides a force that opposes the traction force
effectively without harming the client. A footboard, an overhead trapeze or
elevating the head of the bed are not used to provide counter-traction.

14. The nurse is caring for a client with a diagnosis of gout. Which laboratory diagnosis
would the nurse note in the client?
A. Calcium level of 9.0 mg/dL
B. Uric acid level of 8.0 mg/dL
C. Potassium level of 4.1 mg/dL
D. Phosphorus level of 3.1 mg/dL
CORRECT ANSWER: B.
In addition to the presence of clinical manifestations, gout is diagnosed by the persistence of
hyperuricemia, with a uric acid level higher than 8 mg/dL; a normal value ranges from 2.5 to 8
mg/dL. Options A, C, and D indicate normal laboratory values. In addition, the presence of uric
acid in an aspirated sample of synovial fluid confirms the diagnosis.
15. The nurse prepares to teach a client ambulate with a cane. Before teaching cane-assisted
ambulation, what should the nurse check for as a priority?
A. A high level of stamina and energy
B. Self-consciousness about using a cane
C. Full range of motion in lower extremities
D. Balance, muscle strength and confidence
CORRECT ANSWER: D.
Assessing the clients balance, strength and confidence helps determine if the can is a suitable
assistive device for the client. A high level of stamina and full range of motion are not needed for
walking with a cane. Although body image (self-consciousness) is a component of the
assessment, it is not the priority.
16. The nurse develops postoperative plan of care for a client undergoing arthroscopy. The
nurse should include which priority action in the plan?
A. Monitor intake and output
B. Assess the tissue at the surgical site
C. Monitor the area for numbness or tingling
D. Assess the complete blood cell counts result
CORRECT ANSWER: C.
Arthroscopy provides an endoscopic examination of the joint and is used to diagnose and treat
acute and chronic disorders of the joint. The priority nursing action is to monitor the affected
area for numbness or tingling. Options A, B, and D are also components of postoperative care,
but from the options presented, are not the initial priorities.

17. The nurse has given instructions to a client returning home after knee arthroscopy. Which
statement by the client indicates that the instructions are understood?
A. I can resume regular exercise tomorrow.
B. I cant eat food for the remainder of the day.
C. I need to stay off the leg entirely for the rest of the day.
D. I need to report fever or site inflammation to my health care provider.
CORRECT ANSWER: D.
After arthroscopy, the client can usually walk carefully once the sensation on the leg has
returned. The client is instructed to avoid strenuous exercise for at least a few days. The client
may resume the usual diet. Signs and symptoms f infection should be reported to the health care
provider.
18. The nurse is admitting a client with multiple traumas to the nursing unit. The client has a
leg fracture and had a plaster cast applied. Which position would be best for the casted
leg?
A. Flat for 12 hours, the elevated for 12 hours
B. Elevated for 3 hours, and then flat for 1 hour
C. Flat for 3 hours, then elevated for 1 hour
D. Elevated on pillows continuously for 24 to 48 hours
CORRECT ANSWER: D.
A casted extremity is elevated continuously for the first 24 to 48 hours to minimize swelling and
to promote venous drainage. Options A, B, and C are incorrect.
19. Allopurinol (Zyloprim) is prescribed for a client and the nurse provides medication
instructions to the client. Which instruction should the nurse provide?
A. Drink 3000ml of fluid a day
B. Take the medication on an empty stomach
C. The effect of the medication will occur immediately
D. Any swelling of the lips is a normal expected response
CORRECT ANSWER: A.
Clients taking allopurinol are encouraged to drink 3000ml of fluid a day. A full therapeutic effect
may take 1 week or longer. Allopurinol is to be given with, or immediately after, meals or milk. A
client who develops a rash, irritation of the eyes, or swelling of the lips or mouth should contact
the health care provider because this may indicate hypersensitivity.
20. The nurse is administering an intravenous dose of methocarbamol (Robaxin) to a client
with multiple sclerosis. For which side/adverse effects should the nurse monitor?
A. Tachycardia
B. Rapid pulse
C. Bradycardia
D. Hypertension
CORRECT ANSWER: C.
IV administration of methocarbamol can cause hypotension and bradycardia. Options A, B and

D are not side effects of this medication.


21. The nurse is caring for a client who has had spinal fusion, with insertion of hardware. The
nurse would be most concerned with which assessment finding?
A. Temperature of 101.6oF orally
B. Complaints of discomfort during repositioning
C. Old bloody drainage outlined on the surgical dressing
D. Discomfort during coughing and deep breathing exercises
CORRECT ANSWER: A.
The nursing assessment conducted after spinal surgery is similar to that done after other
surgical procedures. For this specific type of surgery, the nurse assesses the neurovascular status
of the lower extremities, watches for signs and symptoms of infection, and inspects the surgical
site foe evidence of cerebrospinal fluid leakage (clear drainage and tests positive for glucose). A
mild temperature is expected after insertion of hardware, but a temperature of 101.6oF should be
reported.
22. The nurse is one of several persons who witnessed a vehicle hit a pedestrian at fairly low
speed on a small street. The victim is dazed and tries to get up. The leg appears fractured.
Which intervention should the nurse take?
A. Try to reduce the fracture manually
B. Assist the client to get up and walk to the sidewalk
C. Leave the victim for a few moment to call an ambulance
D. Stay with the victim and encourage the person to remain still
CORRECT ASNWER: D.
With a suspected fracture, the victim is not moved unless it is dangerous to remain in that spot.
The nurse should remain with the victim and have someone else call for emergency help. A
fracture is not reduced at the scene. Before the victim is moved, the site of fracture should be
immobilized to reduce further injury.
23. The nurse is preparing discharge instructions for a client receiving baclofen. Which
instruction should be included in the teaching plan?
A. Restrict fluid intake
B. Avoid use of alcohol
C. Stop the medication if diarrhea occurs
D. Notify the health care provider (HCP) if fatigue occurs
CORRECT ANSWER: B.
Baclofen is a skeletal muscle relaxant. The client should be cautioned against the use of alcohol
and other central nervous system depressants because baclofen potentiates the depressant
activity of these agents. Constipation rather than diarrhea is a side/adverse effect. Restriction of
fluids is not necessary, but the client should be warned that urinary retention may occur. Fatigue
is related to a central nervous system effect that is most intense during the early phase of therapy
and diminished with continued medication use. The client does not need to notify the HCP about
fatigue.

24. A client is complaining of low back pain that radiates down the left posterior thigh. The
nurse should ask the client if the pain is worsened or aggravated by which factor?
A. Bed rest
B. Bending or lifting
C. Application of heat
D. Ibuprofen (Motrin IB)
CORRECT ANSWER: B.
Low back pain that radiates into one leg (sciatica) is consistent with herniated lumbar disk. The
nurse assess the client to see whether the pain is aggravated by events that increase intraspinal
pressure, such as bending, lifting, sneezing and coughing, or by lifting the leg straight up while
supine (straight leg-raising test). Bed rest, heat (or sometimes ice), and nonsteroidal antiinflammatory drugs usually relieve back pain.
25. The nurse is caring for a client who had an above-knee amputation two days ago. The
residual limb was wrapped with an elastic compression bandage, which has come off.
Which immediate action should the nurse take?
A. Apply ice to the site
B. Call the health care provider (HCP)
C. Apply a dry sterile dressing and elevate it on one pillow
D. Rewrap the limb with an elastic compression bandage
CORRECT ANSWER: D.
If the client with an amputation has a cast or elastic compression bandage that slips off, the
nurse must wrap the residual limb immediately with another elastic compression bandage.
Otherwise, excessive edema will form rapidly, which could cause a significant delay in
rehabilitation. If the client had a cast slip off, the nurse would have to call the HCP so that a
new one could be applied. Elevation on one pillow is not going to impede the development of
edema greatly once compression is released. Ice would be of limited value on controlling edema
from this cause. If the HCP was called, the prescription likely would be to reapply the
compression dressing anyway.

ONCOLOGY
&
PAIN MANAGEMENT

1. The charge nurse is assigning staff on an oncology unit. One of the staff is a nurse that
was transferred from the psychiatric unit. Which client should the nurse assign to the
psychiatric nurse?
A. A client from the intensive care unit who is post-thoracotomy for lung cancer.
B. A client requiring extensive chemotherapy administration on this shift
C. A client who has just been told that her cancer is terminal
D. A client with metastatic colon cancer, returning from a colon resection this shift
CORRECT ANSWER: A.
The most suitable roommates would be the two similar clients with palliative treatments. The
clients in Answers B and C each involve a risk for infection due to their low WBC counts.
Answer D also has some similarities in diagnoses, but do not fit as well as the clients in Answer
A.
2. Which nurse would be best to assign to the client who is 48 hours bone marrow
transplant?
A. The RN with 4 years experience in the emergency room setting
B. The RN with 4 years experience on the geriatric unit
C. The RN with 5 years experience working with clients with AIDS
D. The RN with 10 years experience working in the labor and delivery unit
CORRECT ANSWER: C.
The nurse with five years of experience working with clients with AIDS is most aware of the
needs of the immune-suppressed client. The nurses with experience in the other units as noted in
A, B, and D are not as well prepared to care for this client, so these answers are incorrect.
3. A client with vaginal cancer is being treated with a radioactive vaginal implant. The
clients husband asks the nurse if he can spend the night with his wife. The nurse should
explain that:
A. Overnight stays by family members is against hospital policy.
B. There is no need for him to stay because staffing is adequate.
C. His wife will rest much better knowing that he is at home.
D. Visitation is limited to 30 minutes when the implant is in place.
CORRECT ANSWER: D.
Clients with radium implants should have close contact limited to 30 minutes per visit. The
general rule is limiting time spent exposed to radium, putting distance between people and the
radium source, and using lead to shield against radium. Teaching the family members these
principles are important. Choices A, B, and C are not empathetic and do not address the
question; therefore they are incorrect.
4. Which of the following laboratory values is expected for a client just diagnosed with
chronic lymphocytic leukemia?
A. Elevated sedimentation rate
B. Uncontrolled proliferation of granulocytes
C. Thrombocytopenia and increased lymphocytes

D. Elevated aspartate aminotransferase and alanine aminotransferase levels.


CORRECT ANSWER:C.
Chronic lymphocytic leukemia shows a proliferation of small abnormal mature B lymphocytes
and decreased antibody response. Thrombocytopenia also is often present. Uncontrolled
proliferation of granulocytes occurs in myelogenous leukemia.
5. At the time of diagnosis of Hodgkins lymphoma, which of the following areas is often
involved?
A. Back
B. Chest
C. Groin
D. Neck
CORRECT ANSWER: D.
At the time of diagnosis, a painless cervical lesion is often present. The back, chest, and groin
areas arent involved.
6. Which of the following statements is correct about the rate of cell growth in relation to
chemotherapy?
A. Faster growing cells are less susceptible to chemotherapy.
B. Non-dividing cells are more susceptible to chemotherapy
C. Faster growing cells are more susceptible to chemotherapy
D. Slower growing cells are more susceptible to chemotherapy.
CORRECT ANSWER: C.
The faster the cell grows, the more susceptible it is to chemotherapy and radiation therapy.
Slow-growing and non-dividing cells are less susceptible to chemotherapy. Repeated cycles of
chemotherapy are used to destroy nondividing cells as they begin active cell division.
7. Which of the following substances has abnormal values early in the course of multiple
myeloma (MM)?
A. Immunoglobulins
B. Platelets
C. Red blood cells
D. White blood cells
CORRECT ANSWER: A.
Multiple Myeloma is characterized by malignant plasma cells that produce an increased amount
of immunoglobin that isnt functional. As more malignant plasma cells are produced, theres less
space in the bone marrow for RBC production. In late stages, platelets and WBCs are reduced
as the bone marrow is infiltrated by malignant plasma cells.
8. When a client has a lobectomy, what fills the space where the lobe was?
A. The space stays empty
B. The surgeon fills the space with gel

C. The lung space fills up with serous fluid


D. The remaining lobe or lobes overexpand to fill the space
CORRECT ANSWER: D.
The remaining lobe or lobes overexpand slightly to fill the space previously occupied by the
removed tissue. The diaphragm is carried higher on the operative side to further reduce the
empty space. The space cant remain empty because truly empty would imply a vacuum, which
would interfere with the intrathoracic pressure changes that allow breathing. The surgeon
doesnt use a gel to fill the space. Serous fluid overproduction would compress the remaining
lobes, diminish their function and possibly, cause a mediastinal shift.
9. Which of the following is the primary goal for surgical resection of lung cancer?
A. To remove the tumor and all surrounding tissue
B. To remove the tumor and as little surrounding tissue as possible
C. To remove all of the tumor and any collapsed alveoli in the same region
D. To remove as much as the tumor as possible, without removing any alveoli
CORRECT ANSWER: B.
The goal of surgical resection is to remove the lung tissue that has a tumor in it while saving as
much surrounding tissue as possible. It may be necessary to remove alveoli and bronchioles, but
care is taken to make sure only whats absolutely necessary is removed.
10. Which of the following assessment findings in a client with leukemia would indicate that
the cancer has invaded the brain?
A. Headache and vomiting
B. Restlessness and tachycardia
C. Hypervigilant and anxious behavior
D. Increased heart rate and decreased blood pressure
CORRECT ANSWER: A.
The usual effect of leukemic infiltration of the brain is increased intracranial pressure. The
proliferation of cells interferes with the flow of cerebrospinal fluid in the subarachnoid space
and at the base of the brain. The increased fluid pressure causes dilation of the ventricles, which
creates symptoms of severe headache, vomiting, irritability, lethargy, increased blood pressure,
decreased heart rate, and eventually, coma. Often children with a variety of illnesses are hypervigilant and anxious when hospitalized.
11. Which of the following tests in performed on a client with leukemia before initiation of
therapy to evaluate the childs ability to metabolize chemotherapeutic agents?
A. Lumbar puncture
B. Liver function studies
C. Complete blood count (CBC)
D. Peripheral blood smear
CORRECT ANSWER: B.
Liver and kidney function studies are done before initiation of chemotherapy to evaluate the

childs ability to metabolize the chemotherapeutic agents. A CBC is performed to assess for
anemia and white blood cell count. A peripheral blood smear is done to assess the maturity and
morphology of red blood cells. A lumbar puncture is performed to assess for central nervous
system infiltration.
12. A client is diagnosed with multiple myeloma. The client asks the nurse about the
diagnosis. The nurse bases the response on which of the following descriptions of this
disorder?
A. Malignant exacerbation in the number of leukocytes
B. Altered red blood cell production
C. Altered production of lymph nodes
D. Malignant proliferation of plasma cells and tumors within the bone
CORRECT ANSWER: D.
Multiple myeloma is a B cell neoplastic condition characterized by abnormal malignant
proliferation of plasma cells and the accumulation of mature plasma cells in the bone marrow.
Option A describes the leukemic process. Options B and C are not characteristics of multiple
myeloma.
13. The nurse is developing a plan of care for the client with multiple myeloma. The nurse
includes which priority intervention in the plan of care?
A. Coughing and deep breathing
B. Encouraging fluids
C. Monitoring red blood cell count
D. Providing frequent oral care
CORECT ANSWER: B.
Hypercalcemia caused by bone destruction is a priority concern in the client with multiple
myeloma. The nurse should administer fluids in adequate amounts to maintain and output of 1.5
to 2 L a day. Clients require about 3 L of fluid pre day. The fluid is needed not only to dilute the
calcium overload but also to prevent protein from precipitating in renal tubules. Options A, C,
and D may be components in the plan of care but are not the priority in this client.
14. The nurse is reviewing the laboratory results of a client receiving chemotherapy. The
platelet count is 10,000 cells/mm. Based on this laboratory value, the priority nursing
assessment is which of the following?
A. Assess level of consciousness
B. Assess temperature
C. Assess bowel sounds
D. Assess skin turgor
CORRECT ANSWER: A.
A high risk of hemorrhage exists when the platelet count is fewer than 20,000. Fatal central
nervous system hemorrhage or massive gastrointestinal hemorrhage can occur when the platelet
count is fewer than 10,000. The client should be assessed for changes in levels of consciousness,
which may be an early indication of an intracranial hemorrhage. Option B is a priority nursing

assessment when the white blood cell count is low and the client is at risk for an infection.
15. The nurse is caring for a client following a modified radical mastectomy. Which
assessment finding would indicate that the client is experiencing a complication related to
this surgery?
A. Sanguineous drainage in the Jackson-Pratt drain
B. Pain at the incisional site
C. Complaints of decreased sensation near the operative site
D. Arm edema on the operative side
CORRECT ANSWER: D.
Arm edema on the operative side (lymphedema) is a complication following mastectomy and can
occur immediately postoperatively or may occur months or even years after surgery. The other
options are expected occurrences.
16. A 32-year-old woman meets with the nurse on her first office visit since undergoing a left
mastectomy. When asked how she is doing, the woman states her appetite is still not
good, she is not getting much sleep because she doesnt go to bed until her husband is
asleep, and she is really anxious to get back to work. Which of the following nursing
interventions should the nurse explore to support the clients current needs?
A. Call the physician to discuss allowing the client to return to work earlier
B. Suggest that the client learn relaxation techniques to help with her insomnia
C. Perform a nutritional assessment to assess for anorexia
D. Ask open-ended questions about sexuality issues related to her mastectomy
CORRECT ANSWER: D.
The content of the clients comments suggests that she is avoiding intimacy with her husband by
waiting until he is asleep before going to bed. Addressing sexuality issues is appropriate for a
client who has undergone a mastectomy. Rushing her return to work may debilitate her and add
to her exhaustion. Suggesting that she learn relaxation techniques to help her with her insomnia
is appropriate; however, the nurse must first address the psychosocial and sexual issues that are
contributing to her sleeping difficulties. A nutritional assessment may be useful, but there is no
indication that she has anorexia.
17. A 56-year-old woman is currently receiving radiation therapy to the chest wall for
recurrent breast cancer. She calls her health care provider to report that she has pain while
swallowing and burning and tightness in her chest. Which of the following complications
of radiation therapy is most likely responsible for her symptoms?
A. Hiatal hernia
B. Stomatitis
C. Radiation enteritis
D. Esophagitis
CORRECT ANSWER: D.
Difficulty in swallowing, pain, and tightness in the chest are signs of esophagitis, which is a
common complication of radiation therapy of the chest wall.

18. Which of the following diagnostic tests should be performed annually over age 50 to
screen for colon cancer?
A. Abdominal CT scan
B. Abdominal x-ray
C. Colonoscopy
D. Fecal occult blood test
CORRECT ANSWER: D.
Surface blood vessels of polyps and cancers are fragile and often bleed with the passage of
stools. Abdominal x-ray and CT scan can help establish tumor size and metastasis. A
colonoscopy can help locate a tumor as well as polyps, which can be removed before they
become malignant.
19. Which of the following diagnostic tests may be performed to determine if a client has
gastric cancer?
A. Barium enema
B. Colonoscopy
C. Gastroscopy
D. Serum chemistry levels
20. A client with gastric cancer can expect to have surgery for resection. Which of the
following should be the nursing management priority for the preoperative client with
gastric cancer?
A. Discharge planning
B. Correction of nutritional deficits
C. Prevention of DVT
D. Instruction regarding radiation treatment
CORRECT ANSWER: C.
A gastroscopy will allow direct visualization of the tumor. A colonoscopy or a barium enema
would help diagnose colon cancer. Serum chemistry levels dont contribute data useful to the
assessment of gastric cancer.
21. Care for the postoperative client after gastric resection should focus on which of the
following problems?
A. Body image
B. Nutritional needs
C. Skin care
D. Spiritual needs
CORRECT ANSWER: B.
After gastric resection, a client may require total parenteral nutrition or jejunostomy tube
feedings to maintain adequate nutritional status.
22. What information is correct about stomach cancer?

A.
B.
C.
D.

Stomach pain is often a late symptom


Surgery is often a successful treatment
Chemotherapy and radiation are often successful treatments
The patient can survive for an extended time with TPN

CORRECT ANSWER: A.
Stomach pain is often a late sign of stomach cancer; outcomes are particularly poor when the
cancer reaches that point. Surgery, chemotherapy, and radiation have minimal positive effects.
TPN may enhance the growth of the cancer.
23. A client is receiving a radiation implant for the treatment of bladder cancer. Which of the
following interventions is appropriate?
A. Flush all urine down the toilet
B. Restrict the clients fluid intake
C. Place the client in a semi-private room
D. Monitor the client for signs and symptoms of cystitis
CORRECT ANSWER: D.
Cystitis is the most common adverse reaction of clients undergoing radiation therapy; symptoms
include dysuria, frequency, urgency, and nocturia. Clients with radiation implants require a
private room. Urine of clients with radiation implants for bladder cancer should be sent to the
radioisotopes lab for monitoring. It is recommended that fluid intake be increased.
24. The community health nurse provides an educational session regarding the risk factors
for cervical cancer to women in the local community. The nurse determines that further
teaching is needed if a woman attending the session identifies which as a risk factor for
this type of cancer?
A. Smoking tobacco
B. Single sex partner
C. Early first intercourse
D. Human papillomavirus (HPV) infection
CORRECT ANSWER: B.
Some risk factors for cervical cancer include having multiple sexual partners or a partner who
had multiple sexual partners, smoking tobacco, early age of first intercourse, and HPV infection.
25. The nurse has completed discharge teaching with a client who has had surgery for lung
cancer. The nurse determines that the client needs additional teaching about the elements
of home management if the client verbalizes the need to follow which instruction?
A. Avoid exposure to crowds
B. Deal with any increases in pain independently
C. Sit up and lean forward to breathe more easily
D. Call the health care provider in case of increased temperature or shortness of
breath
CORRECT ANSWER: B.
The client who has had surgery for lung cancer should not be expected to deal with increases in

pain independently. Health teaching includes avoiding exposure to crowds or persons with
respiratory infections and reporting signs and symptoms of respiratory infection of increases in
pain. The client should also use positions that facilitate respiration such as sitting up and
leaning forward.

ENDOCRINE
SYSTEM

1. The client with a history of diabetes insipidus is admitted with polyuria, polydipsia, and
mental confusion. The priority intervention for this client is:
A. Measure the urinary output.
B. Check the vital signs.
C. Encourage increased fluid intake.
D. Weigh the client
CORRECT ANSWER: B.
The large amount of fluid loss can cause fluid and electrolyte imbalance that should be
corrected. The loss of electrolytes would be reflected in the vital signs. Measuring the urinary
output is important, but the stem already says that the client has polyuria, so answer A is
incorrect. Encouraging fluid intake will not correct the problem, making answer C incorrect.
Answer D is incorrect because weighing the client is not necessary at this time.
2. A client has had a unilateral adrenalectomy to remove a tumor. The most important
measurement in the immediate post-operative period for the nurse to take is:
A. The blood pressure
B. The temperature
C. The urinary output
D. The specific gravity of the urine
CORRECT ANSWER: A.
Blood pressure is the best indicator of cardiovascular collapse in the client who has had an
adrenal gland removed. The remaining gland might have been suppressed due to the tumor
activity. Temperature would be an indicator of infection, decreased output would be a clinical
manifestation but would take longer to occur than blood pressure changes, and specific gravity
changes occur with other disorders; therefore, answers B, C, and D are incorrect
3. A client with Addisons disease has been admitted with a history of nausea and vomiting
for the past three days. The client is receiving IV glucocorticoids (Solu-Medrol). Which
of the following interventions would the nurse implement?
A. Glucometer readings as ordered
B. Intake/output measurements
C. Evaluating the sodium and potassium levels
D. Daily weights
CORRECT ANSWER: A.
IV glucocorticoids raise the glucose levels and often require coverage with insulin. Answer B is
not necessary at this time, sodium and potassium levels would be monitored when the client is
receiving mineral corticoids, and daily weights is unnecessary; therefore, answers B, C, and D
are incorrect.
4. A client had a total thyroidectomy yesterday. The client is complaining of tingling around
the mouth and in the fingers and toes. What would the nurses next action be?
A. Obtain a crash cart.
B. Check the calcium level.
C. Assess the dressing for drainage.

D. Assess the blood pressure for hypertension.


CORRECT ANSWER: B.
The parathyroid glands are responsible for calcium production and can be damaged during a
thyroidectomy. The tingling can be due to low calcium levels. The crash cart would be needed in
respiratory distress but would not be the next action to take; thus, answer A is incorrect.
Hypertension occurs in thyroid storm and the drainage would occur in hemorrhage, so answers
C and D are incorrect.
5. The client has an order for sliding scale insulin at 1900 hours and Lantus insulin at the
same hour. The nurse should:
A. Administer the two medications together.
B. Administer the medications in two injections.
C. Draw up the Lantus insulin and then the regular insulin and administer them
together.
D. Contact the doctor because these medications should not be given to the same
client
CORRECT ANSWER: B.
Lantus insulin cannot be mixed with other insulins, but can be taken by the client taking regular
insulin. A, C, and D are not correct methods of administering Lantus insulin with regular insulin.
6. The physician has prescribed NPH insulin for a client with diabetes mellitus. Which
statement indicates that the client knows when the peak action of the insulin occurs?
A. I will make sure I eat breakfast within two hours of taking my insulin.
B. I will need to carry candy or some form of sugar with me all the time.
C. I will eat a snack around three oclock each afternoon.
D. I can save my dessert from supper for a bedtime snack.
CORRECT ANSWER: C.
NPH insulin peaks in 812 hours, so a snack should be offered at that time. NPH insulin onsets
in 90120 minutes, so answer A is incorrect. Answer B is untrue because NPH insulin is time
released and does not usually cause sudden hypoglycemia. Answer D is incorrect, but the client
should eat a bedtime snack.
7. The physician has ordered a thyroid scan to confirm the diagnosis of a goiter. Before the
procedure, the nurse should:
A. Assess the client for allergies.
B. Bolus the client with IV fluid.
C. Tell the client he will be asleep.
D. Insert a urinary catheter
CORRECT ANSWER: A.
A thyroid scan uses a dye, so the client should be assessed for allergies to iodine. The client will
not have a bolus of fluid, will not be asleep, and will not have a urinary catheter inserted, so
answers B, C, and D are incorrect
8. The client has recently returned from having a thyroidectomy. The nurse should keep

which of the following at the bedside?


A. A tracheotomy set
B. A padded tongue blade
C. An endotracheal tube
D. An airway
CORRECT ANSWER: A.
The client who has recently had a thyroidectomy is at risk for tracheal edema. A padded tongue
blade is used for seizures and not for the client with tracheal edema, so answer B is incorrect. If
the client experiences tracheal edema, the endotracheal tube or airway will not correct the
problem, so answers C and D are incorrect.
9. A client received six units of regular insulin three hours ago. The nurse would be MOST
concerned if which of the following was observed?
A. Kussmaul respirations and diaphoresis.
B. Anorexia and lethargy.
C. Diaphoresis and trembling.
D. Headache and polyuria.
CORRECT ANSWER: C.
Kussmaul respirations are signs of hyperglycemia. Option B is not indicative of hypoglycaemia.
Option C is correctregular insulin peaks in two to four hours; it indicates hypoglycemia; give
skim milk. Option D is not indicative of hypoglycemia
10. The home health care nurse is caring for a 30-year-old woman with type I diabetes
mellitus. The client has been maintained on a regimen of NPH and regular insulin and a
1,800-calorie diabetic diet with normal blood sugar levels. Morning self-monitoring
blood sugar (SMBG) readings the past two days were 205 mg/dL and 233 mg/dL. The
nurse expects the physician to
A. Reduce the clients diet to 1,500 calorie ADA.
B. Order 3 additional units of NPH insulin at 10 PM.
C. Order an additional 10 units of regular insulin at 8 PM.
D. Eliminate the clients bedtime snack.
CORRECT ANSWER: B.
Strategy: All answers are implementations. Determine the outcome of each answer choice. Is it
desired? (A)diet should not be reduced (B) correctdawn phenomena, treatment is to adjust
evening diet, bedtime snack, insulin dose, and exercise to prevent early morning hyperglycemia
(C) peaks in 46 hours, would not prevent dawn phenomena (D) would adjust snack, not
eliminate it
11. The nurse should explain to a client that tolbutamide (Orinase) is effective for diabetics
who
A. Can no longer produce any insulin.
B. Produce minimal amounts of insulin.
C. Are unable to administer their injections.
D. Have sustained decreased blood glucose.

CORRECT ANSWER: B.
Option A, Type I insulin-dependent diabetic is unable to produce insulin. Option B is correct
oral hypoglycemic agents are administered to type II (non-insulindependent) clients who are
able to produce minimal amounts of insulin. Option C, type I diabetics who cannot administer
their injections need alternate plans to be made for them to receive the injection from a family
member. In option D, Orinase would be administered for an increase in blood glucose
12. A 38-year-old woman is returned to her room after a subtotal thyroidectomy for treatment
of hyperthyroidism. Which of the following, if found by the nurse at the patients bedside,
is nonessential?
A. Potassium chloride for IV administration.
B. Calcium gluconate for IV administration.
C. Tracheostomy set-up.
D. Suction equipment.
CORRECT ANSWER: A.
In option A, hypokalemia is not expected after this surgery. Calcium gluconate is used to treat
tetany resulting from possible damage to parathyroid glands. Tracheostomy set-up is an
essential equipment to provide for airway. Suction is needed to maintain a patent airway.
13. The nurse knows that the client with drug-induced Cushings syndrome should FIRST be
instructed about
A. Compression fractures from increased calcium excretion.
B. Decreased resistance to stress.
C. The schedule for gradual withdrawal of the drug.
D. Changes in secondary sex characteristics.
CORRECT ANSWER:C.
Option A and B are problems associated with Cushings syndrome, but are not the first priority.
In option C, if steroids are withdrawn suddenly, the client may die of acute adrenal insufficiency.
Option D is not seen with this medication
14.A patient with newly diagnosed diabetes mellitus is learning to recognize the
symptoms of hypoglycemia. Which of the following symptoms is indicative of
hypoglycemia?

A. Polydipsia.
B. Confusion.
C. Blurred vision.
D. Polyphagia.
CORRECT ANSWER: B
Hypoglycemia in diabetes mellitus causes confusion, indicating the need for carbohydrates.
Polydipsia, blurred vision, and polyphagia are symptoms of hyperglycemia.
15.A patient is admitted to the hospital with a diagnosis of primary

hyperparathyroidism. A nurse checking the patient's lab results would expect


which of the following changes in laboratory findings?

A. Elevated serum calcium.


B. Low serum parathyroid hormone (PTH).
C. Elevated serum vitamin D.
D. Low urine calcium.
CORRECT ANSWER: A.
The parathyroid glands regulate the calcium level in the blood. In hyperparathyroidism, the
serum calcium level will be elevated. Parathyroid hormone levels may be high or normal but not
low. The body will lower the level of vitamin D in an attempt to lower calcium. Urine calcium
may be elevated, with calcium spilling over from elevated serum levels. This may cause renal
stones.
16. A patient with Addison's disease asks a nurse for nutrition and diet advice. Which of the
following diet modifications is NOT recommended?
A. A diet high in grains
B. A diet with adequate caloric intake
C. A high protein diet
D. A restricted sodium diet
CORRECT ANSWER: D.
A patient with Addison's disease requires normal dietary sodium to prevent excess fluid loss.
Adequate caloric intake is recommended with a diet high in protein and complex carbohydrates,
including grains.
17. A patient with a history of diabetes mellitus is in the second post-operative
day following cholecystectomy. She has complained of nausea and isn't able
to eat solid foods. The nurse enters the room to find the patient confused and
shaky. Which of the following is the most likely explanation for the patient's
symptoms?

A. Anesthesia reaction
B. Hyperglycemia
C. Hypoglycemia

D. Diabetic ketoacidosis
CORRECT ANSWER: C.
A post-operative diabetic patient who is unable to eat is likely to be suffering from
hypoglycemia. Confusion and shakiness are common symptoms. An anesthesia reaction would
not occur on the second post-operative day. Hyperglycemia and ketoacidosis do not cause
confusion and shakiness.
18. Nurse Ronn is assessing a client with possible Cushings syndrome. In a client with
Cushings syndrome, the nurse would expect to find:
A. Hypotension.
B. Thick, coarse skin.
C. Deposits of adipose tissue in the trunk and dorsocervical area.
D. Weight gain in arms and legs.
CORRECT ANSWER: C.
Because of changes in fat distribution, adipose tissue accumulates in the trunk, face (moonface),
and dorsocervical areas (buffalo hump). Hypertension is caused by fluid retention. Skin becomes
thin and bruises easily because of a loss of collagen. Muscle wasting causes muscle atrophy and
thin extremities.
19. A male client with primary diabetes insipidus is ready for discharge on desmopressin
(DDAVP). Which instruction should nurse Lina provide?
A. Administer desmopressin while the suspension is cold.
B. Your condition isnt chronic, so you wont need to wear a medical identification
bracelet.
C. You may not be able to use desmopressin nasally if you have nasal discharge or
blockage.
D. You wont need to monitor your fluid intake and output after you start taking
desmopressin.
CORRECT ANSWER: C.
Desmopressin may not be absorbed if the intranasal route is compromised. Although diabetes
insipidus is treatable, the client should wear medical identification and carry medication at all
times to alert medical personnel in an emergency and ensure proper treatment. The client must
continue to monitor fluid intake and output and receive adequate fluid replacement.
20. In a 29-year-old female client who is being successfully treated for Cushings syndrome,
nurse Lyzette would expect a decline in:
A. Serum glucose level.
B. Hair loss.
C. Bone mineralization.
D. Menstrual flow.
CORRECT ANSWER: A.
Hyperglycemia, which develops from glucocorticoid excess, is a manifestation of Cushings
syndrome. With successful treatment of the disorder, serum glucose levels decline. Hirsutism is

common in Cushings syndrome; therefore, with successful treatment, abnormal hair growth also
declines. Osteoporosis occurs in Cushings syndrome; therefore, with successful treatment, bone
mineralization increases. Amenorrhea develops in Cushings syndrome. With successful
treatment, the client experiences a return of menstrual flow, not a decline in it.
21. A male client has recently undergone surgical removal of a pituitary tumor. Dr. Wong
prescribes corticotropin (Acthar), 20 units I.M. q.i.d. as a replacement therapy. What is
the mechanism of action of corticotropin?
A. It decreases cyclic adenosine monophosphate (cAMP) production and affects the
metabolic rate of target organs.
B. It interacts with plasma membrane receptors to inhibit enzymatic actions.
C. It interacts with plasma membrane receptors to produce enzymatic actions that
affect protein, fat, and carbohydrate metabolism.
D. It regulates the threshold for water resorption in the kidneys.
CORRECT ANSWER: C.
Corticotropin interacts with plasma membrane receptors to produce enzymatic actions that
affect protein, fat, and carbohydrate metabolism. It doesnt decrease cAMP production. The
posterior pituitary hormone, antidiuretic hormone, regulates the threshold for water resorption
in the kidneys.
22. Capillary glucose monitoring is being performed every 4 hours for a female client
diagnosed with diabetic ketoacidosis. Insulin is administered using a scale of regular
insulin according to glucose results. At 2 p.m., the client has a capillary glucose level of
250 mg/dl for which he receives 8 U of regular insulin. Nurse Vince should expect the
doses:
A. Onset to be at 2 p.m. and its peak to be at 3 p.m.
B. Onset to be at 2:15 p.m. and its peak to be at 3 p.m.
C. Onset to be at 2:30 p.m. and its peak to be at 4 p.m.
D. Onset to be at 4 p.m. and its peak to be at 6 p.m.
CORRECT ANSWER: C.
Regular insulin, which is a short-acting insulin, has an onset of 15 to 30 minutes and a peak of 2
to 4 hours. Because the nurse gave the insulin at 2 p.m., the expected onset would be from 2:15
p.m. to 2:30 p.m. and the peak from 4 p.m. to 6 p.m.
23. A female client with Cushings syndrome is admitted to the medical-surgical unit. During
the admission assessment, nurse Jazz notes that the client is agitated and irritable, has
poor memory, reports loss of appetite, and appears disheveled. These findings are
consistent with which problem?
A. Depression
B. Neuropathy
C. Hypoglycemia
D. Hyperthyroidism
CORRECT ANSWER: A.

Agitation, irritability, poor memory, loss of appetite, and neglect of ones appearance may signal
depression, which is common in clients with Cushings syndrome. Neuropathy affects clients with
diabetes mellitus not Cushings syndrome. Although hypoglycemia can cause irritability, it
also produces increased appetite, rather than loss of appetite. Hyperthyroidism typically causes
such signs as goiter, nervousness, heat intolerance, and weight loss despite increased appetite.
24. Nurse Ruth is assessing a client after a thyroidectomy. The assessment reveals muscle
twitching and tingling, along with numbness in the fingers, toes, and mouth area. The
nurse should suspect which complication?
A. Tetany
B. Hemorrhage
C. Thyroid storm
D. Laryngeal nerve damage
CORRECT ANSWER IS A.
Tetany may result if the parathyroid glands are excised or damaged during thyroid surgery.
Hemorrhage is a potential complication after thyroid surgery but is characterized by
tachycardia, hypotension, frequent swallowing, feeling of fullness at the incision site, choking,
and bleeding. Thyroid storm is another term for severe hyperthyroidism not a complication of
thyroidectomy. Laryngeal nerve damage may occur postoperatively, but its signs include a
hoarse voice and, possibly, acute airway obstruction.
25. After undergoing a subtotal thyroidectomy, a female client develops hypothyroidism. Dr.
Smith prescribes levothyroxine (Levothroid), 25 mcg P.O. daily. For which condition is
levothyroxine the preferred agent?
A. Primary hypothyroidism
B. Graves disease
C. Thyrotoxicosis
D. Euthyroidism
CORRECT ANSWER: A.
Levothyroxine is the preferred agent to treat primary hypothyroidism and cretinism, although it
also may be used to treat secondary hypothyroidism. It is contraindicated in Graves disease and
thyrotoxicosis because these conditions are forms of hyperthyroidism. Euthyroidism, a term used
to describe normal thyroid function, wouldnt require any thyroid preparation.

PART III
WOMEN & CHILD S
HEALTH

MATERNAL
NURSING

1. An 18-year-old woman comes to the physicians office for a routine prenatal checkup at
34 weeks gestation. Abdominal palpation reveals the fetal position as right occipital
anterior (ROA). At which of the following sites would the nurse expect to find the fetal
heart tone?
A. Below the umbilicus, on the mothers left side.
B. Below the umbilicus, on the mothers right side.
C. Above the umbilicus, on the mothers left side.
D. Above the umbilicus, on the mothers right side.
CORRECT ANSWER: B
Occiput and back are pressing against the right side of the mothers abdomen. Fetal heart tones
would be heard below umbilicus on right side. Option A should be on the right. Options C and D
are both found in breech position.
2. The nurse teaches a 20-year-old primigravida how to measure the frequency of uterine
contractions. The nurse should explain to the patient that the frequency of uterine
contractions is determined by:
A. From the beginning of one contraction to the end of the next contraction.
B. From the beginning of one contraction to the end of the same contraction.
C. By the strength of the contraction at its peak.
D. By the number of contractions that occur within a given period of time.
CORRECT ANSWER: D
This determines the frequency of contraction. Option A is not accurate. Opyion B defines
duration. Option C describes intensity.
3. When the nurse checks the fundus of a client on the first postpartum day, she notes that
the fundus is firm, is at the level of the umbilicus, and is displaced to the right. The next
action the nurse should take is to:
A. Check the client for bladder distention.
B. Assess the blood pressure for hypotension.
C. Determine whether an oxytocic drug was given.
D. Check for the expulsion of small clots.
CORRECT ANSWER: A
If the fundus of the client is displaced to the side, this might indicate a full bladder. The next
action by the nurse should be to check for bladder distention and catheterize, if necessary. The
answers in B, C, and D are actions that relate to postpartal haemorrhage.
4. A client is admitted to the labor and delivery unit in active labor. During examination, the
nurse notes a papular lesion on the perineum. Which initial action is most appropriate?
A. Document the finding.
B. Report the finding to the doctor.
C. Prepare the client for a C-section.
D. Continue primary care as prescribed

CORRECT ANSWER: B.
Any lesion should be reported to the doctor. This can indicate a herpes lesion. Clients with open
lesions related to herpes are delivered by Cesarean section because there is a possibility of
transmission of the infection to the fetus with direct contact to lesions. It is not enough to
document the finding, so answer A is incorrect. The physician must make the decision to perform
a C-section, making answer C incorrect. It is not enough to continue primary care, so answer D
is incorrect.
5. A 15-year-old primigravida is admitted with a tentative diagnosis of HELLP syndrome.
Which laboratory finding is associated with HELLP syndrome?
A. Elevated blood glucose
B. Elevated platelet count
C. Elevated creatinine clearance
D. Elevated hepatic enzymes
CORRECT ANSWER: D.
The criteria for HELLP is hemolysis, elevated liver enzymes, and low platelet count. In answer
A, an elevated blood glucose level is not associated with HELLP. Platelets are decreased, not
elevated, in HELLP syndrome, as stated in answer B. The creatinine levels are elevated in renal
disease and are not associated with HELLP syndrome, so answer C is incorrect.
6. The nurse is assessing the deep tendon reflexes of a client with preeclampsia. Which
method is used to elicit the biceps reflex?
A. The nurse places her thumb on the muscle inset in the antecubital space and taps
the thumb briskly with the reflex hammer.
B. The nurse loosely suspends the clients arm in an open hand while tapping the
back of the clients elbow.
C. The nurse instructs the client to dangle her legs as the nurse strikes the area below
the patella with the blunt side of the reflex hammer.
D. The nurse instructs the client to place her arms loosely at her side as the nurse
strikes the muscle insert just above the wrist.
CORRECT ANSWER: A.
Answer B elicits the triceps reflex, so it is incorrect. Answer C elicits the patella reflex, making it
incorrect. Answer D elicits the radial nerve, so it is incorrect.
7. A primigravida with diabetes is admitted to the labor and delivery unit at 34 weeks
gestation. Which doctors order should the nurse question?
A. Magnesium sulfate 4gm (25%) IV
B. Brethine 10mcg IV
C. Stadol 1mg IV push every 4 hours as needed prn for pain
D. Ancef 2gm IVPB every 6 hours
CORRECT ANSWER: B.
Brethine is used cautiously because it raises the blood glucose levels. Answers A, C, and D are
all medications that are commonly used in the diabetic client, so they are incorrect.

8. Which observation in the newborn of a diabetic mother would require immediate nursing
intervention?
A. Crying
B. Wakefulness
C. Jitteriness
D. Yawning
CORRECT ANSWER: C.
Answer C is correct. Jitteriness is a sign of seizure in the neonate. Crying, wakefulness, and
yawning are expected in the newborn, so answers A, B, and D are incorrect.
9. The nurse caring for a client receiving intravenous magnesium sulfate must closely
observe for side effects associated with drug therapy. An expected side effect of
magnesium sulfate is:
A. Decreased urinary output
B. Hypersomnolence
C. Absence of knee jerk reflex
D. Decreased respiratory rate
CORRECT ANSWER: B.
The client is expected to become sleepy, have hot flashes, and be lethargic. A decreasing urinary
output, absence of the knee-jerk reflex, and decreased respirations indicate toxicity, so answers
A, C, and D are incorrect.
10. Which selection would provide the most calcium for the client who is four months
pregnant?
A. A granola bar
B. A bran muffin
C. A cup of yogurt
D. A glass of fruit juice
CORRECT ANSWER: C.
The food with the most calcium is the yogurt. Answers A, B, and D are good choices, but not as
good as the yogurt, which has approximately 400mg of calcium.
11. The obstetric clients fetal heart rate is 8090 during the contractions. The first action the
nurse should take is:
A. Reposition the monitor.
B. Turn the client to her left side.
C. Ask the client to ambulate.
D. Prepare the client for delivery.
CORRECT ANSWER: B.
The normal fetal heart rate is 120160bpm; 100110bpm is bradycardia. The first action would
be to turn the client to the left side and apply oxygen. Answer A is not indicated at this time.

Answer C is not the best action for clients experiencing bradycardia. There is no data to indicate
the need to move the client to the delivery room at this time.
12. A vaginal exam reveals that the cervix is 4cm dilated, with intact membranes and a fetal
heart tone rate of 160170bpm. The nurse decides to apply an external fetal monitor. The
rationale for this implementation is:
A. The cervix is closed.
B. The membranes are still intact.
C. The fetal heart tones are within normal limits.
D. The contractions are intense enough for insertion of an internal monitor.
CORRECT ANSWER: B.
The nurse decides to apply an external monitor because the membranes are intact. Answers A,
C, and D are incorrect. The cervix is dilated enough to use an internal monitor, if necessary. An
internal monitor can be applied if the client is at 0-station. Contraction intensity has no bearing
on the application of the fetal monitor.
13. The following are all nursing diagnoses appropriate for a gravida 1 para 0 in labor. Which
one would be most appropriate for the primagravida as she completes the early phase of
labor?
A. Impaired gas exchange related to hyperventilation
B. Alteration in placental perfusion related to maternal position
C. Impaired physical mobility related to fetal-monitoring equipment
D. Potential fluid volume deficit related to decreased fluid intake
CORRECT ANSWER: D.
Clients admitted in labor are told not to eat during labor, to avoid nausea and vomiting. Ice
chips may be allowed, but this amount of fluid might not be sufficient to prevent fluid volume
deficit. In answer A, impaired gas exchange related to hyperventilation would be indicated
during the transition phase. Answers B and C are not correct in relation to the stem.
14. As the client reaches 6cm dilation, the nurse notes late decelerations on the fetal monitor.
What is the most likely explanation of this pattern?
A. The baby is sleeping.
B. The umbilical cord is compressed.
C. There is head compression.
D. There is uteroplacental insufficiency.
CORRECT ANSWER: D.
This information indicates a late deceleration. This type of deceleration is caused by
uteroplacental lack of oxygen. Answer A has no relation to the readings, so its incorrect; answer
B results in a variable deceleration; and answer C is indicative of an early deceleration.
15. The nurse notes variable decelerations on the fetal monitor strip. The most appropriate
initial action would be to:
A. Notify her doctor.

B. Start an IV.
C. Reposition the client.
D. Readjust the monitor.
CORRECT ANSWER: C.
The initial action by the nurse observing a late deceleration should be to turn the client to the
sidepreferably, the left side. Administering oxygen is also indicated. Answer A might be
necessary but not before turning the client to her side. Answer B is not necessary at this time.
Answer D is incorrect because there is no data to indicate that the monitor has been applied
incorrectly.
16. Which of the following is a characteristic of an ominous periodic change in the fetal heart
rate?
A. A fetal heart rate of 120130bpm
B. A baseline variability of 610bpm
C. Accelerations in FHR with fetal movement
D. A recurrent rate of 90100bpm at the end of the contractions
CORRECT ANSWER: D.
A deceleration to 90100bpm at the end of contractions are late decelerations. This finding is
ominous (bad) and should be reported. A, B, and D are normal findings and are therefore
incorrect.
17. A client in the family planning clinic asks the nurse about the most likely time for her to
conceive. The nurse explains that conception is most likely to occur when:
A. Estrogen levels are low
B. Lutenizing hormone is high
C. The endometrial lining is thin
D. The progesterone level is low
CORRECT ANSWER: B.
Lutenizing hormone released by the pituitary is responsible for ovulation. At about day 14, the
continued increase in estrogen stimulates the release of lutenizing hormone from the anterior
pituitary. The LH surge is responsible for ovulation, or the release of the dominant follicle in
preparation for conception, which occurs within the next 1012 hours after the LH levels peak.
Answers A, C, and D are incorrect because estrogen levels are high at the beginning of
ovulation, the endometrial lining is thick, not thin, and the progesterone levels are high, not low.
18. A client with diabetes asks the nurse for advice regarding methods of birth control. Which
method of birth control is most suitable for the client with diabetes?
A. Intrauterine device
B. Oral contraceptives
C. Diaphragm
D. Contraceptive sponge
CORRECT ANSWER: C.

The best method of birth control for the client with diabetes is the diaphragm. A permanent
intrauterine device can cause a continuing inflammatory response in diabetics that should be
avoided, oral contraceptives tend to elevate blood glucose levels, and contraceptive sponges are
not good at preventing pregnancy. Therefore, answers A, B, and D are incorrect.
19. The doctor suspects that the client has an ectopic pregnancy. Which symptom is
consistent with a diagnosis of a ruptured ectopic pregnancy?
A. Painless vaginal bleeding
B. Abdominal cramping
C. Throbbing pain in the upper quadrant
D. Sudden, stabbing pain in the lower quadrant
CORRECT ANSWER: D.
The signs of an ectopic pregnancy are vague until the fallopian tube ruptures. The client will
complain of sudden, stabbing pain in the lower quadrant that radiates down the leg or up into
the chest. Painless vaginal bleeding is a sign of placenta previa, abdominal cramping is a sign of
labor, and throbbing pain in the upper quadrant is not a sign of an ectopic pregnancy, making
answers A, B, and C incorrect.
20. The nurse is teaching a pregnant client about nutritional needs during pregnancy. Which
menu selection will best meet the nutritional needs of the pregnant client?
A. Hamburger patty, green beans, French fries, and iced tea
B. Roast beef sandwich, potato chips, baked beans, and cola
C. Baked chicken, fruit cup, potato salad, coleslaw, yogurt, and iced tea
D. Fish sandwich, gelatin with fruit, and coffee
CORRECTANSWER: C.
All of the choices are tasty, but the pregnant client needs a diet that is balanced and has
increased amounts of calcium. Answer A is lacking in fruits and milk. Answer B contains the
potato chips, which contain a large amount of sodium. Answer C contains meat, fruit, potato
salad, and yogurt, which has about 360mg of calcium. Answer D is not the best diet because it
lacks vegetables and milk products.
21. The client with hyperemesis gravidarum is at risk for developing:
A. Respiratory alkalosis without dehydration
B. Metabolic acidosis with dehydration
C. Respiratory acidosis without dehydration
D. Metabolic alkalosis with dehydration
CORRECT ANSWER: B.
The client with hyperemesis has persistent nausea and vomiting. With vomiting comes
dehydration. When the client is dehydrated, she will have metabolic acidosis. Answers A and C
are incorrect because they are respiratory dehydration. Answer D is incorrect because the client
will not be in alkalosis with persistent vomiting.
22. A client tells the doctor that she is about 20 weeks pregnant. The most definitive sign of
pregnancy is:

A.
B.
C.
D.

Elevated human chorionic gonadatropin


The presence of fetal heart tones
Uterine enlargement
Breast enlargement and tenderness

CORRECT ANSWER: B.
The most definitive diagnosis of pregnancy is the presence of fetal heart tones. The signs in
answers A, C, and D are subjective and might be related to other medical conditions. Answers A
and C may be related to a hydatidiform mole, and answer D is often present before menses or
with the use of oral contraceptives
23. A client is admitted to the labor and delivery unit complaining of vaginal bleeding with
very little discomfort. The nurses first action should be to:
A. Assess the fetal heart tones.
B. Check for cervical dilation.
C. Check for firmness of the uterus.
D. Obtain a detailed history
CORRECT ANSWER: A.
The symptoms of painless vaginal bleeding are consistent with placenta previa. Answers B, C,
and D are incorrect. Cervical check for dilation is contraindicated because this can increase the
bleeding. Checking for firmness of the uterus can be done, but the first action should be to check
the fetal heart tones. A detailed history can be done later
24. The physician has ordered an injection of RhoGam for the postpartum client whose blood
type is A negative but whose baby is O positive. To provide postpartum prophylaxis,
RhoGam should be administered:
A. Within 72 hours of delivery
B. Within one week of delivery
C. Within two weeks of delivery
D. Within one month of delivery
CORRECT ANSWER: A.
To provide protection against antibody production, RhoGam should be given within 72 hours.
The answers in B, C, and D are too late to provide antibody protection. RhoGam can also be
given during pregnancy.
25. After the physician performs an amniotomy, the nurses first action should be to assess
the:
A. Degree of cervical dilation
B. Fetal heart tones
C. Clients vital signs
D. Clients level of discomfort
CORRECT ANSWER: B.
When the membranes rupture, there is often a transient drop in the fetal heart tones. The heart

tones should return to baseline quickly. Any alteration in fetal heart tones, such as bradycardia
or tachycardia, should be reported. After the fetal heart tones are assessed, the nurse should
evaluate the cervical dilation, vital signs, and level of discomfort, making answers A, C, and D
incorrect.

PEDIATRIC
NURSING

1. The home health nurse is visiting an 18-year-old with osteogenesis imperfecta. Which
information obtained on the visit would cause the most concern? The client:
A. Likes to play football
B. Drinks carbonated drinks
C. Has two sisters
D. Is taking acetaminophen for pain
CORRECT ANSWER: A.
The client with osteogenesis imperfecta is at risk for pathological fractures and is likely to
experience these fractures if he participates in contact sports. Answers B, C, and D are not
factors for concern.
2. Which of the following foods, if selected by the mother with a child with celiac, would
indicate her understanding of the dietary instructions?
A. Whole-wheat toast
B. Angel hair pasta
C. Reuben on rye
D. Rice cereal
CORRECT ANSWER: D.
The child with celiac disease should be on a gluten-free diet. Answers A, B, and C all contain
gluten, while answer D gives the only choice of foods that does not contain gluten.
3. A full-term male has hypospadias. Which statement describes hypospadias?
A. The urethral opening is absent
B. The urethra opens on the top side of the penis
C. The urethral opening is enlarged
D. The urethra opens on the underside of the penis
CORRECT ANSWER: D.
Hypospadias is a condition in which there is an opening on the under side of the penis. Answers
A, B, and C do not describe hypospadias; therefore, they are incorrect.
4. The nurse is performing an initial assessment of a newborn Caucasian male delivered at
32 weeks gestation. The nurse can expect to find the presence of:
A. Mongolian spots
B. Scrotal rugae
C. Head lag
D. Polyhydramnios
CORRECT ANSWER: C.
The infant who is 32 weeks gestation will not be able to control his head, so head lag will be
present. Mongolian spots are common in African American infants, not Caucasian infants; the
client at 32 weeks will have scrotal rugae or redness. There is no alteration in the amount of
amniotic fluid; therefore, answers A, B, and D are incorrect

5. The toddler is admitted with a cardiac anomaly. The nurse is aware that the infant with a
ventricular septal defect will:
A. Tire easily
B. Grow normally
C. Need more calories
D. Be more susceptible to viral infections
CORRECT ANSWER: A.
The toddler with a ventricular septal defect will tire easily. He will not grow normally but will
not need more calories. He will be susceptible to bacterial infection, but he will be no more
susceptible to viral infections than other children. Therefore, answers B, C, and D are incorrect.
6. The nurse in the well-baby clinic observes a group of children. The nurse notes that one
child is able to sit unsupported, play peek-a-boo with the nurse and is starting to say
mama and dada. The nurse would expect these behaviors in a child that is
A. 5 months of age.
B. 6 months of age.
C. 9 months of age.
D. 1 year of age.
CORRECT ANSWER: C.
In Option A and B, unable to sit unsupported until 8 months. Option C is correctcan pull self up
and assume a sitting position at 8 months, can say few words. In option D, 1 year of age would
be able to say 35 words in addition to dada and mama
7. A four-month-old infant is admitted to the pediatric intensive care unit with a temperature
of 105F (40.5 C). The infant is irritable, and the nurse observes nuchal rigidity. Which
assessment finding would indicate an increase in intracranial pressure?
A. Positive Babinski.
B. High-pitched cry.
C. Bulging posterior fontanelle.
D. Pinpoint pupils.
CORRECT ANSWER: B.
Positive Babinski is normal for the first year of life. High-pitched cry is one of the first signs of
an increase in the intracranial pressure in infants. Posterior fontanelle should be closed by the
third month. With increased pressure, the pupil may respond to light slowly, rather than with the
usual brisk response
8. A child is admitted to the hospital with an uncontrolled seizure disorder. The admitting
physician writes orders for actions to be taken in the event of a seizure. Which of the
following actions would NOT be included?
A. Notify the physician.
B. Restrain the patient's limbs.

C. Position the patient on his/her side with the head flexed forward.
D. Administer rectal diazepam.

CORRECT ANSWER: B.
During a witnessed seizure, nursing actions should focus on securing the patient's
safely and curtailing the seizure. Restraining the limbs is not indicated because
strong muscle contractions could cause injury. A side-lying position with head flexed
forward allows for drainage of secretions and prevents the tongue from falling back,
blocking the airway. Rectal diazepam may be a treatment ordered by the physician,
who should be notified of the seizure.

9. A mother complains to the clinic nurse that her 2 -year-old son is not yet
toilet trained. She is particularly concerned that, although he reliably uses the
potty seat for bowel movements, he isn't able to hold his urine for long
periods. Which of the following statements by the nurse is correct?
A. The child should have been trained by age 2 and may have a
psychological problem that is responsible for his "accidents."
B. Bladder control is usually achieved before bowel control, and the child
should be required to sit on the potty seat until he passes urine.
C. Bowel control is usually achieved before bladder control, and the
average age for completion of toilet training varies widely from 24 to
36 months.
D. The child should be told "no" each time he wets so that he learns the
behavior is unacceptable.

CORRECT ANSWER: C.
Toddlers typically learn bowel control before bladder control, with boys often taking
longer to complete toilet training than girls. Many children are not trained until 36
months and this should not cause concern. Later training is rarely caused by
psychological factors and is much more commonly related to individual
developmental maturity. Reprimanding the child will not speed the process and may
be confusing.

10.Which of the following actions is NOT appropriate in the care of a 2-month-old


infant?
A. Place the infant on her back for naps and bedtime.
B. Allow the infant to cry for 5 minutes before responding if she wakes
during the night as she may fall back asleep.
C. Talk to the infant frequently and make eye contact to encourage
language development.
D. Wait until at least 4 months to add infant cereals and strained fruits to
the diet.

CORRECT ANSWER: B.
Infants under 6 months may not be able to sleep for long periods because their
stomachs are too small to hold adequate nourishment to take them through the
night. After 6 months, it may be helpful to let babies put themselves back to sleep
after waking during the night, but not prior to 6 months. Infants should always be
placed on their backs to sleep. Research has shown a dramatic decrease in sudden
infant death syndrome (SIDS) with back sleeping. Eye contact and verbal
engagement with infants are important to language development. The best diet for
infants under 4 months of age is breast milk or infant formula.

11.A child is admitted to the hospital with suspected rheumatic fever. Which of
the following observations is NOT confirming of the diagnosis?
A. A reddened rash visible over the trunk and extremities.
B. A history of sore throat that was self-limited in the past month.
C. A negative antistreptolysin O titer.
D. An unexplained fever.

CORRECT ANSWER: C.

Rheumatic fever is caused by an untreated group A B hemolytic Streptococcus


infection in the previous 2-6 weeks, confirmed by a positive antistreptolysin O titer.
Rheumatic fever is characterized by a red rash over the trunk and extremities as
well as fever and other symptoms.

12.An infant with congestive heart failure is receiving diuretic therapy at home.
Which of the following symptoms would indicate that the dosage may need to
be increased?
A. Sudden weight gain.
B. Decreased blood pressure.
C. Slow, shallow breathing.
D. Bradycardia.

CORRECT ANSWER: A.
Weight gain is an early symptom of congestive heart failure due to accumulation of
fluid. When diuretic therapy is inadequate, one would expect an increase in blood
pressure, tachypnea, and tachycardia to result.

13.A nurse in the emergency department is observing a 4-year-old child for signs
of increased intracranial pressure after a fall from a bicycle, resulting in head
trauma. Which of the following signs or symptoms would be cause for
concern?
A. Bulging anterior fontanel.
B. Repeated vomiting.
C. Signs of sleepiness at 10 PM.
D. Inability to read short words from a distance of 18 inches.

CORRECT ANSWER: B.

Increased pressure caused by bleeding or swelling within the skull can damage
delicate brain tissue and may become life threatening. Repeated vomiting can be
an early sign of pressure as the vomit center within the medulla is stimulated. The
anterior fontanel is closed in a 4-year-old child. Evidence of sleepiness at 10 PM is
normal for a four year old. The average 4-year-old child cannot read yet, so this too
is normal.

14.The mother of a 2-month-old infant brings the child to the clinic for a well
baby check. She is concerned because she feels only one testis in the scrotal
sac. Which of the following statements about the undescended testis is the
most accurate?
A. Normally, the testes are descended by birth.
B. The infant will likely require surgical intervention.
C. The infant probably has with only one testis.
D. Normally, the testes descend by one year of age.

CORRECT ANSWER: D.
Normally, the testes descend by one year of age. In young infants, it is common for
the testes to retract into the inguinal canal when the environment is cold or the
cremasteric reflex is stimulated. Exam should be done in a warm room with warm
hands. It is most likely that both testes are present and will descend by a year. If
not, a full assessment will determine the appropriate treatment.

15.A child is admitted to the hospital with a diagnosis of Wilm's tumor, stage II.
Which of the following statements most accurately describes this stage?
A. The tumor is less than 3 cm. in size and requires no chemotherapy.

B. The tumor did not extend beyond the kidney and was completely resected.
C. The tumor extended beyond the kidney but was completely resected.
D. The tumor has spread into the abdominal cavity and cannot be resected

CORRECT ANSWER: C.
The staging of Wilm's tumor is confirmed at surgery as follows: Stage I, the tumor is
limited to the kidney and completely resected; stage II, the tumor extends beyond
the kidney but is completely resected; stage III, residual nonhematogenous tumor is
confined to the abdomen; stage IV, hematogenous metastasis has occurred with
spread beyond the abdomen; and stage V, bilateral renal involvement is present at
diagnosis.

16. An infant with hydrocele is seen in the clinic for a follow-up visit at 1 month of age. The
scrotum is smaller than it was at birth, but fluid is still visible on illumination. Which of
the following actions is the physician likely to recommend?
A. Massaging the groin area twice a day until the fluid is gone.
B. Referral to a surgeon for repair.
C. No treatment is necessary; the fluid is reabsorbing normally.
D. Keeping the infant in a flat, supine position until the fluid is gone.

CORRECT ANSWER: C.
A hydrocele is a collection of fluid in the scrotum that results from a patent tunica
vaginalis. Illumination of the scrotum with a pocket light demonstrates the clear
fluid. In most cases the fluid reabsorbs within the first few months of life and no
treatment is necessary. Massaging the area or placing the infant in a supine
position would have no effect. Surgery is not indicated.

17. An infant is brought to the clinic by his mother, who has noticed that he holds his head in
an unusual position and always faces to one side. Which of the following is the most
likely explanation?
A. Torticollis, with shortening of the sternocleidomastoid muscle.
B. Craniosynostosis, with premature closure of the cranial sutures.
C. Plagiocephaly, with flattening of one side of the head.
D. Hydrocephalus, with increased head size.

CORRECT ANSWER: A.
In torticollis, the sternocleidomastoid muscle is contracted, limiting range of motion
of the neck and causing the chin to point to the opposing side. In craniosynostosis
one of the cranial sutures, often the sagittal, closes prematurely, causing the head
to grow in an abnormal shape. Plagiocephaly refers to the flattening of one side of

the head, caused by the infant being placed supine in the same position over time.
Hydrocephalus is caused by a build-up of cerebrospinal fluid in the brain resulting in
large head size.

18. An adolescent brings a physician's note to school stating that he is not to participate in
sports due to a diagnosis of Osgood-Schlatter disease. Which of the following statements
about the disease is correct?
A. The condition was caused by the student's competitive swimming schedule.
B. The student will most likely require surgical intervention.
C. The student experiences pain in the inferior aspect of the knee.
D. The student is trying to avoid participation in physical education.

CORRECT ANSWER: C.
Osgood-Schlatter disease occurs in adolescents in rapid growth phase when the
infrapatellar ligament of the quadriceps muscle pulls on the tibial tubercle, causing
pain and swelling in the inferior aspect of the knee. Osgood-Schlatter disease is
commonly caused by activities that require repeated use of the quadriceps,
including track and soccer. Swimming is not a likely cause. The condition is usually
self-limited, responding to ice, rest, and analgesics. Continued participation will
worsen the condition and the symptoms.

19. A nurse is assigned to the pediatric rheumatology clinic and is assessing a child who has
just been diagnosed with juvenile idiopathic arthritis. Which of the following statements
about the disease is most accurate?
A. The child has a poor chance of recovery without joint deformity.
B. Most children progress to adult rheumatoid arthritis.
C. Nonsteroidal anti-inflammatory drugs are the first choice in treatment.
D. Physical activity should be minimized.
CORRECT ANSWER: C.
Nonsteroidal anti-inflammatory drugs are important first line treatment for juvenile idiopathic
arthritis (formerly known as juvenile rheumatoid arthritis). NSAIDs require 3-4 weeks for the
therapeutic anti-inflammatory effects to be realized. Half of children with the disorder recover
without joint deformity, and about a third will continue with symptoms into adulthood. Physical
activity is an integral part of therapy.
20. A child is admitted to the hospital several days after stepping on a sharp object that
punctured her athletic shoe and entered the flesh of her foot. The physician is concerned
about osteomyelitis and has ordered parenteral antibiotics. Which of the following actions
is done immediately before the antibiotic is started?

A.
B.
C.
D.

The admission orders are written.


A blood culture is drawn.
A complete blood count with differential is drawn.
The parents arrive.

CORRECT ANSWER: B.
Antibiotics must be started after the blood culture is drawn, as they may interfere
with the identification of the causative organism. The blood count will reveal the
presence of infection but does not help identify an organism or guide antibiotic
treatment. Parental presence is important for the adjustment of the child but not for
the administration of medication.

21.A two-year-old child has sustained an injury to the leg and refuses to walk.
The nurse in the emergency department documents swelling of the lower
affected leg. Which of the following does the nurse suspect is the cause of
the child's symptoms?

A.
B.
C.
D.

Possible fracture of the tibia.


Bruising of the gastrocnemius muscle.
Possible fracture of the radius.
No anatomic injury, the child wants his mother to carry him.

CORRECT ANSWER: A.
The child's refusal to walk, combined with swelling of the limb is suspicious for
fracture. Toddlers will often continue to walk on a muscle that is bruised or strained.
The radius is found in the lower arm and is not relevant to this question. Toddlers
rarely feign injury to be carried, and swelling indicates a physical injury.

22. A child has recently been diagnosed with Duchenne's muscular dystrophy. The parents
are receiving genetic counseling prior to planning another pregnancy. Which of the
following statements includes the most accurate information?
A. Duchenne's is an X-linked recessive disorder, so daughters have a 50% chance of
being carriers and sons a 50% chance of developing the disease
B. Duchenne's is an X-linked recessive disorder, so both daughters and sons have a
50% chance of developing the disease
C. Each child has a 1 in 4 (25%) chance of developing the disorder

D. Sons only have a 1 in 4 (25%) chance of developing the disorder

CORRECT ANSWER: A.
The recessive Duchenne's gene is located on one of the two X chromosomes of a
female carrier. If her son receives the X bearing the gene he will be affected. Thus,
there is a 50% chance of a son being affected. Daughters are not affected, but 50%
are carriers because they inherit one copy of the defective gene from the mother.
The other X chromosome comes from the father, who cannot be a carrier.

23. Which of the following blood study results would the nurse expect as most likely when
caring for the child with iron deficiency anemia?
A.
B.
C.
D.

Increased hemoglobin
Normal hematocrit
Decreased mean corpuscular volume (MCV)
Normal total iron-binding capacity (TIBC)

CORRECT ANSWER: C.
For the child with iron deficiency anemia, the blood study results most likely would reveal
decreased mean corpuscular volume (MCV) which demonstrates microcytic anemia, decreased
hemoglobin, decreased hematocrit and elevated total iron binding capacity.
24. The nurse is assessing a 9-month-old boy for a well-baby check up. Which of the
following observations would be of most concern?
A.
B.
C.
D.

The baby cannot say mama when he wants his mother.


The mother has not given him finger foods.
The child does not sit unsupported.
The baby cries whenever the mother goes out.

CORRECT ANSWER: C.
Over 90% percent of babies can sit unsupported by nine months. Most babies cannot say
mama in the sense that it refers to their mother at this time.
25. While examining a 2-year-old child, the nurse in charge sees that the anterior fontanel is
open. The nurse should:
A. Notify the doctor
B. Look for other signs of abuse
C. Recognize this as a normal finding
D. Ask about a family history of Tay-Sachs disease
CORRECT ANSWER: A.

Because the anterior fontanel normally closes between ages 12 and 18 months, the nurse should
notify the doctor promptly of this finding. An open fontanel does not indicate abuse and is not
associated with Tay-Sachs disease.

PART IV
SPECIAL NURSING
TOPICS

EMERGENCY & DISASTER


NURSING

1. Four clients arrive in the emergency room. Which client should receive priority?
A. The client with burns of face and neck
B. The client with gastroenteritis
C. The client with a fractured tibia
D. The client with a migraine headache
CORRECT ANSWER: A.
The client with burns of chest and neck should be seen first because he is at risk for airway
obstruction. Answers B, C, and D are incorrect because these clients do not take priority over
the client with potential airway obstruction.
2. The emergency room nurse is assigned to triage four clients. Which client should be seen
first?
A. The pregnant client with mild abdominal pain
B. The client with emphysema with an oxygen saturation level of 83%.
C. The client with chronic glomerulonephritis with loss of his AV Fistula
D. The client with diabetes with blood glucose of 277 mg/dl.
CORRECT ANSWER: B.
The oxygen saturation is extremely low even for a client with emphysema. Answer A is incorrect
because the clients symptom is mild abdominal pain, which is a vague complaint. Answer C is
incorrect because the client with chronic glomerulonephritis with loss of AV fistula does not
require immediate attention. Answer D is incorrect because a blood glucose level of 277 mg/dL
in the client with diabetes is not life-threatening, although it does require intervention. This
client should be seen next.
3. A client is admitted to the unit two hours after an explosion causing burns to the face. The
nurse would be most concerned to the client developing which of the following?
A. Hypovolemia
B. Laryngeal Edema
C. Hypernatremia
D. Hyperkalemia
CORRECT ANSWER: B.
The nurse should be most concerned with laryngeal edema because of the area of burn. The next
priority should be hypovolemia, as well as hyponatremia and hypokalemia, but these answers
are not of primary concern, so are incorrect.
4. A category four tornado has injured 50 people from the community. The nurse is
responsible for in field triage. According to triage protocol, which client should be treated
last?
A. The 30-year-old with lacerations to the neck and face
B. The 6-year-old with fixed, dilated pupils who is nonresponsive
C. The 70-year-old with chest pain and shortness of breath
D. The 40-year-old with tachypnea and tachycardia
CORRECT ANSWER: B
The six-year-old with fixed, dilated pupils who is unresponsive is unlikely to survive. According
to disaster triage protocol, priority is given to those clients who are expected to survive with
fewer resource expenditures. Answers A, C, and D are incorrect for this reason.
5. The emergency room is flooded with clients injured in a tornado. Which clients can be

assigned to share a room in the emergency department during the disaster?


A. A client having auditory hallucinations and the client with ulcerative colitis
B. The client who is pregnant and the client with a broken arm
C. A child who is cyanotic with severe dypsnea and a client with a frontal head
injury
D. The client who arrives with a large puncture wound to the abdomen and the client
with chest pain
CORRECT ANSWER: B
The pregnant client and the client with a broken arm are the best choices for placing in the same
room. The clients in answers A, C, and D need to be placed in separate rooms due to the serious
natures of their injuries.
6. A community health nurse is working with disaster relief following a flood. Finding safe
housing for survivors, providing support for families, organizing counseling, and
securing physical care are examples of which type of prevention?
A. Aggregate care prevention
B. Primary prevention
C. Secondary prevention
D. Tertiary prevention
CORRECT ANSWER: D
Tertiary prevention involves reducing the degree and quantity of injury, disability and damage
following a disaster or crisis. Aggregate prevention isnt a level of care prevention. Primary
prevention focuses on keeping the crisis or disaster from happening. The goal of secondary
prevention is to reduce the duration and intensity of the disaster or crisis.
7. The primary reason for rapid continuous rewarming of the area affected by frostbite is to:
A. Lessen the amount of cellular damage
B. Prevent the formation of blisters
C. Promote movement
D. Prevent pain and discomfort
CORRECT ANSWER: A
Rapid continuous rewarming of a frostbite primarily lessens cellular damage. It does not prevent
formation of blisters. It does promote movement, but this is not the primary reason for rapid
rewarming. It might increase pain for a short period of time as the feeling comes back into the
extremity; therefore, answers B, C, and D are incorrect.
8. A home care nurse finds a client in the bedroom, unconscious, with a pill bottle in hand.
The pill bottle had contained the selective serotonin reuptake inhibitor, sertraline (Zoloft).
The nurse should immediately assess which item?
A. Pulse
B. Respirations
C. Blood pressure

D. Urinary output
CORRECT ANSWER: B.
In an emergency situation, the nurse should determine breathlessness first, then pulselessness.
Blood pressure would be assessed after these assessments were determined. Urinary output is
also important but not the priority at this time.
9. The client arrives in the emergency room after sustaining a chemical eye injury from a
splash of battery acid. The initial nursing action is to:
A. Begin visual acuity testing
B. Irrigate the eye with sterile normal saline
C. Swab the eye with antibiotic ointment
D. Cover the eye with a pressure patch.
CORRECT ANSWER: B.
Emergency care following a chemical burn to the eye includes irrigating the eye immediately
with sterile normal saline or ocular irrigating solution. In the emergency department, the
irrigation should be maintained for at least 10 minutes. Following this emergency treatment,
visual acuity is assessed.
10. The nurse in the emergency department has admitted five clients in the last two hours
with complaints of fever and gastrointestinal distress. Which question would be most
appropriate for the nurse to ask each client to determine if there is bioterrorism threat?
A. Do you work or live near any large power lines?
B. Where were you immediately before you got sick?
C. Can you write down everything you ate today?
D. What other health problems do you have?
CORRECT ANSWER: B.
The nurse should take note of any unusual illness for the time of year or clusters of clients
coming from a single geographical location who all exhibit signs and symptoms of possible
biological terrorism.
11. The nurse in the emergency department has admitted five clients in the last two hours
with complaints of fever and gastrointestinal distress. Which question would be most
appropriate for the nurse to ask each client to determine if there is bioterrorism threat?
A. Do you work or live near any large power lines?
B. Where were you immediately before you got sick?
C. Can you write down everything you ate today?
D. What other health problems do you have?

CORRECT ANSWER: B.
The nurse should take note of any unusual illness for the time of year or clusters of clients
coming from a single geographical location who all exhibit signs and symptoms of possible
biological terrorism.
12. The triage nurse in a large trauma center has been notified of an explosion in a major
chemical manufacturing plant. Which action should the nurse implement first when the
clients arrive in the emergency department?
A. Triage the clients and send them to their appropriate areas
B. Thoroughly was the clients with soap and water and then rinse
C. Remove the clients clothing and have them shower
D. Assume the clients have been decontaminated in the plant
CORRECT ANSWER: C.
This is the first step. Depending on the type of exposure, this step alone can remove a large
portion of exposure.
13. The nurse at a disaster site is triaging victims when a woman states, I am a Certified
Nurse Aide, Can I do anything to help? Which action should the nurse implement?
A. Request the woman to please leave the area
B. Ask the woman to check the injured clients
C. Tell the woman to try and keep the victims calm
D. Instruct the woman to help the paramedics
CORRECT ANSWER: C.
Unlicensed assistive personnel (UAP) have the ability to keep the victims calm, therefore this is
an appropriate intervention. Option A is wrong because the nurse should utilize as many
individual as possible to help in the situation. UAP cannot assess clients; therefore this is not an
appropriate answer. Option D is wrong because the paramedics dont need civilians assisting
them as they stabilize and transport the victims.
14. A 60-year-old male client comes into the emergency department with complaints of
crushing chest pain that radiates to his shoulder and left arm. The admitting diagnosis is
acute myocardial infarction. Immediate admission orders include oxygen by NC at
4L/minute, blood work, chest x-ray, an ECG, and 2mg of morphine given intravenously.
The nurse should first:
A. Administer the morphine
B. Obtain a 12-lead ECG
C. Obtain the lab work
D. Order the chest x-ray

CORRECT ANSWER: A.
Although obtaining the ECG, chest x-ray, and blood work are all important, the nurses priority
action would be to relieve the crushing chest pain.
15. A client enters the ER complaining of severe chest pain. A myocardial infarction is
suspected. A 12 lead ECG appears normal, but the doctor admits the client for further
testing until cardiac enzyme studies are returned. All of the following will be included in
the nursing care plan. Which activity has the highest priority?
A. Monitoring vital signs
B. Completing a physical assessment
C. Maintaining cardiac monitoring
D. Maintaining at least one IV access site
CORRECT ANSWER: C.
Even though initial tests seem to be within normal range, it takes at least 3 hours for the cardiac
enzyme studies to register. In the meantime, the client needs to be watched for bradycardia, heart
block, ventricular irritability, and other arrhythmias. Other activities can be accomplished
around the MI monitoring.
16. The client is admitted to the ER following a vehicular accident. The client was wearing a
lap seat belt when the accident occurred. The client has hematuria and lower abdominal
pain. To determine further whether the pain is due to bladder trauma, the nurse asks the
client if the pain is referred to which of the following areas?
A. Shoulder
B. Umbilicus
C. Costovertebral angle
D. Hip
CORRECT ANSWER: A.
Bladder trauma or injury is characterized by lower abdominal pain that may radiate to one of
the shoulders. Bladder injury pain does not radiate to the umbilicus, CV angle, or hip.
17. The administrative supervisor is staffing the hospitals medical-surgical units during an
ice storm and has received many calls from staff members who are unable to get to the
hospital. Which action should the supervisor implement first?

A.
B.
C.
D.

Inform the chief nursing officer


Notify the on-duty staff to stay
Call staff members who live close to the facility
Implement the emergency disaster protocol

CORRECT ANSWER: B.
The first action for the administrative supervisor is to make sure the clients receive care. The
supervisor cannot allow the on-duty staff to leave until replacement staff members have been
arranged. The chief nursing officer should be informed, but this is not the first action. The
supervisor should call any staff that can get to the hospital in an attempt to staff the hospital, but
this is not the first action to implement. An emergency disaster protocol may be implemented, but
the first intervention is to ensure that the clients have a nurse on duty.
18. The charge nurse of a surgical unit has been notified of an external disaster with multiple
casualties. Which client should the charge nurse request to be discharged from the
hospital to make room for clients from the disaster?
A. The client scheduled for a bilateral adrenalectomy in the morning whose
preoperative teaching has not been started.
B. The client who had a total abdominal hysterectomy 2 days ago and PCA machine
has been discontinued.
C. The client who is postoperative bilateral thyroidectomy who has hemoglobin of 7
mg/dL and a hematocrit of 22.1%.
D. The client with type 2 diabetes who has just had a kidney transplant and is
experiencing fever and pain at the surgical site.
CORRECT ANSWER: B.
This client is stable and could be prescribed oral pain medication; this patient could be
discharged home and followed by home health nursing if needed. This client is the most
appropriate client to request to be discharged. In option A, the client needs preoperative
teaching and the charge nurse should not request a discharge for a client having surgery in the
morning. In option C, the client is experiencing a complication of surgery and is hemorrhaging;
the hemoglobin and hematocrit is very low. Therefore, this client cannot be sent home. In option
D, the client may be showing signs of acute rejection; therefore, this client cannot be discharged
home.
19. A major disaster has been called, and the charge nurse on the medical unit must
recommend to the medical discharge officer on rounds which client to discharge. Which
client should not be discharged?
A. The client diagnosed with chronic angina pectoris that has been on new
medication for 2 days.
B. The client diagnosed with deep vein thrombosis (DVT) who has had heparin
discontinued and has been on warfarin (Coumadin) for 4 days.
C. The client with an infected leg wound who is receiving Vancomycin IVPB every
24 hours for methicillin-resistant Staphylococcus aureus (MRSA) infection.
D. The client diagnosed with COPD who has the following arterial blood gas (ABG)
levels: pH, 7.34; PCO2, 55; HCO3, 28; PaO2, 89.
CORRECT ANSWER: C.

Because resistant infections are difficult to treat, the client should remain in the hospital for the
required IVPB medication. In option A, the client is already on the second day of her angina
medication; thus, making the client stable for discharge. In option B, the client could be taught
to continue the Coumadin therapy at home and return to the health care providers clinic for
blood work, or a home health nurse may be assigned to the client for a follow-up. In option D,
the ABG levels are expected with a client who has COPD, this client could go home with oxygen
and home health follow-up care.
20. The community health nurse is triaging victims at the scene of a building collapse. Which
intervention should the nurse implement first?
A. Discuss the disaster situation with the media
B. Write the clients name clearly in the disaster log
C. Place disaster tags securely on the victims
D. Identify an area for family members to wait
CORRECT ANSWER: C.
Client tracking is a critical component of casualty management. Disaster tags, which include
name, address, age, location, description of injuries, and treatments or medication administered,
must be securely attached to the client. Option A is wrong because a spokesperson should
address the media away from the victim care area as soon as possible. This could be a nurse in
some situation, but this is not a priority intervention during triage. The disaster tag number and
the clients name should be recorded in the disaster log book, but it is not the priority
intervention. The disaster tag must be attached to the client prior to logging in the disaster log
book. Family and friends arriving at the disaster must be cared for by the disaster workers, but it
is not the first intervention for the nurse who is triaging disaster victims, thus making option D
wrong.
21. A client comes into the ER after hitting his head in an MVA. Hes alert and oriented.
Which of the following nursing interventions should be done first?
A. Assess full ROM to determine extent of injuries
B. Call for an immediate chest x-ray
C. Immobilize the clients head and neck
D. Open the airway with the head-tilt chin-lift maneuver
CORRECT ANSWER: C.
All clients with a head injury are treated as if a cervical spine injury is present until x-rays
confirm their absence. ROM would be contraindicated at this time. There is no indication that
the client needs a chest x-ray. The airway doesnt need to be opened since the client appears
alert and not in respiratory distress. In addition, the head-tilt chin-lift maneuver wouldnt be
used until the cervical spine injury is ruled out.
22. A 20-year-old client who fell approximately 30 feet is unresponsive and breathless. A

cervical spine injury is suspected. How should the first-responder open the clients airway
for rescue breathing?
A. By inserting a nasopharyngeal airway
B. By inserting a oropharyngeal airway
C. By performing a jaw-thrust maneuver
D. By performing the head-tilt, chin-lift maneuver
CORRECT ANSWER: C.
If the client has a suspected cervical spine injury, a jaw-thrust maneuver should be used to open
the airway. If the tongue or relaxed throat muscles are obstructing the airway, a nasopharyngeal
or oropharyngeal airway can be inserted; however, the client must have spontaneous
respirations when the airway is open. The head-tilt, chin-lift maneuver requires neck
hyperextension, which can worsen the cervical spine injury.
23. A newly graduated nurse is learning about the nurses role in disaster relief as part of the
orientation to the hospital. Which of the following concepts is accurate?
A. Nurses take a passive role in helping others to save lives and fulfill an important
obligation
B. Learning about the prevention and mitigation of disasters is nice to know, but not
essential
C. Applying advanced skills can be very helpful until help arrives
D. Nurses may have to assume expanded roles in making decisions for the most
appropriate treatment of casualties
CORRECT ANSWER: D.
Nurses must be aware of the roles nurses play in all aspects of disaster preparedness and
response. Option A is wrong because the nurses role is active, not passive. Option B is incorrect
because learning about disasters is essential. Option C is wrong because basic skills can be
applied.
24. A patient is brought to the emergency department after a bee sting. The family reports a
history of severe allergic reaction, and the patient appears to have some oral swelling.
Which of the following is the most urgent nursing action?
A. Consult a physician
B. Maintain a patent airway
C. Administer epinephrine subcutaneously
D. Administer diphenhydramine (Benadryl) orally

CORRECT ANSWER: B.
The patient may be experiencing an anaphylactic reaction. The most urgent action is to maintain
an airway, particularly with visible oral swelling, followed by the administration of epinephrine
by subcutaneous injection. The physician will see the patient as soon as possible with the above
actions underway. Oral diphenhydramine is indicated for mild allergic reactions and is not
appropriate for anaphylaxis.
25. A client who is in sickle cell crisis arrives in the emergency department. Which should
the nurse prepare for as the priority in the management for the client?
A. Pain management
B. Iron administration
C. Oxygen administration
D. Red blood cell transfusion
CORRECT ANSWER: C.
The priority nursing intervention for a client in sickle cell crisis is to administer supplemental
oxygen because the client is hypoxemic, and as a result, the red blood cells change to a sickle
shape. In addition, oxygen is the priority because airway and breathing are more important than
circulatory needs. The nurse also plans for fluid therapy to promote hydration and reverse the
agglutination of sickled cells, opioid analgesics for relief from severe pain, and blood
transfusions (rather than iron administration) to increase the bloods oxygen-carrying capacity.

TRANSCULTURAL
NURSING

1. The ambulatory care nurse is discussing preoperative procedures with a Japanese


American client who is scheduled for surgery the following week. During the discussion,
the client continually smiles and nods the head. How should the nurse interpret this
nonverbal behavior?
A. Reflecting a cultural value.
B. An acceptance of the treatment
C. Client agreement to the required procedures.
D. Client understanding of the preoperative procedures
CORRECT ANSWER: A.
Nodding or smiling by a Japanese American client may reflect only the cultural value of
interpersonal harmony. This nonverbal behavior may not be an indication of acceptance of the
treatment, agreement with the speaker, or understanding of the procedure.
2. When communicating with a client who speaks different language, which BEST practice
should the nurse implement?
A. Speak loudly and slowly.
B. Arrange for an interpreter to translate.
C. Speak to the client and family together.
D. Stand close to the client and speak loudly.
CORRECT ANSWER: B.
Arranging for an interpreter would be the best practice when communicating with a client who
speaks a different language. Options A and D are inappropriate and are ineffective ways to
communicate. Option C is inappropriate because it violates privacy and does not ensure correct
translation.
3. The nurse educator is providing in-service education to the nursing staff regarding
transcultural nursing care; a staff member asks the nurse educator to describe the concept
of acculturation. The nurse educator should make which of the most appropriate
response?
A. It is a process of learning a different culture to adapt to a new or changing
environment.
B. It is a subjective perspective of the persons heritage and a sense of belonging to
the group.
C. It is a group of individuals in a society who are culturally distinct and have a
unique identity.
D. It is a group that shares some of the characteristics of the larger population group
of which it is a part.
CORRECT ANSWER: A.
Acculturation is a process of learning a different culture to adapt to a new or changing
environment. Option B describes ethnic identity. Option C describes an ethnic group. Option D
describes a subculture.
4. The nurse is providing discharge instructions to a Chinese American client regarding

prescribed dietary modifications. During the teaching session, the client continuously
turns away from the nurse. The nurse should implement which best action?
A. Continue with the instructions, verifying client understanding.
B. Walk around the client so that the nurse constantly faces the client.
C. Give the client a dietary booklet and return later to continue with the instructions.
D. Tell the client about the importance of the instructions for the maintenance of
health care.
CORRECT ANSWER: A.
Most Chinese Americans maintain a formal distance with others, which is a form of respect.
Many Chinese Americans are uncomfortable with face-to-face communications, especially when
eye contact is direct. If the client turns away from the nurse during a conversation, the best
action is to continue with the conversation. Walking around the client so that the nurse faces the
client is in direct conflict with this cultural practice. The client may consider it a rude gesture if
the nurse returns later to continue with the explanation. Telling the client about the importance
of the instructions for the maintenance of health care may be viewed as degrading.
5. The nurse identifies low-risk therapies to a client and should include which therapy(s) in
the discussion. Select all that apply.
A. Herbs
B. Prayer
C. Touch
D. Massage
E. Relaxation
F. Acupuncture
CORRECT ANSWER: B, C, D, E
Low-risk therapies are therapies that have no adverse effects and, when implementing care, can
be used by the nurse who has training and experience in their use. Low-risk therapies include
meditation, relaxation techniques, imagery, music therapy, massage, touch, laughter and humor,
and spiritual measures, such as prayer. The other options are not considered low-risk therapies.
6. Which client has a high-risk of obesity and diabetes mellitus? Select all that apply.
A. A 40-year-old Latino American man
B. A 45-year-old Native American man
C. A 23-year-old Asian American woman
D. A 35-year-old Hispanic American man
E. A 40-year-old African American woman
CORRECT ANSWER: A, B, D, E
Because of their health and dietary practices, Native Americans, Latino Americans, Hispanic
Americans, and African Americans have a high risk of obesity and diabetes mellitus. Asian
Americans have a lower risk for obesity and diabetes mellitus.
7. The nurse is preparing a plan of care for a client who is a Jehovahs Witness. The client
has been told that surgery is necessary. The nurse considers the clients religious

preferences in developing a plan of care and should document which information?


A. The client believes the soul lives on after death.
B. Medication administration is not allowed.
C. Surgery is prohibited in this religious group.
D. The administration of blood and blood products is not allowed.
CORRECT ANSWER: D.
Among Jehovahs Witnesses, surgery is not prohibited, but the administration of blood and blood
products is forbidden. This religious group believes the soul cannot live after death.
Administration of medication is an acceptable practice except if the medication is derived from
blood products.
8. Which meal tray should the nurse deliver to a client of Orthodox Judaism faith who
follows a kosher diet?
A. Pork roast, rice, vegetables, mixed fruit, milk
B. Crab salad on a croissant, vegetables with dip, potato salad, milk
C. Sweet and sour chicken with rice and vegetables, mixed fruit, juice
D. Noodles and cream sauce with shrimp and vegetables, salad, mixed fruit, iced tea
CORRECT ANSWER: C.
Orthodox Judaism believers adhere to dietary kosher laws. In this religion, the dairy-meat
combination is unacceptable. Only fish that have scale and fins are allowed; meats that are
allowed include animals that are vegetable-eaters, cloven-hoofed, and ritually slaughtered.
9. An Asian American client is experiencing a fever. The nurse recognizes that the client is
likely to self-treat the disorder, using which method?
A. Prayer
B. Magnetic therapy
C. Foods considered to be yin
D. Foods considered to be yang
CORRECT ANSWER: C.
In the Asian-American culture, health is believed to be a state of physical and spiritual harmony
with nature and a balance between positive and negative energy forces (yin and yang). Yin foods
are cold and yang foods are hot. Cold foods are eaten when one has a hot illness (fever), and hot
foods (yang) are eaten when one has a cold illness.
10. The role of the nurse regarding complementary and alternative medicine should include
which action?
A. Advising the client about good versus bad therapies
B. Recommending herbal remedies that the client should use
C. Discouraging the client from using any alternative therapies
D. Educating the client about therapies that he or she is using or is interested of using
CORRECT ANSWER: D.
Complementary (alternative) therapies include a wide variety of treatment modalities that are

used in addition to conventional therapy to treat a disease or illness. Educating the client about
therapies that he or she uses or is interested in using is the nurses role. Options A, B, C are
inappropriate actions for the nurse to take because they provide advice to the client.
11. An antihypertensive medication has been prescribed for a client with hypertension. The
client tells the clinic nurse that they would like to take an herbal substance to help lower
their blood pressure. The nurse should take which action?
A. Tell the client that herbal substances are not safe and should never be used.
B. Teach the client how to take their blood pressure so that it can be monitored
closely.
C. Encourage the client to discuss the use of an herbal substance with the health care
provider.
D. Tell the client that if they take the herbal substance they will need to have their
blood pressure checked frequently.
CORRECT ANSWER: C.
Although herbal substances may have some beneficial effects, not all herbs are safe to use.
Clients who are being treated with conventional medication therapy should be encouraged to
avoid herbal substances with similar pharmacological effects because the combination may lead
to an excessive reaction or to unknown interaction effects. The nurse should advise the client to
discuss the use of the herbal substance with the HCP.
12. The nurse educator asks a student to list the five categories of complementary and
alternative medicine (CAM), developed by, the National Center for Complementary and
Alternative Medicine. Which statement, if made by the nursing student, would indicate an
understanding of the five categories of CAM?
A. Herbology, hydrotherapy, acupuncture, nutrition, and chiropractic care
B. Mind-body medicine, traditional Chinese medicine, homeopathy, naturopathy, and
healing touch
C. Biologically based practices, body-based practices, magnetic therapy, massage
therapy, and aromatherapy
D. Whole medical systems, mind-body medicine, biologically based practices,
manipulative and body-based practices, and energy medicine
CORRECT ANSWER: D.
The five categories of complementary and alternative medicine (CAM) include whole medical
systems, mind-body medicine, biologically based practices, manipulative and body-based
practices, and energy medicine. The other options contain therapies within each category of
CAM.
13. The nurse in charge is caring for an Italian client. Hes complaining of pain, but he falls
asleep right after his complaint and before the nurse can assess his pain. The nurse
concludes that:
A. He may have a low threshold for pain
B. He was faking pain
C. Someone else gave him medication

D. The pain went away


CORRECT ANSWER: A.
People of Italian heritage tend to verbalize discomfort and pain. The pain was real to the client,
and he may need medication when he wakes up.
14. Which statement would best explain the role of the nurse when planning care for a
culturally diverse population? The nurse will plan care to:
A. Include care that is culturally congruent with the staff from
predetermined criteria
B. Focus only on the needs of the client, ignoring the nurses beliefs and
practices
C. Blend the values of the nurse that are for the good of the client and
minimize the clients individual values and beliefs during care
D. Provide care while aware of ones own bias, focusing on the clients
individual needs rather than the staffs practices

CORRECT ANSWER: D.
Without understanding ones own beliefs and values, a bias or preconceived belief by the nurse
could create an unexpected conflict or an area of neglect in the plan of care for a client (who
might be expecting something totally different from the care). During assessment values, beliefs,
practices should be identified by the nurse and used as a guide to identify the choices by the
nurse to meet specific needs/outcomes of that client. Therefore identification of values, beliefs,
and practices allows for planning meaningful and beneficial care specific for this client.
15. Transcultural nursing implies:
A. Using a comparative study of cultures to understand similarities and differences
across human groups to provide specific individualized care that is culturally
appropriate
B. Working in another culture to practice nursing within their limitations
C. Combining all cultural beliefs into a practice that is a nonthreatening approach to
minimize cultural barriers for all clients equality of care
D. Ignoring all cultural differences to provide the best generalized care to all clients.
CORRECT ANSWER: A.
Transcultural care means that by understanding and learning about specific cultural practices
the nurse can integrate these practices into the plan of care for a specific individual client who
has the same beliefs or practices to meet the clients needs in a holistic manner of care.
16.The nurse and client are developing a transcultural nursing care plan

related to heart disease. The nurse will include in the plan:


A. Educational material that is given to all clients with heart disease.
B. Religious practices that are relevant to the client.
C. Nutritional teaching specific to a cardiac diet.
D. Contact information for a community support group.

CORRECT ANSWER: B.
Cultural competence includes the ability to incorporate the client's religious
practices and beliefs into nursing care. Educational material needs to be
culturally appropriate and relevant to the client and not the general population.
It is important to understand the client's orientation to health, disease, and
cultural practices in order to provide appropriate nutritional teaching. Before
referring the client to a support group, the nurse needs to understand the
client's beliefs related to health care and healing practices.

17.The nurse is organizing a disease prevention program for a specific


cultural group. To effectively meet the needs of this group the nurse will:
A. Assess the needs of the community in general.
B. Develop generalized goals and objectives for the program.
C. Assess the immediate impact of such a program.
D. Involve those affected by the problem.

CORRECT ANSWER: D.
(A) Assess the needs of the community in general. The nurse should assess the
needs of the cultural community and not the community at large.
(B) Develop generalized goals and objectives for the program. Goals and
objectives need to be specific to the target group.
(C) Assess the immediate impact of such a program. Evaluating the impact of
the program would be conducted after the program has occurred.
(D) Involve those affected by the problem. When planning a program, it is

important to involve those affected by the problem. This will better serve to
meet the group's needs.

18.The most effective way for a nurse to enhance her understanding of a


specific cultural group is to:
A. Treat all clients the same
B. Use educational materials that are simplistic and have many
pictures.
C. Learn about traditional healers within the community.
D. Educate the specific cultural group about Western concepts of
health and illness.

CORRECT ANSWER: C.
(A) Treat all clients the same. Cultural competence and sensitivity requires an
understanding of the culture's orientation to health, disease, health traditions,
and traditional healing practices.
(B) Use educational materials that are simplistic and have many pictures.
Educational materials need to be culturally appropriate and relevant to the
group.
(C) Learn about traditional healers within the community. Learning about
traditional healers and their practices within the community provides the nurse
with a resource for the members of the cultural group. This is the only option
that enhances the nurse's understanding of a specific cultural group.
(D) Educate the specific cultural group about Western concepts of health and
illness. Explaining Western concepts of health and health care should be
relevant and understandable to the culture of interest.

19. Which activity would not be expected by the nurse to meet the cultural needs of the
client?
A. Promote and support attitudes, behaviors, knowledge, and skills to respectfully
meet clients cultural needs despite the nurses own beliefs and practices
B. Ensure that the interpreter understands not only the language of the client but
feelings and attitudes behind cultural practices to make sure an ethical balance can
be achieved
C. Develop structure and process for meeting cultural needs on a regular basis and
means to avoid overlooking these needs with clients

D. Expect the family to keep an interpreter present at all times to assist in meeting
the communication needs all day and night while hospitalized

CORRECT ANSWER: D.
It is not the familys responsibility to assist in the communication process. Many families will
leave someone to help at times, but it is the hospitals legal obligation to find an interpreter for
continued understanding by the client to make sure the client is fully informed and comprehends
in his or her primary language.

20.Cultural awareness is an in-depth self-examination of ones:


A. Background, recognizing biases and prejudices.
B. Social, cultural, and biophysical factors
C. Engagement in cross-cultural interactions
D. Motivation and commitment to caring.

CORRECT ANSWER: A.
Cultural awareness is an in-depth examination of ones own background,
recognizing biases and prejudices and assumptions about other people.

21. Cultural competence is the process of:


A. Learning about vast cultures
B. Acquiring specific knowledge, skills, and attitudes
C. Influencing treatment and care of clients
D. Motivation and commitment to caring.
CORRECT ANSWER: B.
Cultural competence is the process of acquiring specific knowledge, skills, and
attitudes that ensure delivery of culturally congruent care.

22. Culture strongly influences pain expression and need for pain medication. However,
cultural pain:
A. May be suffered by a client whose valued way of life is disregarded by
practitioners.
B. Is more intense, thus necessitating more medication.
C. Is not expressed verbally or physically
D. Is expressed only to others of like culture.
CORRECT ANSWER: A.
Nurses need not assume that pain relief is equally valued across groups.
Cultural pain may be suffered by a client whose valued way of life is disregarded
by practitioners.

23.When the nurse is providing care for a client who is not fluent in English
language, what is the initial nursing action?
A. Determine the clients primary language.
B. Obtain a translation dictionary for the client.
C. Establish a means of communication using gestures and hand
movements.
D. Obtain a pad and paper so that the client can write and draw to
express needs.
CORRECT ANSWER: A
Language is the largest barrier for people who do not speak English or cannot
communicate in English effectively. The nurse needs to assess the clients ability
to communicate. If the client is not fluent in English, the nurse should first
determine his or her primary language. Then a trained medical interpreter
designated by the health care facility should be contacted for communication
with the client to prevent errors in communication that could lead to client harm.
The client should not be expected to use a translation dictionary or use gestures
and hand movements because this could cause confusion and errors in
communication. Writing or drawing needs on a pad of paper could also cause
confusion and lead to error.

24.The nurse who practices culturally sensitive nursing care incorporates


which characteristics? Select all that apply.
A. The expression of pain is affected by learned behaviors.
B. Physiologically, all individuals experience pain in a similar manner.
C. Ethnic culture has an effect on the physiological response to pain
medications.
D. Clients should be assessed for pain regardless of a lack of overt
symptomatology.
E. The use of standardized pain assessment tool ensures unbiased
pain assessment.

CORRECT ANSWER: A, C, D
Pain and its expression are often affected by an individuals ethnic culture in
ways that include learned means of pain expression, the physiological response
to pain medications, and attitudes regarding acceptable ways of dealing with
pain. Physiologically not all individuals, even those of the same ethnic culture,
will respond to pain in a similar manner, and so a standardized pain assessment
tool is not effective in measuring pain in all clients.

25.The nurse is performing a socioeconomic assessment of a Chinese client.


Which questions would be appropriate for the nurse to ask? Select all that
apply.
A. What do you do for a living?
B. How much money do you make yearly?
C. Do you have a primary health care provider?
D. How many years of school did you complete?
E. How different is your life here from in your homeland?

F. What type of work did you do back in your homeland?

CORRECT ANSWER: A, C, D, E, F
Aspects to include in a cultural assessment include biocultural history and
socioeconomic status and the clients country of origin. Other aspects to assess
include religious and spiritual beliefs, communication patterns, time orientation,
caring beliefs and practices, and previous experiences with professional health
care. Asking the client about his or her yearly income is inappropriate,
unnecessary, and unrelated to health care resources.

NURSING ISSUES
&
LEGAL NURSING

1. The nurse is suspected of charting medication administration that he did not give. After
talking to the nurse, the charge nurse should:
A. Call the Board of Nursing.
B. File a formal reprimand.
C. Terminate the nurse.
D. Charge the nurse with a tort.
CORRECT ANSWER: B.
The next action after discussing the problem with the nurse is to document the incident by filing
a formal reprimand. If the behavior continues or if harm has resulted to the client, the nurse may
be terminated and reported to the Board of Nursing, but these are not the first actions requested
in the stem. A tort is a wrongful act to the client or his belongings and is not indicated in this
instance. Therefore, answers A, C, and D are incorrect.
2. Nurses agree to be advocates for their patients. Practice of advocacy calls for the nurse to:
A. Seek out the nursing supervisor in conflicting situations
B. Work to understand the law as it applies to the clients clinical condition.
C. Assess the clients point of view and prepare to articulate this point of view.
D. Document all clinical changes in the medical record in a timely manner.

CORRECT ANSWER: C.
Nurses strengthen their ability to advocate for a client when nurses are able to
identify personal values and then accurately identify the values of the client and
articulate the clients point of view.

3. The nurse hears a client calling for help, hurries down the hallway to the clients room,
and finds the client lying on the floor. The nurse performs an assessment, assists the client
back to bed, notifies the health care provider of the incident, and completes an incidental
report. Which statement should the nurse document on the incident report?
A. The client fell out of bed.
B. The client climb over the side rails.
C. The client was found lying on the floor.
D. The client became restless and tried to get out of bed.
CORRECT ANSWER: C.
The incident report should contain the clients name, age, and diagnosis. The report should
contain a factual description of the incident, any injuries experienced by those involved, and the
outcome of the situation. Options A, B, D are interpretations of the situation and are not factual
information as observed by the nurse.
4. The nurse has just assisted a client back to bed after a fall. The nurse and health care
provider have assessed the client and have determined that the client is not injured. After

completing the incident report, the nurse should implement which action next?
A. Reassess the client.
B. Conduct a staff meeting to describe the fall.
C. Document in the nurses notes that an incident report was completed.
D. Contact the nursing supervisor to update information regarding the fall.
CORRECT ANSWER: A.
After a clients fall, the nurse must frequently reassess the client because potential complications
do not always appear immediately after the fall. The clients fall should be treated as private
information and shared on a need to know basis. Communication regarding the event should
involve only the individuals participating in the clients care. An incident report is a problemsolving document; however its completion is not documented in the nurses notes. If the nursing
supervisor has been made aware of the incident, the supervisor will contact the nurse if status
update is necessary.
5. The nurse arrives at work and is told to report (float) to the intensive care unit (ICU) for
the day because the ICU is understaffed and needs additional nurses to care for the
clients. The nurse has never worked in the ICU. The nurse should take which action first?
A. Call the hospital lawyer.
B. Refuse to float to the ICU.
C. Call the nursing supervisor.
D. Identify tasks that can be performed safely in the ICU.
CORRECT ANSWER: D.
Floating is an acceptable legal practice used by hospitals to solve understaffing problems.
Legally, the nurse cannot refuse to float unless a union contract guarantees that nurses can work
only in a specified area or the nurse can prove the lack of knowledge for the performance of
assigned tasks. When encountering this situation, the nurse should set priorities and identify
potential areas of harm to the client. The nursing supervisor is called when if the nurse is
expected to perform tasks that he or she cannot safely perform. Calling the hospital lawyer is a
premature action.
6. The nurse works on the night shift enters the medication room and finds a co-worker with
a tourniquet wrapped around the upper arm. The co-worker is about to insert a needle,
attached to a syringe containing a clear liquid, into the antecubital area. Which is the most
appropriate action by the nurse?
A. Call security.
B. Call the police.
C. Call the nursing supervisor.
D. Lock the co-worker in the medication room until help is obtained.
CORRECT ANSWER: C.
Nurse practice acts require reporting impaired nurses. The board of nursing has jurisdiction
over the practice of nursing and may develop plans for treatment and supervision of the
impaired nurse. The incident needs to be reported to the nursing supervisor, who will then report
to the board of nursing and other authorities, such as the police, as required. The nurse may call

a security if a disturbance occurs, but no information in the question supports this need, and so
this is not the appropriate action. Option D is an inappropriate and unsafe action.
7. The nurse has made an error in a narrative documentation of an assessment finding on a
client and obtains the clients record to correct the error. The nurse should take which
action to correct the error?
A. Documenting a late entry into the clients record.
B. Trying to erase the error for space to write in the correct data.
C. Using whiteout to delete the error to write in the correct data.
D. Drawing one line through the error, initialing and dating and then documenting
the correct information.
CORRECT ANSWER: D.
If the nurse makes an error in narrative documentation in the clients record, the nurse should
follow agency policies to correct the error. This includes drawing one line through the error,
initialing and dating and then documenting the correct information. A late entry is used to
document additional information not remembered at the initial time of documentation. Erasing
data from the clients record and the use of whiteouts are prohibited.
8. Which identifies accurate nursing documentation notations? Select all that apply.
A. The client slept through the night.
B. Abdominal wound dressing is dry and intact without drainage.
C. The client seemed angry when awakened for vital sign measurement.
D. The client appears to become anxious when it is time for respiratory treatments.
E. The clients left lower medial leg wound is 3 cm in length with redness, drainage,
or edema.
CORRECT ANSWERS: A, B, E
Factual documentation contains descriptive, objective information about what the nurse sees,
hears, feels, or smells. The use of inferences without supporting factual data is not acceptable
because it can be misunderstood. The use of vague terms, such as seemed or appears is not
acceptable because these words suggest that the nurse is stating an opinion.
9. The nurse calls the health care provider (HCP) regarding a new medication prescription
because the dosage prescribed is higher than the recommended dosage. The nurse is
unable to locate the HCP, and the medication is due to be administered. Which action
should the nurse take?
A. Contact the nursing supervisor.
B. Administer the dose prescribed.
C. Hold the medication until the HCP can be contacted.
D. Administer the recommended dose until HCP can be located.
CORRECT ANSWER: A.
If the HCP writes a prescription that requires clarification, the nurses responsibility is to
contact the HCP. If there is no resolution regarding the prescription because the HCP cannot be
located or because the prescription remains as it was written after talking with the HCP, the

nurse should contact the nurse manager or nursing supervisor for further clarification as to
what the next step should be. Under no circumstances should the nurse proceed to carry out the
prescription until obtaining clarification.
10. A client asks the nurse to act as a witness for an advance directive. Which is the best
intervention for the nurse to implement?
A. Decline the clients request politely.
B. Agree to sign the document as a witness.
C. Notify the provider of the clients request.
D. Help the client find an unrelated third party.
CORRECT ANSWER: D.
An advance directive addresses the withdrawal or withholding of life-sustaining interventions
that can prolong life and identifies the person who will make care decisions if the client becomes
incompetent. Two people unrelated to the client witness the clients signature and then sign the
document signifying that the client signed the advance directive authentically. Nurses or
employees of a facility in which the client is receiving care and beneficiaries of the client should
not serve as a witness because of conflict of interest concerns. There is no reason to call the
provider unless the absence of the advance directive interferes with client care.
11. The nurse identifies which clinical situation as slander?
A. The health care provider tells a client that the nurse does not know anything.
B. The nurse tells a client that a nasogastric tube will be inserted if the client
continues to refuse to eat.
C. The nurse restrains a client at bedtime because the client gets up during the night
and wanders around.
D. The laboratory technician restrains the arm of a client refusing to have blood
drawn so that the specimen can be obtained.
CORRECT ANSWER: A.
Defamation takes place when a falsehood is said (slander) or written (libel) about a person that
results in injury to that persons good name and reputation. Battery involves offensive touching
or the use of force by a perpetrator without the permission of the victim. An assault occurs when
a person puts another person in fear of a harmful or offensive act.
12. During the admission process of a client who is admitted to the hospital for surgery to
remove a tumor, the client asks the nurse if a living will, prepared 3 years ago, remains in
effect. Which response should the nurse provide to the client?
A. Yes, a living will never expires.
B. You need to speak with an attorney.
C. I will call someone to answer your question.
D. It requires renewal yearly with your health care provider.
CORRECT ANSWER: D.
The client should discuss the living will with the health care provider (HCP) on a regular basis
(at least annually) to ensure that it contains the clients current wishes and desires based on the

clients current health status. Option A is incorrect. Although the client consults an attorney if
the living will must be changed, the accurate nursing response is to tell the client that a living
will should be reviewed. Option C is not at all helpful to the client and is, in fact, a
communication block and places the clients question on hold.
13. The nurse does not intervene when a client becomes hypotensive after surgery. The client
requires emergency surgery to stop postoperative bleeding later that night. The nurse
could potentially face which types of prosecution for failing to act? Select all that apply.
A. Felony
B. Tort law
C. Malpractice
D. Statutory law
E. Misdemeanor
CORRECT ANSWER: B, C
Tort law deals with wrongful acts intentionally or unintentionally committed against a person or
the persons property. The nurse commits a tort offense by failing to act when the client became
hypotensive. Malpractice occurs when a duty to the client is established and the nurse neglects
to act responsibly. Options A and E are offenses under criminal law. Option D describes laws
enacted by state, federal, or local governments.
14. An individual from the community is admitted to the hospital with a diagnosis of
Parkinsons disease. The nurse gives medical information regarding the clients condition
to a person who is assumed to be a family member. Later the nurse discovers that this
person is not a family member and realizes that this violated which legal concepts of the
nurse-client relationship? Select all that apply.
A. Duty to provide care.
B. Clients right to privacy.
C. Nurses lack of experience
D. Clients right to confidentiality
E. Failure to communicate to the health care provider
CORRECT ANSWERS: B, D
Discussing a clients condition without client permission violates a clients rights to privacy and
confidentiality and places the nurse in legal jeopardy. This action by the nurse is both an
invasion of privacy and affects the confidentiality issue with client rights. Options A, C, and E do
not represent violation of the situation presented.
15. Which clinical situation should be viewed as assault?
A. The nurse threatens to apply restraints to a client who is exhibiting aggressive
behavior.
B. The client requests a medical discharge, but the nurse physically forces the client
to stay.
C. The charge nurse sends an email to a staff member which includes a poor
performance evaluation about another person.
D. The nurse overhears the health care provider making derogatory remarks to the

client about the nurses level of competency.


CORRECT ANSWER: A.
An assault occurs when a person puts another person in fear of a harmful or offensive act.
Battery involves offensive touching or the use of force by a perpetrator without the permission of
the victim. Defamation takes place when a falsehood is said (slander) or written (libel) about a
person that results in injury to that persons good name and reputation.
16. A client had a colon resection. A nasogastric tube was in place when a regular diet was
brought to clients room. The client did not want to eat solid food and asked that the
health care provider be called. The nurse insisted that the solid food was the correct diet.
The client ate and subsequently had additional surgery as a result of complications. The
determination of negligence is based on which premise?
A. The nurse persistence
B. A duty existed and it was breached
C. Not notifying the health care provider
D. The dietary department sending the wrong food
CORRECT ANSWER: B.
For negligence to be proved, there must be a duty, and then a breach of duty; the breach of duty
must cause the injury, and damages or injury must be experienced. Options A, C, D do not fall
under the criteria for negligence. Option B is the only option that fits the criteria of negligence.
17. A client asks the nurse how to become an organ donor. What should the nurse include in
the discussion?
A. The client can donate by written consent.
B. A family member must witness the consent.
C. The donor must be older than 21 years of age.
D. A family member must be present when a client consents to organ donation.
CORRECT ANSWER: A.
The client has the right to donate her or his own organs for transplantation, and any person who
is 18 years of age or older may become an organ donor by written consent without the
permission or presence of the family. In the absence of suitable documentation, a family member
or legal guardian can authorize donation of the decedents organs.
18. A client asks the home care nurse to witness the clients signature on a living will with
the clients attorney in attendance. Which action should the nurse implement?
A. Decline to witness the signature on the living will.
B. Sign the living will as a witness to the signature only.
C. Notify the supervisor that a living will is being witnessed.
D. Sign the living will with identifying credentials and employment agency
CORRECT ANSWER: A.
Living wills are written documents and need to be signed by the client. The clients signature
either must be witnessed by non-agency individuals or notarized; thus, the nurse should decline

to sign the will to avoid a conflict of interest. There is no need to contact the supervisor or sign
the living will with or without credentials because the nurse cannot sign this document as a
witness. Therefore, options B, C, D are incorrect.
19. The nurse notes old and new ecchymotic areas on an older adult clients arms and
buttocks upon admission. The client tells the nurse in confidence that her family members
frequently hit her. Which statement should the nurse use in response?
A. I have a legal obligation to report this type of abuse.
B. Lets get these treated, and I will maintain confidence.
C. Lets talk about ways to prevent someone from hitting you.
D. If this happens again, you must call the emergency department.
CORRECT ANSWER: A.
The nurse should inform the client that nurses cannot maintain confidence about alleged abusive
behavior and that the nurse must report situations related to abuse. The nurse avoids bargaining
with the client about treatment to maintain a confidence that the nurse is legally bound to report.
Options C and D delay protective action and place the client at risk for future abuse.
20. After receiving detailed information about a colonoscopy from the health care provider
(HCP), the nurse asks the client to sign the informed consent form and discovers that the
client cannot write. Which is the best intervention for the nurse to implement?
A. Contact the provider to obtain informed consent.
B. Obtain a verbal informed consent from the client.
C. Have two nurses witness the client sign with an X.
D. Clarify information to the client with another nurse.
CORRECT ANSWER: C.
Nurses are responsible to make sure that the signed informed consent form is in the clients
medical record before a procedure and for clarifying facts that have already been presented by
the HCP. Nonetheless, the person performing the procedure obtains informed consent and
provides the explanations to the client. Informed consent can be obtained verbally, but that is
also the responsibility of the HCP. Clients who cannot write may sign an informed consent with
an X in the presence of two witnesses. Nurses can serve as a witness to the clients signature but
not to the fact that the client is informed.
21. An adult client who has a severe mental impairment is scheduled for gallbladder surgery.
With regard to the informed consent, which should the nurse implement first to facilitate
the scheduled surgery?
A. Check for the identity of the clients legal guardian.
B. Inform the legal guardian about advance directives.
C. Arrange for the surgeon to provide informed consent.
D. Ensure that the legal guardian signed the informed consent.
CORRECT ANSWER: A.
A mentally impaired client is not competent to sign an informed consent, so the nurse should first
verify the identity of the clients legal guardian. This action fulfills part of the nurses duty in

informed consent, helps avoid improperly signed documents, and directs the surgeon to the legal
representatives of the clients interests. The client and/or legal guardian is asked about the
existence of an advance directive at the time of admission, so this should have already been
done, making option B incorrect. The surgeon is responsible for obtaining the informed consent,
but based on the options provided, option C is not the first nursing action. Likewise, option D is
not the first action; the nurse checks the identity of the legal guardian first.
22. The nurse must place a wrist restraint on a client. The client tells the nurse that he does
not want to wear the restraint. Which is the best nursing action to implement at this time?
A. Sedate the client first.
B. Apply the wrist restraint.
C. Contact the clients family.
D. Reconsider alternative measures.
CORRECT ANSWER: D.
Before applying restraints, the nurse must exhaust alternative measures to restraints such as bed
alarm, distraction, and a sitter. If the nurse determines that a restraint is necessary, its use is
discussed with the client and family and a prescription is obtained from the healthcare provider.
The nurse should explain carefully to the client and family the indications for the restraint, the
type of restraint selected, and the anticipated duration for its use. Sedation can be considered as
a chemical restraint. The nurse avoids applying the restraint on a client who refused it to prevent
client coercion and future charges of battery.
23. A hospitalized client wants to leave the hospital before being discharged by the health
care provider (HCP). Which is the priority nursing intervention?
A. Notify the nursing supervisor of the clients plan to leave.
B. Ask the client about transportation plans from the hospital.
C. Arrange medication prescriptions at the clients preferred pharmacy.
D. Discuss the potential consequences of the plans for leaving with the client.
CORRECT ANSWER: A
The nurse notifies the nursing supervisor of the clients plan to leave without the health care
providers approval to ensure client safety and to help the nurse manage the situation. This will
help the nurse manage the situation in a thoughtful, comprehensive manner and complete
nursing interventions that include asking about transportation, arranging medication
prescriptions, and discussing the risks and benefits of leaving or remaining in the hospital. The
HCP should be contacted and the client encouraged to remain until the HCP arrives. The nurse
avoids coercion, restraint, or security measures meant to prohibit the clients exit to prevent
claims of false imprisonment.
24. The nurse administers digoxin (Lanoxin) 0.25mg by mouth rather than the prescribed
dose of 0.125mg to the client. Which should the nurse implement first?
A. Write an incident report.
B. Tell the client about the medication error.
C. Administer digoxin immune Fab (Digibind).
D. Tell the client about the adverse effects of digoxin.

CORRECT ANSWER: A
According to agency policy, the nurse should file an incident report when a medication error
occurs to accurately document the facts. The nurse should assess the client first and then contact
the health care provider (HCP) because in this situation the client received too much medication.
The client should be informed of the error and the adverse effects in a professional manner to
avoid alarm and concern. However, in many situations, the HCP prefers to discuss this with the
client. Digoxin immune Fab is reserved for extreme toxicity and requires a prescription and may
be prescribed depending on the clients response and the serum digoxin level.
25. The nurse is working in the emergency department of a small local hospital when a client
with multiple stab wounds arrives by ambulance. Which action by the nurse is
contraindicated when handling legal evidence?
A. Initiate a chain of custody log.
B. Give clothing and wallet to the family.
C. Cut clothing along seams, avoiding stab holes.
D. Place personal belongings in a labeled, sealed paper bag.
CORRECT ANSWER: B
Potential evidence is never released to the family to take home. Basic rules for handling
evidence include initiating a chain of custody log to track handling and movement of evidence,
limiting the number of people with access to the evidence, and carefully removing clothing and
placing personal belongings in a labeled, sealed paper bag to avoid destroying the evidence.
This also usually includes cutting clothes among seams, while avoiding areas where there are
obvious holes or tears.

COMMUNICABLE
DISEASES

1. A child is seen in the health care clinic, and testing for human immunodeficiency virus
(HIV) is performed because of the childs exposure. Which home care instruction should
the nurse provide to the parents of the child?
A. Avoid sharing toothbrushes
B. Avoid all immunizations until the diagnosis is established
C. Wipe up any blood spills with a rag, and allow them to air dry
D. Wash your hands with half-strength bleach if they come in contact with the childs
blood
CORRECT ANSWER: A
Parents are instructed that toothbrushes are not to be shared. Immunizations must be kept up to
date. Blood spills are wiped up with a paper towel; the area is then washed with soap and water,
rinsed with bleach and water, and allowed to air dry. Hands are washed with soap and water if
they come in contact with blood.
2. During the initial interview, the client reports that she has a lesion on the perineum.
Further investigation reveals a small blister on the vulva that is painful to touch. The
nurse is aware that the most likely source of the lesion is:
A. Syphilis
B. Herpes
C. Gonorrhea
D. Condylomata
CORRECT ANSWER: B.
A lesion that is painful is most likely a herpetic lesion. A chancre lesion associated with syphilis
is not painful, so answer A is incorrect. Condylomata lesions are painless warts, so answer D is
incorrect. In answer C, gonorrhea does not present as a lesion, but is exhibited by a yellow
discharge.
3. A client visiting a family planning clinic is suspected of having an STI. The best
diagnostic test for treponema pallidum is:
A. Venereal Disease Research Lab (VDRL)
B. Rapid plasma reagin (RPR)
C. Fluorescent treponemal antibody (FTA)
D. Thayer-Martin culture (TMC)
CORRECT ANSWER: C.
Fluorescent treponemal antibody (FTA) is the test for treponema pallidum. VDRL and RPR are
screening tests done for syphilis, so answers A and B are incorrect. The Thayer-Martin culture is
done for gonorrhea, so answer D is incorrect.
4. Which action by the nurse indicates understanding of herpes zoster?
A. The nurse covers the lesions with a sterile dressing.
B. The nurse wears gloves when providing care.
C. The nurse administers a prescribed antibiotic.
D. The nurse administers oxygen.

CORRECT ANSWER: B.
Herpes zoster is shingles. Clients with shingles should be placed in contact precautions. Wearing
gloves during care will prevent transmission of the virus. Covering the lesions with a sterile
gauze is not necessary, antibiotics are not prescribed for herpes zoster, and oxygen is not
necessary for shingles; therefore, answers A, C, and D are incorrect
5. A client with AIDS is taking Zovirax (acyclovir). Which nursing intervention is most
critical during the administration of acyclovir?
A. Limit the clients activity.
B. Encourage a high-carbohydrate diet.
C. Use an incentive spirometer to improve respiratory function.
D. Encourage fluids.
CORRECT ANSWER: D.
Clients taking Acyclovir should be encouraged to drink plenty of fluids because renal
impairment can occur. Limiting activity is not necessary, nor is eating a high-carbohydrate diet.
Use of an incentive spirometer is not specific to clients taking Acyclovir; therefore, answers A, B,
and C are incorrect.
6. A nursing instructor asks a nursing student to describe live or attenuated vaccines. What
should the student tell the instructor about these types of vaccines?
A. Live or attenuated vaccines contain bacterial toxins that have been made inactive
by either chemicals or heat
B. Live or attenuated vaccines contain pathogens made inactive by either chemicals
or heat
C. Live or attenuated vaccines have their virulence (potency) diminished so as to not
produce a full-blown clinical illness
D. Live or attenuated vaccines have been obtained from the pooled blood of mant
people and provide antibodies to a variety of diseases
CORRECT ANSWER: C
Live or attenuated vaccines have their virulence (potency) diminished so as to not produce a
full-blown clinical illness. In response to vaccination, the body produces antibodies and causes
immunity to be established. Option A identifies toxoids. Option B identifies killed or inactivated
vaccines. Option D identifies human immune globulin.
7. A 23-year-old client in the AIDS clinic asks the nurse what he should do for the multiple
small, painless, purplish-brown spots on his right leg and ankle. The nurse should instruct
the client to
A. Clean the spots carefully with soap and warm water twice a week and cover them
with a sterile dressing.
B. Clean the lesions twice a day with a diluted solution of povidone-iodine
(Betadine) and leave them open to the air.
C. Shower daily using a mild soap from a pump dispenser and pat the skin dry.
D. Soak in a warm tub three times a day and rub the spots with a washcloth.

CORRECT ANSWER: C.
(A) if lesions are open and draining they must be cleaned and dressed daily to prevent secondary
infection (B) treatment for herpes simplex virus abscess, not Kaposis sarcoma (C) correct
important to keep the skin clean and prevent secondary skin infection (D) increases risk of
secondary skin infection
8. A nurse is assessing a clinic patient with a diagnosis of hepatitis A. Which of
the following is the most likely route of transmission?
A. Sexual contact with an infected partner.
B. Contaminated food.
C. Blood transfusion.
D. Illegal drug use.

CORRECT ANSWER: B.
Hepatitis A is the only type that is transmitted by the fecal-oral route through
contaminated food. Hepatitis B, C, and D are transmitted through infected bodily
fluids.

9. A nonimmunized child appears at the clinic with a visible rash. Which of the
following observations indicates the child may have rubeola (measles)?
A. Small blue-white spots are visible on the oral mucosa.
B. The rash begins on the trunk and spreads outward.
C. There is low-grade fever.
D. The lesions have a "tear drop on a rose petal" appearance.

CORRECT ANSWER: A.
Koplik's spots are small blue-white spots visible on the oral mucosa and are

characteristic of measles infection. The body rash typically begins on the face and
travels downward. High fever is often present. "Tear drop on a rose petal" refers to
the lesions found in varicella (chicken pox).

10.A child is seen in the emergency department for scarlet fever. Which of the
following descriptions of scarlet fever is NOT correct?
A. Scarlet fever is caused by infection with group A Streptococcus
bacteria.
B. "Strawberry tongue" is a characteristic sign.
C. Petechiae occur on the soft palate.
D. The pharynx is red and swollen.

CORRECT ANSWER: C.
Petechiae on the soft palate are characteristic of rubella infection. Choices A, B, and
D are characteristic of scarlet fever, a result of group A Streptococcus infection.

11. A child is diagnosed with scarlet fever. The nurse assesses the child knowing that which
of the following is not a clinical manifestation associated with this disease?
A. Pastia's sign
B. White strawberry tongue
C. Edematous and beefy, red colored pharynx
D. Koplik spots
CORRECT ANSWER: D.
Koplik spots are associated with rubeola. Pastia's sign describes a rash that is seen in scarlet
fever that will blanch with pressure except in areas of deep creases and folds of the joints. The
tongue is initially coated with a white furry covering with red projecting papillae
(white strawberry tongue). The pharynx is edematous and beefy red in color.
12. A nurse is preparing the plan of care for a patient with herpes genitalis. What would be
the priority nursing diagnosis for the patient?
A. Disturbed Sleep pattern
B. Imbalance in Nutrition: Less than Body Requirements
C. Alteration in Comfort: Pain
D. Ineffective Breathing Pattern

CORRECT ANSWER IS C.
Genital herpes is characterized by painful vesicles affecting the external genitalia, vagina and
cervix.
13. The nurse provides home care instructions to the mother of a child with chickenpox about
preventing the transmission of the virus. Which instruction should the nurse include?
A. Isolate the child until the vesicles have dried and crusted
B. Ensure that the child uses a separate bathroom for elimination
C. Bring all household members to the clinic immediately for varicella vaccine
D. Ask the health care provider for a prescription of antibiotics for all household
members
CORRECT ANSWER: A
Chickenpox is caused by the varicella-zoster virus. The communicable period is from 1-2 days
before the onset of the rash to 6 days after the first crop of vesicles, when crusts have formed.
Transmission occurs by direct contact with secretions from the vesicles or contaminated objects,
and via respiratory tract secretions. It is not transmitted via urine or feces. The recommended
preventative schedule for receiving varicella vaccine is at 12 to 15 months of age (first dose) and
4 to 6 years of age (second dose). It is not administered at the time of exposure to the virus.
Antibiotics are not used to treat a viral infection.
14. A 5-year-old child is hospitalized with Rocky Mountain spotted fever (RMSF). The
nursing assessment reveals that the child was bitten by a tick 2 weeks ago. The child
presents with a complaint of headache, fever and anorexia, and the nurse notes a rash on
the palms of the hands and soles of the feet. The nurse reviews the health care providers
prescriptions and anticipates that which medication will be prescribed?
A. Ganciclovir
B. Amantadine (Symmetrel)
C. Doxycycline (Vibramycin)
D. Amphotericin B (Fungizone)
CORRECT ANSWER: C
The nursing care of a child with RMSF includes the administration of doxycycline. An
alternative medication is chloramphenicol. Ganciclovir is used to treat cytomegalovirus.
Amantadine is used to treat Parkinsons disease. Amphotericin B is used for fungal infections.
15. The pediatric nurse specialist provides an educational session to the nursing students
about childhood communicable disease, mumps. The pediatric nurse informs the students
that which clinical manifestation is indicative of the most common complication of this
communicable disease?
A. Pain
B. Deafness
C. Nuchal rigidity
D. A red swollen testicle

CORRECT ANSWER: C
The most common complication of mumps is aseptic meningitis, with the virus being identified in
the cerebrospinal fluid. Common signs include nuchal rigidity, lethargy and vomiting. Muscular
pain, parotid pain and testicular pain may occur, but pain does not indicate a sign of common
complication. Although mumps is one of the cause of unilateral nerve deafness, it does not occur
frequently. A red swollen testicle may be indicative of orchitis. Although this complication
appears to cause most concern among parents, it is not the most common complication. Swollen
and tender salivary glands under the ears on one or both sides of the head (parotitis) is the most
common sign.
16. Which among the following information is true of HIV?
A. It can be transmitted via body fluids such as blood, semen, urine, and
perspiration.
B. Blood, semen, and breast milk have higher concentrations of HIV than urine,
saliva, vomitus and stool
C. A client who by history may be exposed to HIV but test negative for HIV
antibodies can no longer infect others
D. Enzyme-linked immunosorbent assay (ELISA) is the confirmatory test for HIV
CORRECT ANSWER: B.
(A)Perspiration does not contain the virus. (C) This is not always true for HIV. Clients may be
infected with the virus, but has not yet produced antibodies, thereby testing negative, but being
capable of infecting others. It usually takes 6 to 12 weeks (other books: 1 to 3 months) for a host
to manufacture detectable HIV antibodies. (D) Western blot is the confirmatory test for HIV.
17. The following are preventions of gonorrhea transmission EXCEPT:
A. Sex education
B. Case finding
C. Incidence to be reported to health authorities
D. Administration of ophthalmic prophylaxis, as ordered
CORRECT ANSWER: D.
This does not prevent transmission of gonorrhea. It prevents the occurrence of ophthalmia
neonatorum.

18.Jessie, a young man with newly diagnosed acquired immune deficiency


syndrome (AIDS) is being discharged from the hospital. The nurse knows
that teaching regarding prevention of AIDS transmission has been
effective when the client:
A. Verbalizes the role of sexual activity in spread of the disorder.
B. States he will make arrangements to drop his college classes.

C. Acknowledges the need to avoid all contact sports.


D. Says he will avoid close contact with his three-year-old niece.

CORRECT ANSWER: A.
The AIDS virus is spread through direct contact with body fluids such as blood and through
sexual intercourse. Casual contact with other people does not pose a risk of transmission of
AIDS. Unless the client is feeling very ill, there is no need for him to drop his college classes.
Contact sports are not contraindicated unless there is a significant chance of bleeding and direct
contact with others. Casual contact with other people does not pose a risk of transmission of
AIDS. There is no need to limit casual contact with children.
19.Which question is least useful in the assessment of a client with AIDS?
A. Are you a drug user?
B. Do you have many sex partners?
C. What is your method of birth control?
D. How old were you when you became sexually active?

CORRECT ANSWER: D.
Drug use is a risk factor for AIDS. Multiple sex partners is a risk factor for AIDS. Birth control
methods are important to prevent a baby from being born with the AIDS virus. The age at which
sexual activity began it not relevant as it does not usually provide information that identifies the
presence of risk factors for AIDS.
20. A client with acquired immunodeficiency syndrome (AIDS) gets recurrent Candida
infections of the mouth (thrush). The nurse has given the client instructions to minimize
the occurrence of thrush and determines that the client understands the instructions if
which statement is made by the client?
A. I should use warm saline or water to rinse my mouth.
B. I should use a mouthwash at least once a week.
C. I should brush my teeth and rinse my mouth once a day.
D. Increasing the amount of red meat in my diet will keep this from recurring.
CORRECT ANSWER: A.
When a client is in a state of immunosuppression or has decreased levels of some normal oral
flora, an overgrowth of the normal flora Candida can occur. Careful routine mouth care is
helpful to prevent the recurrence of Candida infections. The client should use a mouthwash that
consists of warm saline or water. The time frames given for oral hygiene in options B and C are
too infrequent. Red meat will not prevent thrush.

21. The nurse provides home care instructions to the parents of a child with pertussis who is
in the convalescent stage and is being prepared for discharge. Which statement by a
parent indicates a need for further evaluation?
A. We need to encourage our child to increase fluids.
B. Coughing spells may be triggerred by dust or smoke.
C. Vomiting may occur when our child has coughing episodes.
D. We need to maintain droplet precautions and a quiet environment for at least 2
weeks.
CORRECT ANSWER: D.
Pertussis is transmitted by direct contact or respiratory droplets from coughing. The
communicable period occurs primarily during the catarrhal stage. Respiratory precautions are
not required during the convalescent phase. Options A, B, C are accurate components of home
care instructions.
22. A child is receiving a series of hepatitis B vaccine and arrives at the clinic with his parent
for the second dose. Before administering the vaccine, the nurse should ask the child and
parent about a history of severe allergy to which substance?
A. Eggs
B. Penicillin
C. Sulfonamides
D. A previous dose of hepatitis B vaccine or component
CORRECT ANSWER: D.
A contraindication to receiving the hepatitis B vaccine is a previous anaphylactic reaction to a
previous dose of hepatitis B vaccine or component (aluminum hydroxide or yeast protein) of the
vaccine. An allergy to eggs, penicillin and sulfonamides is unrelated to the contraindication to
receiving this vaccine.
23. The clinic nurse prepares to administer a measles, mumps, rubella (MMR) vaccine to a 5year-old child. The nurse should administer this vaccine by which best route and in which
best site?
A. Subcutaneously in the gluteal muscle.
B. Intramuscularly in the deltoid muscle.
C. Subcutaneously in the outer aspect of the upper arm.
D. Intramuscularly in the anterolateral aspect of the thigh
CORRECT ANSWER: C.
Measles, mumps, rubella (MMR) vaccine is administered subcutaneously in the outer aspect of
the upper arm. The gluteal muscle is not recommended for injections. MMR vaccine is not
administered by the intramuscular route.
24. A child with rubeola (measles) is being admitted in the hospital. In preparing for the
admission of the child, the nurse should plan to place the child on which precautions?
A. Enteric
B. Airborne

C. Protective
D. Neutropenic
CORRECT ANSWER: B.
Rubeola is transmitted via airborne particles or direct contact with infectious droplets. Airborne
droplet precautions are required, and persons in contact with the child should wear masks. The
child is placed in a private room if hospitalized, and the hospital room door remains closed.
Gowns and gloves are unnecessary, but standard precautions are used. Articles that are
contaminated should be bagged and labeled. Special enteric precautions and protective
(neutropenic) isolation are not indicated in rubeola.
25. The clinic nurse is assessing a child who is scheduled to receive a live virus vaccine.
What are the general contraindications associated with receiving a live virus vaccine?
Select all that apply.
A. The child has symptoms of a cold.
B. The child had a previous anaphylactic reaction to the vaccine.
C. Mother reports that the child has not had an appetite and has been fussy.
D. The child has a disorder that caused a severely deficient immune system.
E. Mother reports that the child has recently been exposed to an infectious disease.
CORRECT ANSWERS: B, E.
The general contraindications for receiving live virus vaccines include a previous anaphylactic
reaction to a vaccine or component of a vaccine. In addition, live virus vaccines generally are
not administered to individuals with a severely deficient immune system, individuals with a
severe sensitivity to gelatin, or pregnant women. A vaccine is administered with caution to an
individual with a moderate or severe acute illness, with or without fever. Options A, C, D, F are
not contraindications to receiving a vaccine.

PERIOPERATIVE
NURSING

1. The nurse has just reassessed the condition of a postoperative client who was admitted 1
hour ago to the surgical unit. The nurse plans to monitor which parameter most carefully
during the next hour?
A. Urinary output of 20mL/hour
B. Temperature of 37.6C (99.6F)
C. Blood pressure of 100/70mm Hg
D. Serous drainage on the surgical dressing
CORRECT ANSWER: A
Urine output should be maintained at a minimum of 30mL/hour for an adult. An output of less
than 30mL for each of 2 consecutive hours should be reported to the health care provider. A
temperature higher than 37.7C (100F) or lower than 36.1C (97F) and a falling systolic
blood pressure, lower than 90mm Hg, are usually considered reportable immediately. The
clients preoperative or baseline blood pressure is used to make informed postoperative
comparisons. Moderate or light serous drainage from the surgical site is considered normal.
2. A postoperative client asks the nurse why it is so important to deep-breathe and cough
after surgery. When formulating a response, the nurse incorporates the understanding that
retained pulmonary secretions in a postoperative client can lead to which condition?
A. Pneumonia
B. Hypoxemia
C. Fluid imbalance
D. Pulmonary embolism
CORRECT ANSWER: A
Postoperative respiratory problems are atelectasis, pneumonia, and pulmonary emboli.
Pneumonia is the inflammation of lung tissue that causes productive cough, dyspnea, and lung
crackles and can be caused by retained pulmonary secretions. Hypoxemia is an inadequate
concentration of oxygen in arterial blood. Fluid imbalance can be a deficit or excess related to
fluid loss or overload. Pulmonary embolus occurs as a result of a blockage of the pulmonary
artery that disrupts blood flow to one or more lobes of the lung; this is usually due to clot
formation.
3. The nurse is developing a plan of care for a client scheduled for surgery. The nurse
should include which activity in the nursing care plan for the client on the day of surgery?
A. Avoid oral hygiene and rinsing with mouthwash.
B. Verify that the client has not eaten for the last 24 hours.
C. Have the client void immediately before going into surgery.
D. Report immediately any slight increase in blood pressure or pulse.
CORRECT ANSWER: C
The nurse would assist the client to void immediately before surgery so that the bladder will be
empty. Oral hygiene is allowed, but the client should not swallow any water. The client usually
has a restriction of food and fluids for 6 to 8 hours before surgery instead of 24 hours. A slight
increase in blood pressure and pulse is common during the postoperative period and is usually
the result of anxiety.

4. A client with perforated gastric ulcer is scheduled for surgery. The client cannot sign the
operative consent form because of sedation from opioid analgesics that have been
administered. The nurse should take which most appropriate action in the care of this
client?
A. Obtain a court order for the surgery.
B. Have the charge nurse sign the informed consent immediately.
C. Send the client to surgery without the consent form being signed.
D. Obtain a telephone consent from a family member, following agency policy.
CORRECT ANSWER: D
Every effort should be made to obtain permission from a responsible family member to perform
surgery if the client is unable to sign the consent form. A telephone consent must be witnessed by
two persons who hear the family members oral consent. The two witnesses then sign the consent
with the name of the family member, noting that an oral consent was obtained. Consent is not
informed if it is obtained from a client who is confused, unconscious, mentally incompetent, or
under the influence of sedatives. In an emergency, a client may be unable to sign and family
members may not be available. In this situation, a health care provider is permitted legally to
perform surgery without consent, but in this case it is not an emergency. Options A and C are not
appropriate in this situation. Also, agency policies regarding informed consent should always be
followed.
5. A preoperative client expresses anxiety to the nurse about upcoming surgery. Which
response by the nurse is most likely to stimulate further discussion between the client and
the nurse?
A. If its any help, everyone is nervous before surgery.
B. I will be happy to explain the entire surgical procedure to you.
C. Can you share with me what youve been told about your surgery?
D. Let me tell you about the care youll receive after surgery and the amount of pain
you can anticipate.
CORRECT ANSWER: C
Explanations should begin with the information that the client knows. By providing the client
with individualized explanations of care and procedures, the nurse can assist the client in
handling anxiety and fear for a smooth preoperative experience. Clients who are calm and
emotionally prepared for surgery withstand anesthesia better and experience fewer
postoperative complications. Option A does not focus on the clients anxiety. Explaining the
entire surgical procedure may increase the clients anxiety. Option D avoids the clients anxiety
and is focused on postoperative care.
6. The nurse is conducting preoperative teaching with a client about the use of an incentive
spirometer. The nurse should include which piece of information in discussions with the
client?
A. Inhale as rapidly as possible.
B. Keep a lose seal between the lips and the mouthpiece.
C. After maximum inspiration, hold the breath for 15 seconds and exhale.

D. The best results are achieved when sitting up or with the head of the bed elevated
45 to 90 degrees.
CORRECT ANSWER: D
For optimal lung expansion with the incentive spirometer, the client should assume the semiFowlers or high Fowlers position. The mouthpiece should be covered completely and tightly
while the client inhales slowly, with a constant flow through the unit. The breath should be held
for 5 seconds before exhaling slowly.
7. The nurse has conducted preoperative teaching for a client scheduled for surgery in 1
week. The client has a history of arthritis and has been taking acetylsalicylic acid
(aspirin). The nurse determines that the client needs additional teaching if the client
makes which statement?
A. Aspirin can cause bleeding after surgery.
B. Aspirin can cause my ability to clot blood to be abnormal.
C. I need to continue to take aspirin until the day of surgery.
D. I need to check with my health care provider about the need to stop the aspirin
before the scheduled surgery.
CORRECT ANSWER: C
Anticoagulants alter normal clotting factors and increase the risk of bleeding after surgery.
Aspirin has properties that can alter the clotting mechanism and should be discontinued at least
48 hours before surgery. However, the client should always check with his or her health care
provider regarding when to stop taking the aspirin when a surgical procedure is scheduled.
Options A, B, D are accurate client statements.
8. The nurse assesses a clients surgical incision for signs of infection. Which finding by the
nurse would be interpreted as a normal finding at the surgical site?
A. Red, hard skin
B. Serous drainage
C. Purulent drainage
D. Warm, tender skin
CORRECT ANSWER: B
Serous drainage is an expected finding at a surgical site. The other options indicate signs of
wound infection. Signs and symptoms of infection include warm, red, and tender skin around the
incision. Wound infection usually appears 3 to 6 days after surgery. The client also may have
fever and chills. Purulent material may exit from drains or from separated wound edges.
Infection may be caused by poor aseptic technique or a contaminated wound before surgical
exploration; existing client conditions such as diabetes mellitus or immunocompromised may
place the client at risk.
9. The nurse is monitoring the status of a postoperative client. The nurse would become
most concerned with which sign that could indicate an evolving complication?
A. Increasing restlessness
B. A pulse of 86 beats/minute

C. Blood pressure of 110/70 mm Hg


D. Hypoactive bowel sounds in all four quadrants
CORRECT ANSWER: A
Increasing restlessness is a sign that requires continuous and close monitoring because it could
indicate a potential complication, such as hemorrhage, shock, or pulmonary embolism. A blood
pressure of 110/70 mmHg with a pulse of 86 beats/minute is within normal limits. Hypoactive
bowel sounds heard in all four quadrants are a normal occurrence.
10. A client who has had abdominal surgery complains of feeling as though something gave
way in the incisional site. The nurse removes the dressing and notes the presence of a
loop of bowel protruding through the incision. Which nursing interventions should the
nurse take? Select all that apply.
A. Contact the surgeon.
B. Instruct the client to remain quiet.
C. Prepare the client for wound closure.
D. Document the findings and actions taken.
E. Place a sterile saline dressing and ice packs over the wound.
F. Place the client in a supine position without a pillow under the head.
CORRECT ANSWER: A, B, C, D
Wound dehiscence is the separation of the wound edges. Wound evisceration is protrusion of the
internal organs through an incision. If wound dehiscence or evisceration occurs, the nurse
should call for help, stay with the client, and ask another nurse to contact the surgeon and
obtain needed supplies to care for the client. The nurse places the client in a low Fowlers
position, and the client is kept quiet, and instructed not to cough. Protruding organs are covered
with a sterile saline dressing. Ice is not applied because of its vasoconstrictive effect. The
treatment for evisceration is usually immediate wound closure under local or general anesthesia.
The nurse also documents the findings and actions taken.
11. A client who has undergone preadmission testing has had blood drawn for serum
laboratory studies, including a complete blood count, coagulation studies, and
electrolytes and creatinine levels. Which laboratory result should be reported to the
surgeons office by the nurse, knowing that it could cause surgery to be postponed?
A. Sodium, 141 mEq/L
B. Hemoglobin, 8.0 g/dL
C. Platelets 210,000/mm
D. Serum creatinine, 0.8 mg/dL
CORRECT ANSWER: B
Routine screening tests include a complete blood count, serum electrolyte analysis, coagulation
studies, and a serum creatinine test. The complete blood count includes the hemoglobin analysis.
All these values are within the normal range except for hemoglobin. If a client has a low
hemoglobin level, the surgery likely could be postponed by the surgeon.
12. The client tells the nurse during the preoperative history that he is a three-pack a day

cigarette smoker. This information alerts the nurse to which potential complication during
the intraoperative and postoperative periods?
A. A decreased tolerance to pain
B. A decreased clotting ability
C. An increased risk for atelectasis and hypoxia
D. An increased risk for excessive scar tissue formation
CORRECT ANSWER: C
Smoking increases the level of circulating carboxyhemoglobin, which decreases oxygen delivery
to the tissues. In addition, cigarette smoking damages the cilia of mucous membranes,
decreasing transport of secretions and increasing the risk of pulmonary infection and atelectasis.
13. Twenty minutes after the client has received a preoperative injection of atropine and
midazolam (Versed), the client tells the nurse that he must be allergic to the medication
because his mouth is dry and his heart seems to be beating faster than normal. What is the
nurse's best first action?
A. Document the findings as the only action.
B. Check the client's pulse and blood pressure.
C. Prepare to administer epinephrine and diphenhydramine (Benadryl).
D. Explain to the client that these symptoms are normal responses to the medication.
CORRECT ANSWER: B
Although these are the expected physiologic responses to the preoperative medication, any time
the client states that he or she can feel a change in normal cardiac function, the system should
be assessed. If the client's pulse and blood pressure are within normal limits, the nurse should.
14. Which nursing action or statement is most likely to reduce anxiety in a client being
brought to the surgical suite?
A. Asking the client if he or she has talked with the hospital chaplain
B. Asking the client what specific surgery he or she is having done today
C. Asking the client if he or she wants family members to be with them in the
holding area
D. Explaining to the client that the surgical area is the most technologically advanced
in the city
CORRECT ANSWER: C
Most anxious clients would feel some relief by having one or more familiar persons waiting with
them until surgery begins. In addition, asking the client what he or she wants allows the client to
have more control over the situation. Asking the client if he or she has visited with the hospital
chaplain and telling the client about the advanced technology can imply to the client that the
procedure is dangerous. Although the client must be asked what procedure he or she is having
(to ascertain that the client does know what is to be done), this question may make the client
worry about the competency of the staff.
15. In the operating room, the client tells the circulating nurse that he is going to have the
cataract in his left eye removed. The nurse notes that the consent form indicates that

surgery is to be performed on the right eye. What is the nurse's best first action?
A. Assume that the client is a little confused because he is older and has received
midazolam intramuscularly.
B. Check to see if the client has received any preoperative medications.
C. Notify the surgeon and anesthesiologist.
D. Ask the client his name.
CORRECT ANSWER: D
Ensuring proper identification of the client is a responsibility of all members of the surgical
team. Especially in a specialty surgical setting, where many people undergo the same type of
surgery each day, such as cataract removal, it is possible that the client and the record do not
match. The nurse identifies the client and the client's consent form before the physicians are
notified.
16. Which client is at greatest risk for respiratory complications after surgery under general
anesthesia?
A. 65-year-old woman taking a calcium channel blocker for hypertension
B. 55-year-old man with chronic allergic rhinitis
C. 45-year-old woman with diabetes mellitus type 1
D. 35-year-old man who smokes two packs of cigarettes daily
CORRECT ANSWER: D
Cigarette smoking greatly increases the risk for pulmonary problems following general
anesthesia because the cilia of the mucous membranes may be absent or hypoactive, the lining of
the airways may be hypertrophied, and the alveoli may be less compliant. Age and gender are
not significant in this case.
17. The client who is 24 hours postoperative from abdominal surgery has light brown fluid
with small particles that look like coffee grounds in the NG tube drainage. What is the
nurse's best action?
A. Notify the physician.
B. Irrigate the tube with normal saline.
C. Clamp the tube and advance it 1 to 2 inches.
D. Document the finding as the only action.
CORRECT ANSWER: A
This type of drainage indicates possible gastrointestinal bleeding and should be explored further
as soon as possible.
18. The nurse is caring for a client after a supratentorial craniotomy. The nurse places a sign
above the clients bed stating that the client should be maintained in which position?
A. Prone
B. Supine
C. Semi-Fowlers
D. Dorsal recumbent
CORRECT ANSWER: C

After supratentorial surgery, the clients head is usually elevated 30 degrees to promote venous
outflow through the jugular veins and modulate intracranial pressure (ICP). Options A, B, D are
incorrect positions after this surgery because they are likely to increase ICP.
19. The nurse instructs a preoperative client in the proper use of an incentive spirometer.
What should the nurse use to determine that the client is using thee incentive spirometer
effectively?
A. Cloudy sputum
B. Shallow breathing
C. Unilateral wheezing
D. Productive coughing
CORRECT ANSWER: D
Incentive spirometry helps reduce atelectasis, open airways, stimulate coughing, and help
mobilize secretions for expectoration, via vital client participation in recovery. Cloudy sputum,
shallow breathing, and wheezing indicate that the incentive spirometry is not effective because
they point to infection, counterproductive depth of breathing, and bronchoconstriction,
respectively.
20. The nurse is encouraging the client to cough and deep breathe after cardiac surgery. The
nurse ensures that which item is available to maximize the effectiveness of this
procedure?
A. Nebulizer
B. Ambu bag
C. Suction equipment
D. Incisional splinting pillow
CORRECT ANSWER: D
The use of an incisional splint such as a cough pillow can ease discomfort during coughing
and deep breathing. The client who is comfortable will do more effective deep breathing and
coughing exercises. Use of an incentive spirometer is also indicated. Options A, B, C will not
encourage the client to cough and deep breathe.
21. A client being seen in the health care providers office for follow-up 2 weeks after
pneumonectomy complains of numbness and tenderness at the surgical site. Which
statement should the nurse make to the client?
A. This is not likely to be permanent, but may last for some months.
B. You are having a severe problem and will probably be hospitalized.
C. This is probably caused by permanent nerve damage as a result of surgery.
D. This is often the first sign of a wound infection, and I will check your
temperature.
CORRECT ANSWER: A
Clients who undergo pneumonectomy or other surgical procedures may experience numbness,
altered sensation, or tenderness in the area that surrounds the incision. These sensations may
last for months. It is not considered to be a severe problem and is not indicative of a wound

infection.
22. The nurse teacher a preoperative client about the nasogastric tube that will be inserted in
preparation for surgery. The nurse determines that the client understands when the tube
will be removed during the postoperative period based on which statement by the client?
A. When the doctor says so.
B. When I can tolerate food without vomiting.
C. When my gastrointestinal system is healed.
D. When my bowels begin to function again and I begin to pass gas.
CORRECT ANSWER: D
NG tubes are discontinued when normal function returns to the GI tract. Although the health
care provider determines when the NG tube will be removed, option A does not determine the
effectiveness of teaching. Food would not be administered unless bowel function returns. The
tube will be removed well before GI healing occurs.
23. The nurse prepares a client for a parathyroidectomy when the client states, I guess Ill
have to wear a scarf after this surgery. Considering this statement which problem is
appropriate for this client?
A. Denial that the surgery is necessary
B. Trouble coping with the need for surgery
C. Issues with potential changes to body image
D. Anxiety about the scar the surgery will cause
CORRECT ANSWER: C
The clients statement reflects a psychosocial concern regarding his or her appearance after
surgery, so option C is the correct option. The remaining options identify unsuitable problems
that are not supported by the provided client data.
24. A client is scheduled to have surgery. The nurse should place priority on determining
whether the surgeon wants which medications held in the preoperative period?
A. Furosemide (Lasix)
B. Famotidine (Pepcid)
C. Warfarin (Coumadin)
D. Multivitamins with minerals
CORRECT ANSWER: C
The nurse is careful to question the surgeon about whether warfarin should be administered in
the preoperative period. This medication is often withheld for a period of time preoperatively to
minimize the risk of hemorrhage during surgery. The pother medications may also be withheld if
specifically prescribed, but usually they are discontinued as part of an NPO after midnight
prescription.
25. A postoperative client displays signs of anxiety when the nurse explains that the
intravenous (IV) line will need to be discontinued as a result of an infiltration. Which
appropriate statement should the nurse make to the client?

A. This will be a totally painless experience. It is nothing to worry about.


B. Im sure it will be a relief for you just as soon as I discontinue this IV for good.
C. Just relax and take a deep breath. This procedure will not take long, and it will be
over soon.
D. I can see that youre anxious. Removal of the IV shouldnt be painful, but the IV
will need to be restarted in another location.
CORRECT ANSWER: D
Option D addresses the clients anxiety and honestly informs the client that the IV may need to be
restarted. This option uses the therapeutic technique of giving information, and it also
acknowledges the clients feelings. Although discontinuing an IV is a painless experience, it is
not therapeutic to tell a client not to worry. Option B does not acknowledge the clients feelings,
and it does not tell the client that an infiltrated IV may need to be restarted. Option C does not
address the clients feelings.

GERIATRICS

1. The nurse is performing an assessment on an older client who is having difficulty


sleeping at night. Which statement by the client indicates the need for further teaching
regarding measures to improve sleep?
A. I swim three times a week.
B. I have stopped smoking cigars.
C. I drink hot chocolate before bedtime.
D. I read for 40 minutes before bedtime.
CORRECT ANSWER: C
Many nonpharmacological sleep aids can be used to influence sleep. However, the client should
avoid caffeinated beverages and stimulants such as tea, cola and chocolate. The client should
exercise regularly because exercise promotes sleep by burning off tension that accumulates
during the day. A 20 to 30-minute walk, swim, or bicycle ride three times a week is helpful. The
client should sleep on a bed with a firm mattress. Smoking and alcohol should be avoided. The
client should avoid large meals, peanuts, beans, fruit, raw vegetables, and other foods that
produce gas; and snacks that are high in fat because they are difficult to digest.
2. The nurse is providing medication instructions to an older client who is taking digoxin
(Lanoxin) daily. The nurse notes that which age-related bidy change could place the
client at risk for digoxin toxicity?
A. Decreased muscle strength and loss of bone density
B. Decreased cough efficiency and decreased vital capacity
C. Decreased salivation and decreased gastrointestinal motility
D. Decreased lean body mass and decreased glomerular filtration rate
CORRECT ANSWER: D
The older client is at risk for medication toxicity because of decreased lean body mass and ageassociated decreased glomerular filtration rate. Although options A, B and C identify agerelated changes that occur in the older client, they are not associated with digoxin toxicity.
3. The nurse is caring for an older client in a long-term care facility. Which action
contributes to encouraging autonomy in the client?
A. Planning meals
B. Decorating the room
C. Scheduling haircut appointments
D. Allowing the client to choose social activities
CORRECT ANSWER: D
Autonomy is the personal freedom to direct ones own life as long as it does not impinge on the
rights of others.an autonomous person is capable of rational thought. This individual can
identify problems, search for alternatives, and select solutions that allow continued personal
freedom as long as others and their rights and property are not harmed. Loss of autonomy, and
therefore independence, is a real fear of older clients. The correct option is the only one that
allows the client to be a decision maker.
4. The nurse is performing an assessment on an older adult client. Which assessment data
would indicate a potential complication associated with the skin?

A.
B.
C.
D.

Crusting
Wrinkling
Deepening of expression lines
Thinning and loss of elasticity of the skin

CORRECT ANSWER: A
The normal physiological changes that occur in the skin of older adults include thinning of the
skin, loss of elasticity, deepening of expression lines and wrinkling. Crustingnoted on the skin
would indicate a potential complication.
5. The home health nurse is visiting a client for the first time. While assessing the clients
medication history, it is noted that there are 19 prescriptions and several over-the-counter
medications that the client has been taking. Which intervention should the nurse take
first?
A. Check for medication interactions
B. Determine whether there are medication duplications
C. Call the prescribing health care provider and report polypharmacy
D. Determine whether a family member supervises medication administration
CORRECT ANSWER: B
Polypharmacy is a concern in the older client. Duplication of medications needs to be identified
before medication interactions can be determined because the nurse needs to know what the
client is taking. Asking about medication administration supervision may be part of the
assessment but is not the first priority. The phone call to the health care provider is the
intervention after all other information has been collected.
6. The visiting nurse observes that the older male client is confined by his daughter-in-law
to his room. When the nurse suggests that he walk to the den to join his family, he says,
Im in everyones way; my daughter-in-law needs me to stay here. Which is the most
important action for the nurse to take?
A. Say to the daughter-in-law, Confining you father-in-law to his room is
inhumane.
B. Suggest to the client and daughter-in-law that they consider a nursing home for
the client
C. Say nothing because it is best for the nurse to remain neutral and wait to be asked
for help
D. Suggest appropriate resources to the client and daughter-in-law, such as respite
care and a senior citizens center
CORRECT ANSWER: D
Assisting clients and families to become aware of available community support system is a role
and responsibility of the nurse. Observing that the client has begun to be confined to his room
makes it necessary for the nurse to intervene legally and ethically, so option C is not appropriate
and is passive in terms of advocacy. Option B suggests committing the client to a nursing home
and is a premature action on the nurses part. Although the data provided tell the nurse that this
client requires nursing care, the nurse does not know the extent of the nursing care required.

Option A is incorrect and judgmental.


7. The nurse is providing an educational session to new employees, and the topic is abuse of
the older client. The nurse helps the employees identify which client as most typically a
victim of abuse?
A. A 75-year-old man who has moderate hypertension
B. A 68-year-old man who has newly diagnosed cataracts
C. A 90-year-old woman who has advanced Parkinsons disorder
D. A 70-year-old woman who has early diagnosed Lyme disease
CORRECT ANSWER: C
Elder abuse includes physical, sexual, or psychological abuse, misuse of property, and violation
of rights. The typical abuse victim is a woman of advanced age with few social contacts and at
least one physical or mental impairment that limits her to perform activities of daily living. In
addition, the client usually lives alone or with the abuser, and depends on the abuser for care.
8. The long-term care nurse is performing assessments on several of the residents. Which
are normal age-related physiological change(s) the nurse expects to note? Select all that
apply.
A. Increased heart rate
B. Decline in visual acuity
C. Decreased respiratory rate
D. Decline in long-term memory
E. Increased susceptibility to urinary tract infections
F. Increased incidence of awakening after sleep onset
CORRECT ANSWER: B, E, F
Anatomical changes to the eye affect the individuals visual ability, leading to potential problems
with activities of daily living. Light adaptation and visual fields are reduced. Although lung
function may decrease, the respiratory rate usually remains unchanged. Heart rate decreases
and heart valves thicken. Age-related changes that affect the urinary tract increase an older
clients susceptibility to UTI. Short-term memory may decline with age, but long-term memory
usually is maintained. Change in sleep patterns is a consistent age-related change. Older
persons experience an increased incidence of awakening after sleep onset.
9. The home care nurse is visiting an older client whose spouse died 6 months ago. Which
behavior by the client indicates ineffective coping?
A. Neglecting personal grooming
B. Looking at old snapshots of the family
C. Participating in a senior citizens program
D. Visiting their spouses grave once a month
CORRECT ANSWER: A
Coping mechanisms are behaviors used to decrease stress and anxiety. In response to a death,
ineffective coping is manifested by an extreme behavior that in some cases may be harmful to the
individual physically or psychologically. The correct option is indicative of a behavior that

identifies an ineffective coping behavior in the grieving process.


10. The nurse is setting up an education session with an 85-year-old patient who will be
going home on anticoagulant therapy. Which strategy would reflect consideration of
aging changes that may exist with this patient?
A. Show a colorful video about anticoagulation therapy.
B. Present all the information in one session just before discharge.
C. Give the patient pamphlets about the medications to read at home.
D. Develop large-print handouts that reflect the verbal information presented.
CORRECT ANSWER: D
Option D addresses altered perception in two ways. First, by using visual aids to reinforce
verbal instructions, one addresses the possibility of decreased ability to hear high-frequency
sounds. By developing the handouts in large print, one addresses the possibility of decreased
visual acuity. Option A does not allow discussion of the information; furthermore, the text and
print may be small and difficult to read and understand.
11.When developing the plan of care for an older adult who is hospitalized for
an acute illness, the nurse should:
A. Use a standardized geriatric nursing care plan.
B. Plan for likely long-term-care transfer to allow additional time for
recovery.
C. Consider the preadmission functional abilities when setting patient
goals.
D. Minimize activity level during hospitalization.

CORRECT ANSWER: C
The plan of care for older adults should be individualized and based on the
patients current functional abilities. A standardized geriatric nursing care plan is
unlikely to address individual patient needs and strengths. A patients need for
discharge to a long-term-care facility is variable. Activity level should be
designed to allow the patient to retain functional abilities while hospitalized and
also to allow any additional rest needed for recovery from the acute process.

12.Which information obtained by the home health nurse when making a visit

to an 88-year-old with mild forgetfulness is of the most concern?


A. The patient's son uses a marked pillbox to set up the patient's
medications weekly.
B. The patient has lost 10 pounds (4.5 kg) during the last month.
C. The patient is cared for by a daughter during the day and stays with
a son at night.
D. The patient tells the nurse that a close friend recently died.

CORRECT ANSWER: B
A 10-pound weight loss may be an indication of elder neglect or depression and requires further
assessment by the nurse.
13.In reviewing changes in the older adult, the nurse recognizes that which of
the following statements related to cognitive functioning in the older
client is true?
A. Delirium is usually easily distinguished from irreversible dementia.
B. Therapeutic drug intoxication is a common cause of senile dementia.
C. Reversible systemic disorders are often implicated as a cause of
delirium.
D. Cognitive deterioration is an inevitable outcome of the human aging
process.

CORRECT ANSWER: C
Delirium is a potentially reversible cognitive impairment that is often due to a physiological
cause such as an electrolyte imbalance, cerebral anoxia, hypoglycemia, medications, tumors,
cerebrovascular infection, or hemorrhage.
14.Which of the following interventions should be taken to help an older
client to prevent osteoporosis?
A. Decrease dietary calcium intake
B. Increase sedentary lifestyles
C. Increase dietary protein intake.
D. Encourage regular exercise.

CORRECT ANSWER: D.
Encourage regular exercise. Key word in question is prevent. Weight-bearing exercises helps to
fight off degeneration of bone in osteoporosis
15.A long-term care facility sponsors a discussion group on the administration
of medications. The participants have a number of questions concerning
their medications. The nurse responds most appropriately by saying:
A. "Don't worry about the medication's name if you can identify it by
its color and shape."
B. "Unless you have severe side effects, don't worry about the minor
changes in the way you feel."
C. "Feel free to ask your physician why you are receiving the
medications that are prescribed for you."
D. "Remember that the hepatic system is primarily responsible for the
pharmacotherapeutics of your medications."

CORRECT ANSWER: C
The nurse should encourage the older adult to question the physician and/or pharmacist about
all prescribed drugs and over-the-counter drugs. The older adult should be taught the names of
all drugs being taken, when and how to take them, and the desirable and undesirable effects of
the drugs.
16.In performing a physical assessment for an older adult, the nurse
anticipates finding which of the following normal physiological changes of
aging?
A. Increased perspiration
B. Increased airway resistance
C. Increased salivary secretions
D. Increased pitch discrimination

CORRECT ANSWER: B
Normal physiological changes of aging include increased airway resistance in the older adult.
The older adult would be expected to have decreased perspiration and drier skin as they
experience glandular atrophy (oil, moisture, sweat glands) in the integumentary system. The
older adult would be expected to have a decrease in saliva. A normal physiological change of the
older adult related to hearing is a loss of acuity for high-frequency tones (presbycusis).
17. To minimize confusion to the elderly hospitalized client, the nurse should:
A. Provide sensory stimulation by varying daily routine
B. Keep the room brightly lit and the television on to provide orientation to time
C. Encourage visitors to limit visitation to phone calls to avoid overstimulation
D. Provide explanations in a calm, caring manner to avoid overstimulation
CORRECT ANSWER: D
Hospitalized elderly clients frequently become confused. Providing simple explanations in a
calm, caring manner will help minimize anxiety and confusion. Answers A and B will increase
clients confusion, and answer C is incorrect because personal visits from family and friends
would benefit the client.
18. The nurse cares for an elderly patient with moderate hearing loss. The nurse should teach
the patients family to use which of the following approaches when speaking to the
patient?
A. Raise your voice until the patient is able to hear you.
B. Face the patient and speak quickly using a high voice.
C. Face the patient and speak slowly using a slightly lowered voice.
D. Use facial expressions and speak as you would normally.
CORRECT ANSWER: C
(A) Raise your voice until the patient is able to hear you would result in high tones patient
unable to hear
(B) Face the patient and speak quickly using a high voice usually unable to hear high tones
(C) Face the patient and speak slowly using a slightly lowered voice CORRECT: also
decrease background noise; speak at a slow pace, use nonverbal cues
(D) Use facial expressions and speak as you would normally nonverbal cues help, but need
low tones
19. The nurse knows that a 60-year-old female clients susceptibility to osteoporosis is most
likely related to:
A. Lack of exercise
B. Hormonal disturbances
C. Lack of calcium
D. Genetic predisposition
CORRECT ANSWER: B
After menopause, women lack hormones necessary to absorb and utilize calcium. Doing weightbearing exercises and taking calcium supplements can help to prevent osteoporosis but are not

causes, so answers A and C are incorrect. Body types that frequently experience osteoporosis are
thin Caucasian females, but they are not most likely related to osteoporosis, so answer D is
incorrect.
20. All of the following characteristics would indicate to the nurse that an elder client might
experience undesirable effects of medicines except:
A. Increased oxidative enzyme levels.
B. Alcohol taken with medication.
C. Medications containing magnesium.
D. Decreased serum albumin.
CORRECT ANSWER: A
Oxidative enzyme levels decrease in the elderly, which affects the disposition of medication and
can alter the therapeutic effects of medication. Alcohol has a smaller water distribution level in
the elderly, resulting in higher blood alcohol levels. Alcohol also interacts with various drugs to
either potentate or interfere with their effects. Magnesium is contained in a lot of medications
elder clients routinely obtain over the counter. Magnesium toxicity is a real concern. Albumin is
the major drug-binding protein. Decreased levels of serum albumin mean that higher levels of
the drug remain free and that there are less therapeutic effects and increased drug interactions.
21. The nurse is caring for residents in a long term care setting for the elderly. Which of the
following activities will be MOST effective in meeting the growth and development
needs for persons in this age group?
A. Aerobic exercise classes
B. Transportation for shopping trips
C. Reminiscence groups
D. Regularly scheduled social activities
CORRECT ANSWER: C
According to Eriksons theory, older adults need to find and accept the meaningfulness of their
lives, or they may become depressed, angry, and fear death. Reminiscing contributes to
successful adaptation by maintaining self-esteem, reaffirming identity, and working through
loss.
22. Mrs. Parker, a 70-year-old woman with severe macular degeneration, is admitted to the
hospital the day before scheduled surgery. The nurses preoperative goals for Mrs. M.
would include:
A.
B.
C.
D.

Independently ambulating around the unit.


The routine preoperative education materials.
Maneuvering safely after orientation to the room.
Using a bedpan for elimination needs.

CORRECT ANSWER: C
Independently ambulating around the unit is not appropriate because the unit environment can
change and injury could result. Assistance is necessary because of the clients visual deficit. It is
unlikely the client can see well enough to read the materials. Maneuvering safely after

orientation to the room is a realistic goal for a person with impaired vision. Orienting the client
to the room should help the client to move safely. Using the bedpan is an unnecessary restriction
on the client as she can be oriented to the bathroom or to call for assistance.
23. James Perez, a nurse on a geriatric floor, is administering a dose of digoxin to one of his
patients. The woman asks why she takes a different pill than her niece, who also has heart
trouble. James replies that as people get older, liver and kidney function decline, and if
the dose is as high as her nieces, the drug will tend to:
A. Have a shorter half-life.
B. Accumulate.
C. Have decreased distribution.
D. Have increased absorption.
CORRECT ANSWER: B
The decreased circulation to the kidney and reduced liver function tend to allow drugs to
accumulate and have toxic effects.
24. The nurse is caring for an elderly client who has been diagnosed as having sundown
syndrome. He is alert and oriented during the day but becomes disoriented and disruptive
around dinnertime. He is hospitalized for evaluation. The nurse asks the client and his
family to list all of the medications, prescription and nonprescription, he is currently
taking. What is the primary reason for this action?
A. Multiple medications can lead to dementia
B. The medications can provide clues regarding his medical background
C. Ability to recall medications is a good assessment of the clients level of
orientation.
D. Medications taken by a client are part of every nursing assessment.
CORRECT ANSWER: A
Drugs commonly used by elderly people, especially in combination, can lead to dementia.
Assessment of the medication taken may or may not provide information on the clients medical
background. However, this is not the primary reason for assessing medications in a client who is
exhibiting sundown syndrome. Ability to recall medications may indicate short-term memory and
recall. However, that is not the primary reason for assessing medications in a client with
sundown syndrome. Medication history should be a part of the nursing assessment. In this client
there is an even more important reason for evaluating the medications taken.
25. The nurse needs to carefully assess the complaint of pain of the elderly because older
people
A. Are expected to experience chronic pain
B. Have a decreased pain threshold
C. Experience reduced sensory perception
D. Have altered mental function
CORRECT ANSWER: C
Degenerative changes occur in the elderly. The response to pain in the elderly maybe lessened
because of reduced acuity of touch, alterations in neural pathways and diminished processing of
sensory data.

NUTRITION

1. The nurse is teaching a client who has iron deficiency anemia about foods she should
include in her diet. The nurse determines that the client understands the dietary
modifications if she selects which items from her menu?
A. Nuts and milk
B. Coffee and tea
C. Cooked rolled oats and fish
D. Oranges and dark green leafy vegetables
CORRECT ANSWER: D
Dark green leafy vegetables are a good source of iron and oranges are a good source of Vitamin
C, which enhance iron absorption. All other options are not food sources that are high in iron
and Vitamin C.
2. A client who is recovering from surgery has been advanced form a clear liquid diet to a
full liquid diet. The client is looking forward to the diet change because he has been
bored with the clear liquid diet. The nurse should offer which full liquid item to the
client?
A. Tea
B. Gelatin
C. Custard
D. Ice pop
CORRECT ANSWER: C
Full liquid food items include items such as plain ice cream, sherbet, breakfast drinks, milk,
pudding and custard, soups that are strained, refined cooked cereals, and strained vegetable
juices. A clear liquid diet consists of food that are relatively transparent. The food items in the
incorrect options are clear liquids.
3. The nurse is planning to teach a client with malabsorption syndrome about the necessity
of the following a low-fat diet. The nurse develops of high-fat foods to avoid and should
include which food item on the list?
A. Oranges
B. Broccoli
C. Cream cheese
D. Broiled haddock
CORRECT ANSWER: C
Fruits and vegetables tend to be lower in fat because they do not come from animal sources.
Broiled haddock is also naturally lower in fat. Cream cheese is high-fat food.
4. The nurse is conducting a dietary assessment on a client who is on a vegan diet. The
nurse should provide dietary teaching and should focus on foods high in which vitamin
that may be lacking in vegan diet?
A. Vitamin A
B. Vitamin B12
C. Vitamin C
D. Vitamin E

CORRECT ANSWER: B
Vegans do not consume any animal products. Vitamin B12 is found in animal products and
therefore more likely be lacking in a vegan diet. Vitamins A, C and E are found in fresh fruits and
vegetables, which are consumed in a vegan diet.
5. Which nursing action is essential prior to initiating a new prescription for 500mL of fat
emulsion (lipids) to infuse at 50mL/hour?
A. Ensure that the client does not have diabetes
B. Determine whether the client has an allergy to eggs
C. Ass regular insulin to the fat emulsion using aseptic technique
D. Contact the health care provider to have a central line inserted for fat emulsion
infusion
CORRECT ANSWER: B
The client beginning infusions of fat emulsions must be first assessed for known allergies to eggs
to prevent anaphylaxis. Egg yolk is a component of the solution and provides emulsification. The
remaining options are unnecessary and are not related to the administration of fat emulsion.
6. A client with hypertension has been told to maintain a diet low in sodium. The nurse who
is teaching this client about foods that are allowed should include which food item in a
list provided by the client?
A. Tomato soup
B. Broiled soup
C. Instant oatmeal
D. Summer squash
CORRECT ANSWER: D
Foods that are in lower in sodium include fruits and vegetables (summer squash), because they
do not contain physiological saline. Highly processed or refined foods (tomato soup, broiled
soup, instant oatmeal) are higher in sodium unless their food labels specifically state low
sodium. Saltwater fish and shellfish are high in sodium.
7. The nurse is caring for a client with cirrhosis of the liver. To minimize the effects of the
disorder, the nurse teaches the clients about foods that are high in thiamine. The nurse
determines that the client has the best understanding of the dietary measures to follow if
the client states an intention to increase the intake of which food?
A. Pork
B. Milk
C. Chicken
D. Broccoli
CORRECT ANSWER: A
The client with cirrhosis needs to consume foods high in thiamine. Thiamine is present in a
variety of foods of plant and animal origin. Pork products are especially rich in this vitamin.
Other good food sources include nuts, whole grain cereals, and legumes. Milk contains vitamins

A, D, and B2. Poultry contains niacin. Broccoli contains vitamins C, E, and K and folic acid.
8. The nurse instructs a client with chronic kidney disease who is receiving hemodialysis
about dietary modifications. The nurse determines that the client understands these
dietary modifications if the client selects which items from the dietary menu?
A. Cream of wheat, blueberries, coffee
B. Sausage and eggs, banana, orange juice
C. Bacon, cantaloupe melon, tomato juice
D. Cured pork, grits, strawberries, orange juice
CORRECT ANSWER: A
The diet for a client with chronic kidney disease who is receiving hemodialysis should include
controlled amounts of sodium, phosphorus, calcium, potassium and fluids, which is indicated in
the correct option. The food items in the remaining options are high in sodium, phosphorus or
potassium.
9. A client is recovering from abdominal surgery and has a large abdominal wound. The
nurse should encourage the client to eat which food item that is naturally high in vitamin
C to promote wound healing?
A. Milk
B. Oranges
C. Banana
D. Chicken
CORRECT ANSWER: B
Citrus fruits and juices are especially high in Vitamin C. bananas are high in potassium. Meats
and dairy products are two food groups that are high in the B vitamins.
10. A postoperative client has been placed on a clear liquid diet. The nurse should provide the
client with which item(s) that re allowed to be consumed on this diet? Select all that
apply.
A. Broth
B. Coffee
C. Gelatin
D. Pudding
E. Vegetable juice
F. Pureed vegetables
CORRECT ANSWER: A, B, C
A clear liquid diet consists of foods that are relatively transparent to light and are clear and
liquid at room and body temperature. These foods include items such as water, bouillon, clear
broth, carbonated beverages, gelatin, hard candy, lemonade, ice pops, and regular or
decaffeinated coffee or tea. The incorrect items are for a full liquid diet.
11. The nurse is instructing a client with hypertension on the importance of choosing foods
low in sodium. The nurse should teach the client to limit intake of which food?

A.
B.
C.
D.

Apples
Bananas
Smoked sausage
Steamed vegetables

CORRECT ANSWER: C
Smoked foods are high in sodium, which is noted in the correct option. The remaining options
are fruits and vegetables which are low in sodium.
12. Clients receiving hypertonic tune feedings most commonly develop diarrhea because of:
A. Increased fiber intake
B. Bacterial contamination
C. Inappropriate positioning
D. High osmolality of the feedings
CORRECT ANSWER: D
The increased osmolarity (concentration) of many formulas draws fluid into the interstitial tract,
which would cause diarrhea; such feedings may need to be diluted initially until the client
develops tolerance.
13. A client has had abdominal surgery and the physician has ordered a bland diet 3 days
post-surgery. Which of the following trays would have portions removed because it does
not adhere to the dietary regimen?
A. Scrambled eggs, cereal, and white toast
B. Baked potato, cottage cheese, and coffee
C. Cream soup, jello, and white toast
D. Cooked cereal, boiled egg, and milk
CORRECT ANSWER: B
Coffee is one food eliminated from a bland diet because it is chemically irritating to the stomach.
All of the other foods are allowed on a bland diet. Other foods eliminated are raw, spicy, gas
forming, very hot or very cold foods, alcohol, and carbonated drinks.
14. In an initial assessment of a client, the nurse is looking for physical indicators of good
nutrition in the client. What are the physical characteristics that reflect good nutrition?

Select all that apply.


A. Alert expression
B. Clear skin
C. Firm flesh
D. Greasy, blemished complexion with poor color
E. Shiny hair
F. Well-developed bone structures
CORRECT ANSWER: A, B, C, E, F
Shiny hair, clear skin, firm flesh, well developed bone structures, and an alert expression are
some of the characteristics of good nutritional status. Greasy, blemished complexion with poor
color is one characteristic of poor nutritional status.
15. The nurse is educating a client about the need for adequate protein intake. What reason
will the nurse give the client for the importance of this nutrient?
A. Protein helps regulate circulation and respiration
B. Protein helps regulate digestion and elimination
C. Protein is necessary to build and repair tissues
D. Protein is a good source of fiber
CORRECT ANSWER: C
Protein is necessary to build and repair body tissues. Protein does not contain fiber. Vitamins,
minerals, and water help regulate respiration, circulation, and digestion and elimination.
16. The nurse provides dietary instruction to the parents of a child with a diagnosis of cystic
fibrosis. The nurse should tell the parents that which diet plan should be followed?
A. Fat-free
B. Low in sodium
C. Low in protein
D. High in calories
CORRECT ANSWER: D
Children with cystic fibrosis are managed with a high-calorie, high-protein diet; pancreatic
enzyme replacement therapy; fat-soluble vitamin supplements; and if nutritional problems are
severe, nighttime gastrostomy feedings or parenteral nutrition. Fats are not restricted unless
steatorrhea cannot be controlled by increased pancreatic enzymes. Sodium intake is unrelated to
this disorder.

17. The nurse is teaching a client with acute kidney injury to include proteins in the diet that
are considered high quality or complete proteins. The nurse determines that the client
needs further teaching if he indicates that which food item is considered high quality?
A. Fish
B. Eggs
C. Chicken
D. Broccoli
CORRECT ANSWER: D
High quality or complete proteins comes from animal sources and include such foods as eggs,
chicken, meat and fish. Low-quality or incomplete proteins are derived from plant sources and
include vegetables and foods made from grains. Because the renal diet is limited in protein, it is
important that the proteins ingested are of high quality.
18. The nurse instructs a client who is hospitalized and is on a low-fat diet. Which menu does
the nurse provide for the client?
A. Shrimp, avocado, and tomato salad
B. Calfs liver, potato salad, and sherbet
C. Lean steak, mashed potatoes, and gravy
D. Turkey breast, boiled rice, and strawberries
CORRECT ANSWER: D
Turkey breast without the skin, boiled rice, and strawberries offer the client nourishing foods
that are low in fat. Some sources of fat include meats, avocado, salad dressing, mayonnaise,
butter, cheese and bacon. Options A, B. and C contain high-fat foods.
19. A client with gastroesophageal reflux disease (GERD) has just received a breakfast tray.
The nurse setting up the tray for the client notices that which is the only food that will
increase the lower esophageal sphincter (LES) pressure and thus lessen the clients
symptoms?
A. Coffee
B. Nonfat milk
C. Fresh scrambled eggs
D. Whole wheat toast with butter
CORRECT ANSWER: B
Foods that increase the LES pressure will decrease reflux and lessen the symptoms of GERD.
The food substance that will increase LES pressure is nonfat milk. The other substances listed
decrease the LES pressure, thus increasing reflux symptoms. Aggravating substances include
chocolate, coffee, fatty foods, and alcohol and should be avoided in the diet of a client with
GERD.
20. A client with chronic kidney disease (CKD) has received dietary counseling about
potassium restriction in the diet. The nurse determines that the client has learned the
information correctly if the client states that he will do what when preparing vegetables?
A. Eat only fresh vegetables

B. Boil them and discard the water


C. Use salt substitute on them liberally
D. Buy frozen vegetables whenever possible
CORRECT ANSWER: B
The potassium content of vegetables can be reduced by boiling them and discarding the cooking
water. Options A and D are incorrect. Clients with CKD should avoid the use of salt substitutes
altogether because they tend to be high in potassium content.
21. The nurse is assisting the client with hepatic encephalopathy to fill out the dietary menu.
The nurse advises the client to avoid which entre items that could aggravate the clients
condition?
A. Tomato soup
B. Fresh fruit plate
C. Vegetable lasagna
D. Ground beef patty
CORRECT ANSWER: D
Clients with hepatic encephalopathy have impaired ability to convert ammonia to urea and must
limit intake of protein and ammonia-containing foods in the diet. The client should avoid foods
such as chicken, beef, ham, cheese, milk, peanut butter and gelatin. The food items in options A,
B, and C are acceptable to eat.
22. A client with colostomy is complaining of gas building up in the colostomy bag. The
nurse instructs the client that which food items should be consumed to best prevent this
problem? Select all that apply.
A. Yogurt
B. Broccoli
C. Cabbage
D. Crackers
E. Cauliflower
F. Toasted bread
CORRECT ANSWER: A, D, F
Consumption of yogurt, crackers, and toast or crackers can help prevent gas. Gas-forming foods
include broccoli, mushrooms, cauliflower, onion, peas, and cabbage. These should be avoided by
the client with a colostomy until tolerance to them is determined.
23. The nurse is teaching a pregnant client about nutrition. The nurse should include which
information in the clients teaching plan?
A. All mothers are at high risk for nutritional deficiencies
B. Calcium intake is not necessary until the third trimester
C. Iron supplements are not necessary unless the mother has iron deficiency anemia
D. The nutritional status of the mother significantly influences fetal growth and
development
CORRECT ANSWER: D

Poor nutrition during pregnancy can negatively influence fetal growth and development.
Although pregnancy poses some nutritional risk for the mother, not all clients are at high risk.
Calcium intake is critical during the third trimester but must be increased from the onset of
pregnancy. Intake of dietary iron is insufficient for the majority of pregnant women, and iron
supplements are routinely prescribed.
24. The nurse provides dietary instructions to a client who needs to limit the intake of
sodium. The nurse instructs the client that which food items must be avoided because of
their high-sodium content? Select all that apply.
A. Ham
B. Apples
C. Broccoli
D. Soy sauce
E. Asparagus
F. Cantaloupe
CORRECT ANSWER: A, D
Food highest in sodium include table salt, some cheeses, soy sauce, cured pork, canned foods
because of the preservatives and foods such as cold cuts. Fruits and vegetables contain minimal
amount of sodium.
25. Which assessment is the most important for the nurse to make before advancing a client
from liquid to solid food?
A. Bowel sounds
B. Chewing ability
C. Current appetite
D. Food preferences
CORRECT ANSWER: B
The nurse need to assess the clients chewing ability before advancing a client from liquid to
solid food. It may be necessary to modify a clients diet to a soft or mechanical chopped diet of
the client has difficulty chewing because of the risk of aspiration. Bowel sound should be present
before introducing any diet, including liquids. Appetite will affect the amount of food eaten, but
not the type of diet prescribed. Food preferences should be ascertained on admission assessment

DELEGATION

1. The nurse is assigned to care for four clients. In planning client rounds, which client
should the nurse assess first?
A. A client scheduled for a chest x-ray
B. A client requiring daily dressing changes
C. A postoperative client preparing for discharge
D. A client receiving nasal oxygen who had difficulty breathing during the previous
shift
CORRECT ANSWER: D.
Airway is always the highest priority and the nurse would attend to the client who has been
experiencing an airway problem first. The clients described in options A, B, C have needs that
would be identified as intermediate priorities.
2. The nurse employed in an emergency department is assigned to triage clients coming to
the emergency department for treatment on the evening shift. The nurse should assign
priority to which client?
A. A client complaining of muscle aches, a headache, and malaise
B. A client who twisted her ankle when she fell while rollerblading
C. A client with a minor laceration on the index finger sustained while cutting an
eggplant
D. A client with chest pain who states that he just ate pizza that was made with a
very spicy sauce
CORRECT ANSWER: D.
In an emergency department, triage involves brief client assessment to classify clients according
to their need for care and includes establishing priorities of care. The type of illness or injury,
the severity of the problem, and the resources available govern the process. Clients with trauma,
chest pain, severe respiratory distress or cardiac arrest, limb amputation, and acute neurological
deficits, or who have sustained chemical splashes to the eyes are classified emergent and are the
number 1 priority. Clients with conditions such as a simple fracture, asthma without respiratory
distress, fever, hypertension, abdominal pain, or a renal stone have urgent needs and are
classified as number 2 priority. Clients with conditions such as a minor laceration , sprain, or
cold symptoms are classified as nonurgent and are a number 3 priority.
3. A nursing graduate is attending an agency orientation regarding the nursing model of
practice implemented in the health care facility. The nurse is told that the nursing model
is a team nursing approach. The nurse understands that planning care delivery will be
based on which characteristic of this type of nursing model of practice?
A. A task approach method is used to provide care to clients
B. Managed care concepts and tools are used in providing client care
C. A single registered nurse is responsible for providing care to a group of clients
D. A registered nurse leads nursing personnel in providing care to a group of clients
CORRECT ANSWER: D.
In team nursing, nursing personnel are lead by a registered nurse leader in providing care to a
group of clients. Option A identifies functional nursing. Option B identifies a component of case

management. Option C identifies primary nursing (relationship-based practice).


4. A nurse has received the assignment for the day shift. After making initial rounds and
checking all of the assigned clients, which client should the nurse plan to care first?
A. A client who is ambulatory
B. A client scheduled for physical therapy at 1pm
C. A client with a fever who is diaphoretic and restless
D. A postoperative client who has just received pain medication
CORRECT ANSWER: C.
The nurse should plan to care for the client who has a fever and is diaphoretic and restless first
because this clients needs are the priority. The client who is ambulatory and the client
scheduled for physical therapy later in the day do not have priority needs related to care.
Waiting for pain medication to take effect before providing care to the postoperative client is
best.
5. The nurse is giving a bed bath to an assigned client when an unlicensed assistive
personnel (UAP) enters the clients room and tells the nurse that another assigned client is
in pain and needs pain medication. Which is the most appropriate nursing action?
A. Finish the bed bath and then administer the pain medication to the other client.
B. Ask the UAP to find out when the last pain medication was given to the client.
C. Ask the UAP to tell the client in pain that medication will be administered as soon
as the bed bath is complete
D. Cover the client, raise the side rails, tell the client that you will return shortly, and
administer the pain medication to the other client.
CORRECT ANSWER: D.
The nurse is responsible for the care provided to assigned clients. The appropriate action in this
situation is to provide safety to the client who is receiving the bed bath and prepare to administer
the pain medication. Options A and C delay the administration of the medication to the client in
pain. Option B is not a responsibility of the UAP.
6. The nurse manager has implemented a change in the method of the nursing delivery
system from functional to team nursing. An unlicensed assistive personnel (UAP) is
resistant to the change and is not taking an active part in facilitating the process of
change. Which is the best approach in dealing with the UAP?
A. Ignore the resistance
B. Exert coercion on the UAP
C. Provide a positive reward system for the UAP
D. Confront the UAP to encourage verbalization of feelings regarding the change
CORRECT ANSWER: D.
Confrontation is an important strategy to meet resistance head on. Face-to-face meetings to
confront the issue at hand will allow verbalization of feelings, identification of problems and
issues and development of strategies to solve the problem. Option A will not address the
problem. Option B may produce additional resistance. Option C may provide a temporary

solution to the resistance, but will not address the concern specifically.
7. The registered nurse is planning the client assignments for the day. Which is the most
appropriate assignment for an unlicensed assistive personnel (UAP)?
A. A client requiring a colostomy irrigation
B. A client receiving continuous tube feedings
C. A client who requires urine specimen collections
D. A client with difficulty swallowing food and fluids
CORRECT ANSWER: C.
The nurse must determine the most appropriate assignment based on the skills of the staff
member and needs of the client. In this case, the most appropriate assignment for the UAP would
be to care for the client who requires urine specimen collections. The UAP is skilled in this
procedure. Colostomy irrigations and tube feedings are not performed by unlicensed personnel.
The client with difficulty swallowing food and fluids is at risk for aspiration.
8. The nurse employed in a long term care facility is planning assignments for the clients on
a nursing unit. The nurse needs to assign four clients and has a licensed practical
(vocational) nurse and three unlicensed assistive personnel (UAP) on a nursing team.
Which client would the nurse most appropriately assign to the licensed practical
(vocational) nurse?
A. A client who requires a bed bath
B. An older client requiring frequent ambulation
C. A client who requires hourly vital sign measurements
D. A client requiring abdominal wound irrigations and dressing changes every 3
hours
CORRECT ANSWER: D.
When delegating nursing assignments, the nurse needs to consider the skills and educational
level of the nursing staff. Giving a bed bath, assisting with frequent ambulation, and taking vital
signs can be provided most appropriately by the UAP. The licensed practitioner (vocational)
nurse is skilled in wound irrigation and dressing changes and most appropriately would be
assigned to the client who needs this care.
9. The nurse should use which guideline(s) to plan delegation and assignment-making
activities? Select all that apply.
A. Ensuring client safety
B. Requests from the staff
C. The clustering of the room in the unit
D. The number of anticipated client discharges
E. Client needs and workers needs and abilities
CORRECT ANSWERS: A, E
There are guidelines that the nurse should use when delegating and planning assignments. These
include the following: ensure client safety, be aware of individual variations in work abilities;
determine which tasks can be delegated and to whom; match the task to the delegatee on the

basis of the nurse practice act and appropriate position descriptions; provide directions that are
clear, concise, accurate, and complete; validate the delegatees understanding of the directions;
communicate of a feeling of confidence to the delegatee, and provide feedback promptly after the
task is performed; and maintain continuity of care as much as possible when assigning client
care. Staff requests, convenience as in clustering client rooms, and anticipated changes in unit
census are not specific guidelines to use when delegating and planning assignments.
10. The nurse and unlicensed assistive personnel (UAP) are caring for a client with rightsided paralysis. Which action by the UAP requires the nurse to intervene?
A. The assistant places a gait belt around the clients waist prior to ambulating.
B. The assistant places the client on the back with the clients head to the side.
C. The assistant places her hand under the clients right axilla to help him/her move
up in bed.
D. The assistant praises the client for attempting to perform ADLs independently.
CORRECT ANSWER: C.
This action is inappropriate and would require intervention by the nurse because pulling on a
flaccid shoulder joint could cause shoulder dislocation; as always use a lift sheet for the client
and nurse safety. All the other actions are appropriate.
11. The registered nurse (RN) is creating a plan for the assignments for the day and is leading
a team composed of a licensed practical nurse (LPN) and an unlicensed assistive
personnel (UAP). Based on licensure, which client is most appropriate to assign to the
LPN?
A. A client with dementia
B. A 1-day postoperative mastectomy client
C. A client who requires some assistance with bathing
D. A client who requires some assistance with ambulation
CORRECT ANSWER: B.
Assignment of task must be implemented based on the job description of the LPN and UAP, the
level of education and clinical competence, and state law. The 1-day postoperative mastectomy
client will need care that requires the skill of a licensed nurse. The UAP has the skills to care for
a client requiring noninvasive care such as a client with dementia, a client who requires some
assistance with bathing, and a client who requires some assistance with ambulation.
12. The nurse is creating a plan for delegating unit activities for the day. Which activities
should the nurse delegate to the unlicensed assistive personnel (UAP)? Select all that
apply.
A. Deliver fresh water to clients.
B. Empty urine out of Foley bags.
C. Take temperatures, pulses, respirations, and blood pressures.
D. Count the substance control medications in the opioid medication supply.
E. Check the crash cart (cardiopulmonary resuscitation cart) for necessary supplies
using a checklist.
F. Check all intravenous (IV) solution bags on clients receiving IV therapy for the

remaining amounts of solution in the bags.


CORRECT ANSWER: A, B, C
Delegation is the transfer of responsibility for the performance of an activity or task while
retaining accountability for the outcome. When delegating an activity, the nurse must consider
the educational preparation and experience of the individual. UAP are trained to perform
noninvasive tasks and those that meet basic client needs. Although UAP are trained in
performing cardiopulmonary resuscitation, the UAP is not trained to check a crash cart, and this
activity must be assigned to a licensed nurse. Any activities related to medications and IV
therapy must be delegated to a licensed nurse.
13. In which situation is the nurse utilizing autocratic leadership style?
A. The nurse manager provides solution for a unit problem.
B. The nurse manager allows the staff to solve their own unit problem.
C. The nurse manager proposes several alternatives and has the unit staff vote on the
best proposal.
D. The nurse manager arranges for a staff meeting where all unit employees can
share proposals to solve a problem.
CORRECT ANSWER: A.
The autocratic style of leadership is task oriented and directive. The leader uses his or her
power and position in an authoritarian manner to set and implement organizational goals or
solutions. Decisions are made without input from the staff. The situational leadership style uses
a style depending on the situation and events. Democratic styles best empower staff toward
excellence because this style of leadership allows nurses to provide input regarding the decisionmaking process and an opportunity to grow professionally. Participatory leadership encourages
input from the staff.
14. A clinical nurse manager conducts an in-service educational session for the staff nurses
about case management. Which premise, if stated by the staff nurse, regarding case
management, should necessitate a need for further teaching?
A. Manages client care by managing the client care environment.
B. Maximizes hospital revenues while providing for optimal client care.
C. Represents a primary health prevention focus managed by a single case manager.
D. Is designed to promote appropriate use of hospital personnel and material
resources.
CORRECT ANSWER: C.
Case management represents an interdisciplinary health care delivery system to promote
appropriate use of hospital personnel and material resources to maximize hospital revenues
while providing for optimal client care. It manages client care by managing the client care
environment and includes assessment and development of a plan of care, coordination of all
services, referral and follow-up.
15. A registered nurse is delegating activities to the nursing staff. Which activities can be
safely assigned to the unlicensed assistive personnel (UAP)? Select all that apply.

A.
B.
C.
D.
E.

Collecting a urine specimen from a client.


Obtaining frequent oral temperatures on a client.
Assessing a client who returned from the recovery room 6 hours ago.
Assisting a post cardiac catheterization client who needs to lie flat to eat lunch.
Accompanying a client being discharged too meet his spouse at the hospital exit
door.

CORRECT ANSWERS: A, B, D
Work that is delegated to others must be consistent with the individuals level of expertise and
licensure, if any. Options A, B, D do not include situations that indicate that these activities
carry foreseeable risk. The least appropriate activities for unlicensed assistive personnel would
be assessing a client and assisting the post cardiac catheterization client. The UAP is not trained
or educated to safely and accurately perform an assessment on a client. Since the post cardiac
catheterization client needs to eat while lying flat, the client is at risk for aspiration.
16. The nurse is planning the client assignments for the shift. Which client should the nurse
assign to the unlicensed assistive personnel (UAP)?
A. A client requiring dressing changes.
B. A client requiring frequent ambulation.
C. A client on a bowel management program requiring rectal suppositories and a
daily enema.
D. A client with diabetes mellitus requiring daily insulin and reinforcement of dietary
measures.
CORRECT ANSWER: B.
Assignment of tasks to UAP needs to be made based on job description, level of clinical
competence, and state law. The client described in option B has needs that can be met by UAP.
Options A, C, D involve care that requires the skill of a licensed nurse.
17. Which scenarios demonstrate the participative style of leadership? Select all that apply.
A. The nurse manager presents a problem to the staff and tells the staff to solve the
problem.
B. The nurse manager arranges unit meetings for all shifts to deal with an identified
problem.
C. The nurse manager assesses a problem and informs the staff of the solution to be
implemented.
D. The nurse manager proposes several methods of dealing with a problem and
invites team input.
E. The nurse manager proposes several solutions to a problem and has the unit staff
vote on the best option.
F. The nurse manager considers staff input related to a problem but makes the final
decision on implementation of the solution.
CORRECT ANSWERS: B, D, F
Participative leadership demonstrates an in-between style, neither authoritarian nor
democratic. In participative leadership, the manager presents an analysis of problems and

proposals for actions to team members, inviting critique and comments. The participative leader
then analyzes the comments and makes the final decision. The autocratic style of leadership is
task oriented and directive. A laissez-faire leader abdicates leadership and responsibilities,
allowing staff to work without assistance, direction, or supervision. The democratic style of
leadership involves a majority rule.
18. The nurse is preparing the client assignment for the day and needs to assign clients to a
licensed practical nurse (LPN) and unlicensed assistive personnel (UAP). Which client
should the nurse assign to the LPN because of client needs that cannot be met by UAP?
Select all that apply.
A. A client requiring frequent suctioning.
B. A client requiring dressing change to the foot.
C. A client requiring range-of-motion exercises twice daily.
D. A client requiring reinforcement of teaching about a diabetic diet
E. A client on bed rest requiring vital sign measurements every 4 hours.
F. A client requiring collection of a urine specimen for urinalysis testing.
CORRECT ANSWERS: A, B, D
Delegation is the transferring to a competent individual the authority to perform a nursing task.
When the nurse plans client assignments, he or she needs to consider the educational level and
experience of the individual and the needs of the client. The LPN is trained to perform all the
tasks indicated in the options; the clients who have the needs that cannot be met by the UAP are
those requiring suctioning, a dressing change, and reinforcement of teaching about a diabetic
diet. UAP are trained to perform range-of-motion exercises, measure vital signs, and collect a
urine specimen.
19. When the nurse manager encourages staff to provide input in the decision-making
process, which type of leadership style is the nurse manager demonstrating?
A. Autocratic
B. Situational
C. Democratic
D. Laissez-faire
CORRECT ANSWER: C.
The democratic style of leadership best empowers staff toward excellence because this style of
leadership allows nurses to provide input regarding the decision-making process and an
opportunity to grow professionally. The autocratic style of leadership is task oriented and
directive. The leader uses his or her power and position in an authoritarian manner to set and
implement organizational goals. Decisions are made without input from the staff. The situational
leadership style uses a style depending on the situation and events. The laissez-faire style allows
staff to work without assistance, direction, or supervision.
20. Which situation represents the primary nursing care delivery model?
A. The registered nurse (RN) performs all tasks needed by the individual client to
optimize health.
B. The RN provides care to 4 clients while the unlicensed assistive personnel (UAP)

is assigned to care for 2 clients.


C. The RN develops a plan of care for each client and collaborates with other staff
members assigned to the same group of clients.
D. The UAP is assigned to make beds and fill water pitchers. The RN is assigned to
administer medications.
CORRECT ANSWER: A.
In primary nursing, option A, concern is with keeping the nurse at the bedside actively involved
in care, providing goal-directed and individualized client care. Option B does not follow the
guidelines for any specific type of nursing care delivery approach. Team nursing, option C is
characterized by a high degree of communication and collaboration among members. The team
is generally led by a registered nurse, who is responsible for assessing, developing nursing
diagnoses, planning, and evaluating each clients plan of care. The functional model of care
involves an assembly-line approach to client care, with major tasks being delegated by the
charge nurse to individual staff members.
21. The nurse receives a telephone call from the emergency department and is told that a
child with a diagnosis of tonic-clonic seizures will be admitted to the pediatric unit. The
nurse prepares for the admission of the child and instructs the unlicensed assistive
personnel (UAP) to place which items at the bedside?
A. A tracheostomy set and oxygen
B. Suction apparatus and oxygen
C. An endotracheal tube and an airway
D. An emergency cart and laryngoscope
CORRECT ANSWER: B.
Tonic-clonic seizures cause tightening of all the body muscles followed by tremors. Obstructed
airway and increased oral secretions are the major complications during and after a seizure.
Suction is helpful to prevent choking and oxygen is helpful to prevent cyanosis. Options A and C
are incorrect because inserting an endotracheal tube or a tracheostomy is not performed. It is
not necessary to have an emergency cart (which contains a laryngoscope) at the bedside, but a
cart should be available in the treatment room or on the nursing unit.
22. A registered nurse in charge of the client care unit is preparing the assignments for the
day. The RN assigns unlicensed assistive personnel (UAP) to make beds and bathe one of
the clients on the unit and assigns additional UAP to fill the water pitchers and serve juice
to all of the clients. Another RN is assigned to administer all medications. Based on the
assignments designed by the RN in charge, which type of nursing care is being
implemented?
A. Team nursing
B. Primary nursing
C. Functional nursing
D. Exemplary nursing
CORRECT ANSWER: C.
The functional model of care involves an assembly-line approach to client care, with major tasks

being delegated by the charge nurse to individual staff members. Team nursing is characterized
by a high degree of communication and collaboration among members. The team is generally
led by a registered nurse, who is responsible for assessing, developing nursing diagnoses,
planning, and evaluating each clients plan of care. In primary nursing, concern is with keeping
the nurse at the bedside actively involved in care, providing goal-directed and individualized
client care. In an exemplary model nursing of team nursing, each staff member works fully
within the realm of educational and clinical experience in an effort to provide comprehensive
individualized client care. Each staff member is accountable for client care and outcomes of
care.
23. Which are the characteristics of case management? Select all that apply.
A. A case manager usually does not provide direct care.
B. Critical pathways and CareMaps are types of case management.
C. A case manager does not need to be concerned with standards of cost
management.
D. A case manager collaborates with and supervises the care delivered by other staff
members.
E. The evaluation process involves continuous monitoring and analysis of the needs
of the client and services provided.
F. A case manager coordinates a hospitalized clients acute care and follows up with
the client after discharge to home.
CORRECT ANSWERS: A, D, E, F
Case management is a care management approach that coordinates health care services to
clients and their families while maintaining quality of care and minimizing health care costs.
Case managers usually do not provide direct care; instead they collaborate with and supervise
the care delivered by other staff members and actively coordinate client discharge planning. A
case manager is usually held accountable for some standard of cost management. A case
manager coordinates a hospitalized clients acute care, follows up client with the client after
discharge to home, and is responsible and accountable for appraising the overall usefulness and
effectiveness of the case managed services. This evaluation process involves continuous
monitoring and analysis of the clients needs and services provided. Critical pathways or
CareMaps are not types of case management; rather they are multidisciplinary treatment plans
used in a case management delivery system to implement timely interventions in a coordinated
care plan.
24. An unlicensed assistive personnel (UAP) is caring for a client who has an indwelling
urinary catheter. Which directions should the registered nurse provide to the UAP
regarding urinary catheter care?
A. Loop the tubing under the clients leg.
B. Place the tubing below the clients knee.
C. Use soap and water to cleanse the perineal are.
D. Keep the drainage bag above the level of the bladder.
CORRECT ANSWER: C.
Proper care of an indwelling urinary catheter is especially important to prevent infection. The

perineal area is cleansed thoroughly using mild soap and water at least three times a day and
after a bowel movement. The drainage tubing is not placed or looped under the clients leg
because this would inhibit the flow of urine. The drainage bag is kept below the level of the
bladder to prevent urine from being trapped in the bladder. The tubing must drain freely at all
times.
25. The nurse is planning care for a client with acute glomerulonephritis. Which action
should the nurse instruct the unlicensed assistive personnel (UAP) to take in the care of
the client?
A. Ambulate the client frequently.
B. Encourage a diet that is high in protein.
C. Monitor the temperature every 2 hours.
D. Remove the water pitcher from the bedside.
CORRECT ANSWER: D.
A client with acute glomerulonephritis commonly experiences fluid volume excess and fatigue.
Interventions include fluid restriction, as well as monitoring weight and intake and output. The
client may be placed on bed rest or at least encouraged to rest because a direct correlation exists
among proteinuria, hematuria, edema, and increased activity levels. The diet is high in calories
but low in protein. It is unnecessary to monitor the temperature as frequently as every 2 hours.

You might also like